Thursday, June 24, 2010

What is Naturalism? Part V: Alternative Ontologies and Why We Should Care

So far, my primary aim in my series of posts on naturalism has been to offer an account of metaphysical naturalism—that is, naturalism construed as an ontology (an account of the nature of reality). I have proposed to define metaphysical naturalism as follows: it is the view that the picture of the world that emerges through the exercise of the scientific method (roughly as described in previous posts) is an exhaustive picture of the fundamental reality behind the world of appearances. In other words, it is the view that the scientific “picture” that emerges through the scientific method describes not merely the world that is constructed by the subject as it receives and organizes data coming in from things-in-themselves; it actually describes for us what the underlying noumenal reality is like. Or, more precisely, this is what would be achieved by the picture of the world that science would ultimately produce if it were able to achieve and adequately test an ultimate model of everything. In any event, science is bringing us ever closer to an exhaustive grasp of noumenal reality.
If this is what metaphysical naturalism maintains, then metaphysical supernaturalism becomes the view that even if science were to achieve that elusive theory of everything, and test it rigorously and successfully, the picture it gave us would not be an exhaustive depiction of noumenal reality.

But if we define metaphysical supernaturalism in those terms, then a further division becomes possible, between two broad species. The first species shares the naturalist premise that the scientific method is generating an account of reality as it is in itself, and not merely as it appears to us—but it denies that science gives us all of reality. This is the view that there are two kinds of reality—what might be called material or physical reality, and then some other kind (spiritual or mental, perhaps). Descartes can be seen as epitomizing metaphysical supernaturalism in this sense, insofar as he endorses a dualistic ontology that sharply divided the physical world and the mental, and insofar as he takes science to be capable of studying only the former.

The second kind of supernaturalism, by contrast, denies the naturalist premise altogether. That is, it denies that science and its methods have taken us past the domain of phenomena. This would be Kant’s contention. He would argue that at every stage in the process of science, we are dealing with the objective pole of consciousness, which is constructed by the engagement of the self with the world. What science gives us, then, is an increasingly useful phenomenal picture—that is, an understanding of the interface between self and external reality that facilitates at least one species of successful engagement, namely prediction and control. But this useful picture is still just a picture—the world as it appears to us, mediated through our faculties, our concepts, and our categories—and so is not to be identified with reality as it is in itself.

With respect to this second kind of supernaturalism, a further subdivision becomes possible, based on whether noumenal reality is thought to be utterly unknowable or simply unknowable through scientific means. The former is Kant’s position. The latter position admits of even further variations, in terms of both how much of the noumenal realm is thought to be accessible to human cognition and what means are thought to be of use in helping us understand the noumenal. These further subdivisions will not interest me here. It should be clear from what I have said in previous posts, however, that both Hegel and Schleiermacher fall within this broad category, as do many others—including, most significantly, a philosopher who preceded Kant by many, many centuries, and who might be regarded as the founder of Western philosophy. I mean, of course, Plato.

So, let us lay out these alternatives in more formal terms:

Metaphysical Naturalism: The theory that noumenal reality is to be exhaustively identified with the “idealized scientific picture of the world”—that is, the picture that would be generated by the scientific process of careful observation, modeling, and empirical testing of models, if only that process were allowed to continue to completion.

Metaphysical Supernaturalism A (Cartesian Supernaturalism): The theory that the idealized scientific picture would give us an accurate portrait of one kind or dimension of noumenal reality (what we might call material or physical reality) but that there is another kind or dimension (mental or spiritual) that lies outside the scope of science to adequately represent.

Metaphysical Supernaturalism B (Kantian Supernaturalism): The theory that the idealized scientific picture does not give us noumenal reality at all, but offers at best only an ever-more-useful phenomenal picture. Noumenal reality is something entirely other than what emerges in the scientific picture, and is furthermore inaccessible through any other means of inquiry. Noumenal reality, in short, is unknowable.

Metaphysical Supernaturalism C (Platonic Supernaturalism): The theory that the idealized scientific picture does not give us noumenal reality at all, but offers at best only an ever-more-useful phenomenal picture. Noumenal reality is something entirely other than what emerges in the scientific picture, but can be at least partly grasped through other means.

Now none of these categories offers a specific account of the nature of reality. What I have offered here are categories into which more precisely characterized ontologies might fall—but I have done so primarily in epistemological terms (that is, in terms of what and how we know). I have done so in large measure because my starting point was with naturalism, and because at least as I understand naturalism, the naturalist’s ultimate allegiance is epistemological.

Let me explain. If you are a metaphysical naturalist as I have defined it, that means you identify ultimate reality with the picture that would emerge through an idealized science. But insofar as contemporary scientists disagree about numerous things at a very basic level, being a naturalist in this sense doesn’t commit you to a specific picture of what ultimate reality is like. A metaphysical naturalist may have such a picture (and most usually do), but an allegiance to metaphysical naturalism entails a fallibilism with respect to any such picture, insofar as science might advance in ways that force the picture’s abandonment. There are surely metaphysical naturalists who believe string theory offers a (barely comprehensible) portrait of ultimate reality. But to be a metaphysical naturalist in the sense I have been articulating here is to care more about the scientific method than about any specific scientific means of modeling the world. If we advanced to a point where string theory could be empirically tested, and if it were then falsified, anyone who clung to string theory at that point would not be a metaphysical naturalist in my sense.

But this also means that, given my taxonomy, those who thinks noumenal reality is utterly inaccessible through any means, including scientific ones, are metaphysical supernaturalists (of the Kantian sort)—even if they also believe that (a) the inaccessibility of noumenal reality entails its irrelevance for human life, (b) all we should care about is understanding the phenomenal world in which we live our lives, and (c) we ought to rely entirely on scientific methods (and, perhaps, scientifically endorsed methods) in shaping our beliefs.

But persons who embrace (a)-(c) are not apt to call themselves supernaturalists, even though in terms of their views about ultimate reality they deny that an idealized science would put us in touch with that reality (and hence, in effect, believe that there is more to reality than meets the scientific eye). They are, instead, apt to call themselves naturalists. And of course, there is something very significant they have in common with metaphysical naturalists as defined above: they share a deep allegiance to the scientific method and its outcomes.

What this shows is that metaphysical naturalism is only one species of naturalism, and that for a full understanding of naturalism we need to make some further distinctions. In effect, the Kantian “supernaturalists” described above dismiss the entire category of “noumenal reality” as unimportant for human existence, and identify “reality” in the only significant sense as that which we must contend with in experience. The world of experience is the world we live in, and that world can be more or less accurately depicted. Accuracy is measured in terms of the depiction’s usefulness for prediction and control—and the method that's proven itself to consistently produce the best depictions for these purposes is the scientific method.

This perspective is what might be called “pragmatic naturalism.” For the purposes of living our lives successfully, noumenal reality is irrelevant. The only thing that matters is what the phenomenal world is like, in terms of such things as the rules it follows and the state it is in—in short, in terms of the features that we must contend with whether we like it or not. This is “reality” in the pragmatic sense, and the best tool for putting us in touch with that reality is science. Theological speculation, by contrast, is utterly useless in telling us what we need to contend with—and so is useless in helping us to better predict what will happen next and how we can better control outcomes. Praying over grandpa doesn’t restore his heart to health and so keep him alive longer. If we want to do that, we need to learn through scientific methods how the heart works and what interferes with its ability to do its job—and then use that knowledge to devise life-saving surgeries.

Now from this broadly pragmatic perspective, there is no doubt at all that science is extremely important and that we ignore it at our peril. For the sake of predicting and controlling outcomes in the world we live in (however merely phenomenal that world might be), science is really the only game in town. But once we’ve stepped away from the question of what is ultimately real and into the question of what is most useful for living our lives in the world of appearances we inhabit, we need to acknowledge that there can be different ways in which a picture of things is useful. Prediction and control is only one species of usefulness. Conduciveness to a sense of wholeness or meaning should surely be construed as useful as well—and why should usefulness in terms of prediction and control be given priority over other forms of usefulness?

Furthermore, if the whole reason for attaching a special priority to the lessons of science is that they enable us to more effectively engage with our world, science is being assessed within a context of agency. That is, we are assuming that we have the power to make decisions based on our understanding of things. But decision-making depends not only on an understanding of what effects we are most likely to produce by this course of action as opposed to that course of action. Decision-making also depends on an assessment of which effects are most worthy of pursuing, and on judgments about the legitimacy of the available means for pursuing those results. A Nazi researcher conducting experiments on unwilling Jewish subjects might have been as committed to the scientific method as anyone else, and as conversant with its lessons as anyone of his era…and yet it doesn’t follow that this scientist is operating with the most useful belief system for successfully living a human life.

Put more simply, that science offers the most successful beliefs for the sake of prediction and control doesn't mean it offers the most successful beliefs in all areas. It doesn't mean it can ground ethics. In fact, some are convinced that the picture of the world generated by science is entirely value-neutral—reducing values to mere epiphenomenal facts about brains. But agency cannot operate without values. In fact, the entire pragmatic approach to deciding how and what we should believe is a value-laden one: “usefulness” is a value concept. And if values are wholly subjective, then the usefulness of any enterprise—including science—is radically relativized. And so it no longer becomes possible to say, as the pragmatic naturalist does, that the scientific method offers the “best” way of forming beliefs about our world. It is only the best relative to certain subjective value systems, that is, relative to certain brain impulses.

A pragmatic naturalist might attempt to avoid these problems by maintaining, as Sam Harris does, that science can be relied upon to give us the ethical norms we need for decision-making, and that these scientifically-grounded norms are in some sense “better” than any others. I have already expressed my skepticism about that, but for now it is enough to note that in looking to science for an objective grounding for ethics and values, Harris is at best expressing a pious hope. It certainly isn’t the case that the scientific method has demonstrated that science can ground objective morality.

There's another issue rooted in the fact that pragmatic naturalism operates in the context of agency. Specifically, what needs to be the case for “agency” to be possible at all? To be an agent is to make decisions based on reasons—but is that even possible under a purely scientific view of the self? To act based on reasons is a very different thing than to exhibit certain behaviors based on the causal influence of pre-existing conditions in accord with natural laws. The latter accounts for behavior in terms of past events producing an outcome. The former would have it that we are in some way capable of bringing about a course of action because of a judgment to the effect that something is worthy of being done.

One might attempt to take the “judgment that this act is worthy of being done” to be just folk-psychology short-hand for a complex predecessor brain state that produces further brain stimuli in accord with natural laws. But in that case, the conscious judgment as such is simply an epiphenomenal by-product of the brain state that causes the behavior.

This is important for understanding why Kant thought we had to postulate something distinctive about the noumenal self in order for morality to be possible. For Kant, moral principles are truths of reason. They are true by virtue of the nature of rational consistency itself. But unlike other truths of reason, such as mathematical truths (2+2=4, say) these are expressly about what behavior it is rational to engage in (whether or not anyone actually engages in it or has any desire to do so).

For Kant, being moral is about being directly motivated by one of these distinctive truths of reason rather than being motivated by what he calls “inclination” (by which he means any appetite, desire, instinct, emotional impulse, etc.). So, in order for it to be possible to be moral, we have to be capable of doing something, not because there is some prior inclination that causes us to do it, but rather because we are directly motivated by a truth of reason. But truths of reason are not brain states. 2+2=4 is not identical with any brain state, even if a brain state might represent mathematical truths. What makes 2+2=4 true is not some neurological event in the brain but the fact that the concept of “2” is related to the concept of “4” in the indicated way. It’s an abstract truth, not a physical fact.

But here’s the key thing: being moral means, for Kant, being motivated to action by an abstract truth rather than by a physical fact. If I do what I do necessarily and exclusively because of complex brain states that bring it about, then I am necessarily removed from the domain of moral agency.

And if I am removed from the domain of moral agency, then I am removed from the domain in which there might be any objective values that can guide my behavior (as opposed to the subjective preferences that are really just physiological impulses which feel a certain way). My behavior then falls into the same category as the behavior of the weather or the tides, except that the causal mechanism is far more complex and so much harder to predict (and is wedded to epiphenomena that are causally inefficacious). But if my behavior is in the same category as that of the weather or tides, then offering reasons to value science or speaking of the pragmatic value of science becomes utterly pointless. Giving reasons and arguments, making the case for valuing this or that—all of this presupposes a genuine capacity to be responsive to abstract ideas and judgments and principles, as opposed to being motivated by nothing but causal forces.

Or consider artistic activity. If all human actions are caused by prior physical brain states rather than being motivated by abstract principles and ideas, then a poem produced by a great artist is not in a different category than (to borrow B.F. Skinner’s crude metaphor) a turd squeezed out by toddler. The causal mechanism responsible for the former is more complex—but without the capacity for human beings to be guided behaviorally by abstracted ideas themselves (even if we concede that these ideas are always represented by brain states), none of the things we would like to say about the poem and the poet (things it makes no sense to say about the toddler and the turd) can meaningfully be said.

That’s how both Kant and I see it, in any event—and I cannot help but think that those who think materialism can easily make sense of human moral and artist agency just haven’t thought about it long enough to understand the problem. I’m not saying that the problem is necessarily insurmountable (although I can’t see a way to surmount it). What I’m saying is that the problem is real and it’s HUGE. Unless the problem can be surmounted, a worldview that admits in nothing but what science offers us about human nature and the mind is a worldview that systematically strips human life of everything that, well, makes us human.

If we are operating purely at a pragmatic level, rather than at the level of ultimate truth, then I cannot see how anyone could view such a worldview as pragmatically the best. While I fully accept that any pragmatically useful worldview must value the scientific method and treat its conclusions concerning the phenomenal world (the world we must contend with) as invaluable for the sake of prediction and control, the fact is that there isn’t just a phenomenal world I have to contend with. There is also a “me” who has to contend with the world. And there are some presumptions about this “me” that I cannot help but adopt as I engage with the world. One of these is that I can be and really am responsive to ideas and reasons and arguments—abstract entities that, like 2+2=4, cannot coherently be identified with any physical state of affairs (even if, as may very well be the case, these abstract entities are represented by a physical state of affairs, that is, by a brain state).

For Kant, this is precisely the point at which an unknowable noumenal reality becomes relevant despite its unknowability. Because Kant was convinced that the capacity for agency was not part of our phenomenal experience of the world. This may sound strange, since most of us have an instinctive understanding of ourselves as capable of making choices based on reasons. But this capacity for agency isn’t what we experience when we make ourselves the object of experience. When we look at ourselves in that way—when we put ourselves at the objective pole of consciousness and study ourselves the way that we study the stars and the inner working of biological organisms—the result is an understanding of ourselves as causally determined to behave as we do. And so as soon as we make of the self an object of experience--as soon as it becomes part of the phenomenal world--we are presented with an object about which we cannot attribute free agency.

Our sense of freedom and agency is present only when we don’t turn our attention to ourselves, only when we don’t make of ourselves an object of experience. It is immediately felt, we might say—part of our subjective experience of ourselves rather than part of our objective experience of ourselves. That this feeling isn’t corroborated when we investigate ourselves empirically would be devastating for the possibility of moral agency if the outcomes of such empirical investigation represented reality as it is in itself. But because it doesn’t, the determinism found in the empirical world doesn’t preclude us from assuming what we need to assume in order for human life to have the meaningfulness and dignity that (for Kant as well as myself) only comes from moral agency.

In other words, for Kant the chief value of an unknowable noumenal reality is that it makes it possible for us to coherently make postulates that transcend what science can teach us about the empirical world, postulates that we must make in order for our overall worldview to really work, not merely for the sake of prediction and control but for the sake of acknowledging ourselves to be what we intuitively take ourselves to be, and what we must be if our lives and projects are to be more meaningful than a turd.

All of this is a way of saying that Kant is not merely a metaphysical supernaturalist of type B, but also a pragmatic supernaturalist in something analogous to Metaphysical Supernaturalism A. If a pragmatic naturalist finds the most useful holistic picture of the world to be a purely scientific one that makes no room for positing entities or powers that defy scientific modeling, then Kant is a pragmatic supernaturalist in the sense that he finds a holistic picture that includes “transcendent” elements to be more useful overall. These transcendent elements are not objects of knowledge—because they are postulates about what falls outside the phenomenal world and hence outside the realm in which (for Kant) knowledge is possible. That there is a noumenal reality beyond the phenomenal world, even if we can't know anything about it,  make it possible to include in our overall worldview postulates that don't describe the empirical world and aren’t knowable—and this in turn makes it possible for us to operate in the phenomenal world with a worldview that is pragmatically more useful than any worldview limited to affirming what empirical investigation can support.

In effect, then, I am interpreting Kant as embracing Metaphysical Supernaturalism A and pairing it with pragmatic supernaturalism—in contrast with those who embrace Metaphysical Supernaturalism A and pair it with pragmatic naturalism.

Both of these positions need to be further contrasted with someone who embraces Metaphysical Naturalism but then pairs it with pragmatic supernaturalism. This would be someone who thinks that supernatural beliefs are false but useful and so should be encouraged despite being out of touch with reality. Daniel Dennett seems to think that many modern defenders of religion fall into this category, when he talks about “belief in belief” in Breaking the Spell. Richard Dawkins seems to treat Karen Armstrong in these terms as well.

I suspect they’re wrong. I think the theologians and philosophers they pigeonhole in this way are, like Kant, pragmatically-minded adherents to Metaphysical Supernaturalism B. That is, they think that ultimate reality is beyond our reach—including the reach of science—but that we nevertheless need to operate in terms of a (fallible) picture of reality, and that the best procedure for doing so is pragmatic. A picture that denies the models produced by the best science is less useful than any picture that affirms them—but one that affirms nothing but them is less useful than some pictures that affirm something more.

As anyone who’s read my book should know, I lean towards a kind of pragmatic supernaturalism—but I don’t identify reality with what is pragmatically useful. That is, I think reality is what it is, so to speak. But I also think that what we must contend with in experience, and which ways of thinking are most helpful in contending successfully, offer important guidance into bringing our worldview into alignment with a deeper reality (in this respect I am closer to Hegel than to Kant).

I’ve also been convinced by certain philosophical arguments (especially the Leibniz/Clarke Cosmological Argument which I discuss in my book) that the kind of reality pictured in scientific modeling cannot offer ultimate explanations. And so, if there is to be a reason for why there is something rather than nothing (a principle I find myself unable to set aside), then there must be a noumenal reality crucially unlike what science depicts. As such, I cannot accept Metaphysical Naturalism. But when it comes to which species of Metaphysical Supernaturalism I adopt, I find myself waffling quite a bit--but I think I ultimately lean towards the Platonic variety.

But I'm not content to simply go with my leanings here, because I think Schleiermacher and Hegel are right about something important: Whatever the noumenal reality is, I’m a part of it. Not the "me" who is an object of experience—that’s the phenomenal me. And the naïve “phenomenal me” that comes from immediate introspection is no less phenomenal than what scientists look at when they study my brain. What bearing it has on the “noumenal me” remains an open question.

But still, I am what I am—and so in being me (as opposed to putting myself at the objective pole of conscious observation and then studying me) I am being part of noumenal reality. And there may be a way to leverage that fact into some kind of understanding of noumenal reality. That’s what Hegel tries to do in The Phenomenology of Spirit--and whether a project like that is going to work is, I think, a question we should not attempt to answer in advance of pursuing it. I think the way to decide whether noumenal reality can be accessed in some fashion or other is to really try out different strategies and methodologies. The one Hegel recommends is, in effect, a pragmatic one—but one that fills in some crucial holes in more conventional forms of pragmatism.

And so there’s my set-up for talking about Hegel in future posts. But it may take awhile to get those posts put up, in part because I'm getting ready to leave for vacation, but more significantly because talking about Hegel is hard. There are two reasons for this. First, his writing is so danged obscure, so loaded with his own invented jargon. Second, Hegel has an authorial voice I can’t stand, one that is weighted down by the man’s obvious arrogance and sense of self-importance. But unlike certain others whose authorial voices turn me off, I’ve become convinced that Hegel’s is one I need to contend with—because, even if he is a pompous ass, he’s also brilliant.

142 comments:

  1. Bottom line: we all seek ultimate reality otherwise known as higher consciousness, nirvana, cosmic consciousness. Brahman and so much more. Jesus called it The Holy Spirit. The so-called mystical experience is the onset of the mystical state. It is a sudden insight, an inspiration, a realization, an idea! It is an idea that can only arrive from within. I suggest and offer that now we know not only WHY this gift occurs, but HOW it can be attained.

    I note you refer to Hegel. Hegel gave us these words: "Because it's familiar, a thing remains unknown." Similarly, Whithead wrote, "Familiar things happen and mankind does not bother about them. It requires a very unusual mind to undertake the analysis of the obvious." We now know how the words of these two minds apply to consciousness ... and why.

    Respectfully submitted,
    Emmanuel Karavousanos
    Author, Speaker

    ReplyDelete
  2. It seems that Eric is very attached to the word noumenal. I wans finding that this historian really clearly describes what both supernaturalism and naturalism are, very clearly,

    recordings.talkshoe.com/TC-84743/…

    and I don't think once uses the term noumenal. I am wondering if Eric would be able to describe naturalism without the term noumenal or if he is bound to it?

    Cheers!
    RichGriese.NET

    ReplyDelete
  3. Hi, Eric-

    You hit the nail on the head that naturalism is at its base epistemological. I was going to add that earlier, but didn't. Philosophy needs to be centered on what we can possibly know anything about, which is what leads to naturalist accounts.

    "They are, instead, apt to call themselves naturalists. And of course, there is something very significant they have in common with metaphysical naturalists as defined above: they share a deep allegiance to the scientific method and its outcomes."

    And they share one more thing... neither one has the faintest idea what "noumenal" means. And they aren't alone- no one else does, either. It has been thrown around in this series as though it stands for a god's eye view of reality, all-knowing and all-seeing. This a live idea for theists. But it is a dead idea for everyone else. No meaning whatsoever, since such omniscience is purely imaginary.


    "Conduciveness to a sense of wholeness or meaning should surely be construed as useful as well—and why should usefulness in terms of prediction and control be given priority over other forms of usefulness?"

    Ah- note how your criterion here becomes one of story-telling and imagination, not one of "sophy"- truth. You are starting down the road of making stuff up for our urgent and understandable psychological needs. Indeed, you are unfair with the portrayal of "prediction and control". That is only part of the story. Retrodiction is important too, like knowing about the dinosaurs. Naturalists care about the truth in all its senses and possibilities (insofar as we are capable to know it).

    Whether philosophy should be in the business of myth-making for the cultivation of good people.. I guess Plato was a big fan, but how did his system end up? Not a very good reflection on the philosophical profession, frankly. Perhaps this end of things should be dropped. Indeed, one thing that philosophy can tell us is that morals are not objective anyhow. What is good for us and other creatures is to some partial extent objective (as per Harris), but humans are capable of inhabiting remarkably diverse social and imaginative worlds, leading to moral questions far beyond the bare issues of objectively measurable "goods".

    Anyhow, all these issues are not ones of truth, but of taste and pragmatism. It would be odd indeed for philosphers to conclude (a la Leo Strauss) that lying to themselves and others was the best way for humans to live. It may well be, but calling that philosophy seems ironic, if not corrosive.


    "For Kant, being moral is about being directly motivated by one of these distinctive truths of reason rather than being motivated by what he calls “inclination” (by which he means any appetite, desire, instinct, emotional impulse, etc.)."

    This doesn't work. The so-called truths of reason are staked on what? They are staked on what we want.. what we want in the long run (happy marriage) rather than what we might want in the short run (an affair to remember). This language of reason vs inclination is purely a cover for exchanging short-term blind desires for long-term holistically considered desires. A very fine and upstanding process, but entirely staked on inclinations from first to last.

    .. cont ..

    ReplyDelete
  4. "If I do what I do necessarily and exclusively because of complex brain states that bring it about, then I am necessarily removed from the domain of moral agency."

    Well, I am afraid to say that you are exactly in that condition. There is no evidence that our best reasoned and considered desires for hearth, home, and community are any less the product of complex brain states than are basest and baldest cravings. And yet we retain moral agency because we can exchange socially abhorent behaviors for approved ones from time to time.. and thus becomes well-trained civilized beings, occasionally even capable of devising our own rules for common betterment based on our sensitivity to the predicaments of those around us.

    "But if my behavior is in the same category as that of the weather or tides, then offering reasons to value science or speaking of the pragmatic value of science becomes utterly pointless. Giving reasons and arguments, making the case for valuing this or that—all of this presupposes a genuine capacity to be responsive to abstract ideas and judgments and principles, as opposed to being motivated by nothing but causal forces."

    Mechanisms are responsive to abstract ideas! What have we been doing for decades in computer science, after all? Have philospphers been sleeping while their computers have learned to read, to speak, to compute abstract ideas, to build airplanes?

    "If all human actions are caused by prior physical brain states rather than being motivated by abstract principles and ideas, then a poem produced by a great artist is not in a different category than (to borrow B.F. Skinner’s crude metaphor) a turd squeezed out by toddler."

    Jeez- how tired is this argument going to get? The different effects of the two productions on sensitive mechanisms such as ourselves is quite accountable by mechanisms, starting with our sense of smell. If you want to argue for souls, and supernaturalism, it would be better to do so directly than by trying to smuggle it in via these theistic mantras.

    You point to a huge problem with this perspective ...

    "That this feeling isn’t corroborated when we investigate ourselves empirically would be devastating for the possibility of moral agency if the outcomes of such empirical investigation represented reality as it is in itself. But because it doesn’t, the determinism found in the empirical world doesn’t preclude us from assuming what we need to assume in order for human life to have the meaningfulness and dignity that (for Kant as well as myself) only comes from moral agency."

    Wow- this is very interesting. So you go with your intuition over the plain facts of the matter. I take it that "assuming what we need to assume" is fairly construed as making up whatever makes you feel OK and comfortable?

    This is a sad point to come to. I agree that some amount of self-deception is not only natural to the human condition, but also requisite to practical existence. Our minds are extremely well-designed, after all, coming with such functions built-in. But all the same, one's philosophical dedication to truth should rate a little higher, shouldn't it?

    At any rate, I freely grant that you are more meaningful than a turd, by virtue of the exquisite enjoyment I have received in reading and commenting on your work. ... Meaning being created ex nihilo among beings who experience the deep emotions of joy and despair.

    Best wishes on your vacation.

    ReplyDelete
  5. Burke,

    Ah, is that what it is? Is noumenal a code word used by supernaturalists like Eric? I listened to Richard Carrier talk for just under an hour explaining what supernaturalism and naturalism are, in what I thought was very clear, and didn't hear that word noumenal once. Is it a code word pushed by supernaturalists?

    Cheers!
    RichGriese.NET

    ReplyDelete
  6. Burk, I'll let Eric respond to your points, but I feel like you're reading him *awfully* uncharitably.

    And Rich, I'm not sure what your point is supposed to be. Eric is following Kantian usage: the fact that other people might use different terminology doesn't mean anything at all (though, in Carrier's case, I wouldn't be particularly surprised to find out he doesn't know the definition of "noumenal" and hasn't put much thought into the distinction Eric is discussing.) "Democracy" doesn't appear in the Gettysburg Address, but that's no reason to claim it's a "code word" used by Confederate sympathizers.

    ReplyDelete
  7. Dustin,
    And Rich, I'm not sure what your point is supposed to be. Eric is following Kantian usage

    So Eric is a follower of Kant? Or is he using Kant as part of a simply mental exercise. I guess I am having a hard time understanding what the point of this series is. This is now like the 4 or 5 item, and it originally started out as "I am going to describe what naturalism is". Well, I have to tell you, Eric has rambled quite a bit, and written a great deal, and I don't think about is getting any closer to understanding what he thinks it is. He tends to go on and on, and not really come to a point.

    Now he is one to describing whey he thinks it's flawed, and you know. I haven't even got a clue what he thinks it is.

    People forget all about their daily of grammer school where they taught you how to write. You state your thesis right up front, and then you make your case.

    Don't mean to be harsh. I simply have become more amused as this epic has gone on.

    I guess since Eric is a supernaturalist it would make sense he would not like naturalism. But he seems to either be a really bad writer, or he is being pretty dishonest in describing it. If his article was the the Encyclopedia Britannica, he should probably be fired for what he has written so far.

    It seems he could in less words and more clear. Heck if he simply stopped using the word "noumenon" that would eliminate like 700 words right there.

    Lol... sorry, was kind of on a roll there.

    I guess my advice to him would be only talk about things he believes in . So if you are saying that Eric is a follower of Kant, then fine. I got the impression he tended to talk about people and things that he did not actually agree with in his argument. That is always a bad idea.

    BTW.. Richard Carrier, if you listened to the audio file i posted explained supernaturalism in like a sentence. if it is a "non reduceable mental process" it is supernatural. See... he gives a good testable useful working outline. You and I can now probably consider 50 different things, and we could each probably see if they fit that and WHAM! we got it.

    As I said, if you held a gun to my head, I wouldn't have a clue what Eric things naturalism is. I know as a supernaturalist he doesn't like it. So I guess that is why he just can't explain it.

    Cheers!
    RichGriese.NET

    ReplyDelete
  8. Rich,

    Yeah—academic philosophical writing is sometimes hard to follow, especially when the philosopher uses technical terms. My students always wrestle with it—and some of them, like you, conclude that because they don’t get it, it must be deliberately obscure gobbledygook.

    Of course, there's gobbledygook out there. But the reality is that much of what my students dismiss in these terms is actually more precise, more clear, more accurate, than many of the things they claim to understand—things which, in fact, are hopelessly vague and inaccurate once one digs beneath the surface.

    Ordinary language is routinely sloppy and imprecise, obscuring nuanced distinctions, blurring lines in a way that makes it hard to distinguish alternative views, operating in terms of implicit background assumptions that are contestable, etc. While most discourse doesn’t require greater precision, some conversations get mired in confusion and misunderstanding, and parties end up talking past each other or making false assumptions about what the other party means.

    When that happens, philosophical precision can help—but it often requires making distinctions that aren’t ordinarily made, being explicit about background assumptions, attaching half a dozen qualifiers to one’s statements in order to make them say exactly what one wants to say rather than something only in the ballpark. The result, of course, is that the writing becomes much harder to follow. Ordinary language conveys “the gist” of things at the expense of failing to say precisely what one means, while philosophy’s pursuit of clarity and precision often introduces so much detailed qualification that it’s hard to get the gist.

    It’s like when someone asks for directions and you reply by waving in a general area. That’s far less complicated then giving detailed directions. If the directions are complicated, someone who is unused to following directions can get confused—especially if the person doesn’t know the meaning of such technical terms as “left” and “right.” Even if the person giving directions defines these terms, the person may forget halfway through the trip, or lose track of which way is north—and so even the most careful and detailed directions become unhelpful.

    But the way to learn how to follow detailed directions is to practice. And those who learn how to follow detailed directions are able to go places they wouldn’t be able to get to if they were only able to follow vague gestures.

    If you’d rather not practice with my writing (or if you don’t believe that I’m giving good directions), I’d say start with a cogent introductory book on some philosophical subject by a reputable philosopher. In fact, I’ll offer a recommendation: William Rowe’s Philosophy of Religion: An Introduction. Rowe is an atheist, so you won’t have to worry that he’s trying to trick you somehow into believing in the supernatural.

    Rowe will also, I hope, help disavow you of the disturbing view that people can and should only explicate ideas they themselves completely agree with.

    ReplyDelete
  9. Hi, Dustin-

    Yes, I'll agree with you. I was being critical. That is the nature of criticism, and I put myself out on a limb to do so, implicitly asking for you to point out where my arguments are wrong. If you want to point out where they are uncharitable, feel free as well. I will likely agree.

    These are very important issues, since Hegel's philosophy led to so much world historical harm, through German romanticism, which forked into nationalism and Marxism, and on to the wars of the last century.

    Here is a key paragraph from the Hegel wiki page, on his foundations:

    "Hegel's thinking can be understood as a constructive development within the broad tradition that includes Plato and Kant. To this list one could add Proclus, Meister Eckhart, Leibniz, Plotinus, Jakob Boehme, and Rousseau. What all these thinkers share, which distinguishes them from materialists like Epicurus, the Stoics, and Thomas Hobbes, and from empiricists like David Hume, is that they regard freedom or self-determination both as real and as having important ontological implications, for soul or mind or divinity. This focus on freedom is what generates Plato's notion (in the Phaedo, Republic, and Timaeus) of the soul as having a higher or fuller kind of reality than inanimate objects possess. While Aristotle criticizes Plato's 'Forms', he preserves Plato's preoccupation with the ontological implications of self-determination, in his conceptions of ethical reasoning, the hierarchy of soul in nature, the order of the cosmos, and the prime mover. Kant, likewise, preserves this preoccupation of Plato's in his notions of moral and noumenal freedom, and God."

    I'd call that theology, and Hegel was indeed educated in a seminary. The idea that this stuff remains influential in philosophy, let alone in wider intellectual discourse, is disturbing, and that is part of what motivates me to provide vigorous critique.

    The situation appears similar to that of creationism, where a deeper cultural agenda is threatened by evolutionary theory, generating a cottage industry of spurious attacks, unable to deal with the merits, but rather expressing emotional discomfort and revanchism.

    Theology in very broad terms seems to me to hold a similar position, whether openly espoused or hidden in public philosophy departments, expressing a politico-emotional position that draws devoted cultural support despite a rather glaring lack of intellectual merit. Which is to say, lacking a solid premise on which to stake its project, other than mysticism and I-wish-ism, plus an enormous tradition of thought along these lines. And then its practitioners pass themselves off as plucky adversaries of the dominant materialist paradigm.

    ReplyDelete
  10. Rich, quite frankly you're being rude. If you hear an expert in a field discussing their field and you don't understand what they're saying, your first reaction should be to realize that they have a more thorough knowledge of the field than you do and may be speaking with people who know more about the field than you do in mind, and to ask for clarification about things that aren't clear to you. Only after a lengthy process of attempted clarification should you, if things still don't make any sense, feel like you understand the situation well enough to decide that they are talking nonsense. Your first reaction definitely should *not* be to mock them, tell them they're a terrible writer, complain because someone else addressed similar issues in a way that you liked better, get hung up because you don't understand a word they spent an entire essay explaining, accuse them of trying of hoodwink you, and so on. Quite frankly, it's uncharitable and it's immensely arrogant. And I'm also pretty confused about the idea that people should never discuss ideas they don't agree with...


    Burk, I would say that you're reading *me* uncharitably as well--you wrote me an awfully long post, and as far I can tell maybe one or two sentences address what I actually said--but maybe my use of "reading charitably" is not as common as I thought. It is not at all the same as disagreeing with something--it is prior to disagreeing with something, since it's part of figuring out what the something is to begin with. It's a question of really trying to understand what the other person is saying, of trying to understand why they would think what they think, of giving them, so to speak, the benefit of the doubt and interpreting their words, not with the expectation that they will be stupid and wrong, but as though they are someone you really want to engage with. So, for instance, you might wonder if someone on a philosophy blog is really taking umbrage at your disagreeing with someone else when there is a more charitable interpretation available.

    I get the same feeling that you are not really addressing what Eric is saying, but rather some twisted parody version of it. You do it kind of a lot, but I will give just one example. You say this:

    So you go with your intuition over the plain facts of the matter.

    But I am simply not sure how anyone reading charitably could think this is what he's saying. I take Eric to mean that--while scientific observation is obviously immensely important--if we are going to get to the ultimate truth of things, we are more likely to do it by *existing* than by examining. When we examine ourselves as objects, we see ourselves as determined, but when we experience ourselves as subjects, we apprehend that we are free. The difference is in the former, what we are examining is a phenomenal construction, while the latter is an experience of an actual noumenal reality--the self, in itself, with the veil torn away. It would be a very strange thing to think that Eric is saying the latter is some sort of arbitrary hunch, or is admitting that the former constitutes "the plain facts of the matter."

    I am not certain that I have quite captured what Eric is saying--I am only a second year philosophy student, and I may have missed something important. And I am not saying I agree with everything with all of that. But I have made a good faith effort at interpretation, and I am just not sure if you have.

    ReplyDelete
  11. Thanks Eric

    This is the post I've been waiting for because it brings the whole thing together very clearly, and helps me think about things in a new way.

    I still lean towards a flavour of pragmatic naturalism I think, or least would agree with Kant that only the phenomenal is knowable. Science leads us to our very best understanding of the physical world we inhabit.

    It may be unfair to assume that people like me are not aware of the resulting problems for our ethics and aesthetics, I ponder this point frequently and absolutely agree that it becomes the crucial question for my belief system. And the view that ethics can somehow arise from the scientific model is at best hopeful.

    I am also a naturalist in the sense put in one of the links Rich put up, which is to say I suspect our brain states are equivalent to our mind, even if they are too complex for us to currently, or perhaps ever, understand. I think this remains an open question, but am excited about neuroscience's growing capacity in this regard.

    Oddly perhaps, I don't have your problem with moral agency, and perhaps this is definitional. My mind is not like a weather system in a crucial respect, it forms models of other minds, and my belief about their beliefs and attitudes towards me are one of the most important factors in determining my 'choices'. Because other people treating me as if I have moral agency clearly impacts upon the way I respond to the web of social obligations I am a part of, there is a very practical place for moral agency.

    I feel as if I make choices and hence am happy to accept their consequences, and I treat others as if they are capable of choices and this in turn informs their behaviour. So social contracts become possible, as does a sense of self control. This is all I require from my freedom, and it is perfectly consistent with my feeling that my brain is part of the deterministic (or at least probalistically deterministic) universe. I think this is what Daniel Dennett refers to as 'a freedom worth having.

    It is in line with the pragmatism you mention, I behave as if I have moral agency because it works out well for me (according to the values which themselves have been programmed into me by my genes, my culture, my experience and chance).

    It may be there is something incoherent about this approach (I sometimes feel like I'm cheating with it) but I'm not seeing it yet. Perhaps you can help. I look forward to your piece on Hegel because I've always struggled to get a handle on him.

    Thanks

    Bernard

    ReplyDelete
  12. Dustin,

    You captured what I was trying to say very well. And you offer a nice and very helpful reminder about the importance of following the principle of charity, something I wrote about here: http://thepietythatliesbetween.blogspot.com/2009/04/principle-of-charity-and-some-other.html. In any event, thanks for your clarity (and for saying some things I've been wanting to say...)

    ReplyDelete
  13. Bernard,

    The issue of freedom and its relationship to morality is very difficult. I find Kant more compelling than Dennett--but I think this is largely because of how I understand morality itself.

    Dennett is very much a Humean, and I think his thinking about what kind of freedom is required for moral responsibility is rooted in a Humean understanding of morality. Kant, however, was actively trying to REFUTE Hume's contention that "reason is a slave to the passions" when it comes to guiding human behavior. And I find Kant's efforts to show how reason can guide us divorced from inclination rather compelling (if not unproblematic).

    I may address some of that in a later post (I've been thinking about doing more ethics on this blog).

    ReplyDelete
  14. Burk,

    I’m tempted to just say, “What Dustin said.” But let me respond to two additional points.

    You say: “... neither one has the faintest idea what ‘noumenal’ means. And they aren't alone-no one else does, either. It has been thrown around in this series as though it stands for a god's eye view of reality, all-knowing and all-seeing. This a live idea for theists. But it is a dead idea for everyone else. No meaning whatsoever, since such omniscience is purely imaginary.”

    I hardly know where to begin in my response. First of all, one needs to distinguish between the question of what “noumenal” means and the question of what the ultimate noumenal reality consists in. I’ll happily grant that no one has the faintest idea what the noumenal reality is. But you yourself not only know what the term means, you’ve used it for your own purposes in various posts.

    “Noumenal” refers to the reality behind appearances—the thing in itself, as it is apart from our experience of it. And you have consistently (until this series of posts, at least) been insisting in your comments on this blog that science gets us in touch with the reality behind appearances—that neuroscience tells us the truth about consciousness whereas introspection offers only an illusory appearance, etc. You may not have been using Kant's terms, but you have been relying on the concepts. So the concept of “noumenal” is one you understand, whether you use the term or not. You just don’t like the Kantian-style arguments that put the noumenal out of the reach of science and take science to be offering us an account of appearances only—because, I think, you are a metaphysical naturalist (in the sense I’ve been describing here).

    The other chief difficulty with this comment has to do with the reasoning you use to dismiss the noumenal. A straightforward reconstruction of your reasoning runs as follows:

    1. “Noumenal” stands for a God’s eye view of reality—that is, the view offered by an omniscient being.
    2. Atheists don't believe in an omniscient being
    3. Therefore, “noumenal” is meaningless for atheists.

    The first premise here is ambiguous as stated. To render it accurate, it would need to be unambiguated as follows:

    1*. “Noumenal” refers to that reality which an omniscient being, were there one, would be in touch with.

    I don’t suppose I need to go on to show why your conclusion doesn’t follow.

    You say: “The so-called truths of reason are staked on what? They are staked on what we want.. what we want in the long run (happy marriage) rather than what we might want in the short run (an affair to remember). This language of reason vs inclination is purely a cover for exchanging short-term blind desires for long-term holistically considered desires. A very fine and upstanding process, but entirely staked on inclinations from first to last.”

    You obviously haven’t read any of Kant’s moral philosophy or any neo-Kantians. What you describe here is a Humean view of ethics, which Kant emphatically rejected. I can’t in a comment do justice to Kant’s response to Hume or his understanding of how reason grounds ethics, so I won’t try. But trust me, Kant’s entire ethical project is to EXCLUDE “what we want” from having a determining role in moral obligation. A moral law, for Kant, simply IS a law that is binding regardless of our inclinations (short-term or long-term). You might think that such a thing isn’t possible—but before you make bold pronouncements on that matter, you might want to read some of the neo-Kantian ethicists—maybe Christine Korsgaard or Alan Gewirth (if not Kant himself).

    ReplyDelete
  15. Hi Eric

    I don't know Kant's take on ethics well, I've only read a treatment of it by Roger Scruton and at the time remember thinking 'no, I'm with Hume on this,' but will have to revisit it before I can add anything to this debate.

    This seems to me to be one of three crucial impulses though keeping me from a theistic POV. First, if science doesn't approach the world as it is, then how are we to account for its success? It is perfectly easy to imagine a world where the laws of electromagnetism were never happened upon or we never worked out what stars were made of. That always eats away at me. I struggle with any philosophy that doesn't attack this head on. Second, given that my physical environment clearly affects my sense of self, or being, or whatever word I might choose to use, whereas the reverse doesn't appear to be true (drugs can alter my sense of existing in the world, but my sense of existing in the world can not alter the physics that keeps my plane in the sky) then I need a very good reason not to be suspicious that brain states and the mind are indeed identical things described differently. This, I suspect, is where I will part company with Hegel, at the premise that my unexamined feeling of self is enough to establish its existence. I find that the harder I peer at consciousness the less stable a base it becomes to build upon and I have read arguments from a good many philosophers of the mind who share this doubt.

    And finally this one you mention, no matter how I search for a basis for my moral judgement I find only layers of causation, in my past, my inclinations, my culture, and my personality. I am glad about this last point because it requires of me a great degree of tolerance and respect towards the views of others.

    So, if there is a compelling case against any of these three points I would be fascinated, because they would force me to move somehow, which is what one constantly seeks. I'll return to Kant (you are referring to this idea of the categorical imperative right?) and see if that might not yield some questions.

    Bernard

    ReplyDelete
  16. Suppose the phenomenal universe simply contains the noumenal universe -- such that everything real is both experienced and examinable -- and such that anything illusory is just bonus material or noise, depending on your interpretive outlook. This would allow for a form of metaphysical naturalism that doesn't require any "breaking through" of anything, because there's no strong epistemological barrier in the first place. All that's required is a little noise filtering.

    I'm not sure if that's the view I'd necessarily go with, but it does seem to me to be closer to the common sense view of most people who would call themselves naturalists. And it doesn't seem to be a view you've countered in your most recent posts (though maybe you did in an earlier post, and I've just forgotten already).

    ReplyDelete
  17. Hi, Dustin and Eric-

    Thanks for your correctives. I do aim for a higher level of incisiveness and civility. On Kant and Hegel, I am grateful to hear about them, but aim to respond to the thoughts that are presented in this blog with my own thoughts. That seems like a fair bargain, though the less you hear from me, probably the better. I'd love to know more than I do, but life is only so long. On ethics, at least I have reconstructed Hume, despite not having read him either! As I say, I am relating my own ideas, especially where I think they draw on reason & evidence.

    " “Noumenal” refers to the reality behind appearances." As the current blog's metaphysical smorgasbord of options indicates, it means many things to many people, from the pathagorean theorem to the eye of god. And it seems to mean the soul for many as well- that "reality" that lurks behind the human face. This does not seem conducive to clear thinking, and reading a pile of Kant/Plato is unlikely to help at this late date. So this isn't my problem about not knowing what noumenal means. I was pointing out a problem with the idea, its definition, and usage.

    It would also seem incorrect to characterize theistic concepts like noumenal as some kind of exciting intellectual research frontier, like quantum weirdness, dark matter, and the like. The latter is compelled upon us by observations of the world. The former is imaginary, not even a matter of logic as math is. Indeed its likely fate is that of most other theistic conceptions (angels, demons, saints) to wither away on the rubbish heap of human psycho-history, to be taken up by Jungians in some later epoch(!).

    But let me try to approach the matter in another way. Suppose for a moment one naturalist premise is true- that the mind is entirely material, giving rise to consciousness from its natural workings. From what I understand, that would largely undermine Eric's position, since there would be no more rationale for a noumenal self than there would be for a machine that we built and programmed. There would be no rationale for contact with the divine, for moral instruction from that divinity, etc. There would be no soul. Without this support, Eric would be left with the ontological argument, deism, and other much less satisfying arguments for theism, and a thinner, impersonal theism or pantheism, if that.

    What is the difference between these mind premises, naturalist and supernaturalist? The supernaturalist extends the "hope" that his/her intuition of a disembodied soul might yet be correct, and then hopes on top of that this connects him or her to some super-reality with super-beings, creators of everything, etc. However, our evolutionary origin makes little rhyme or reason for such souls. Neuroscience is drilling into the mind-brain connection. Cognitive science assumes naturalism. Every functional piece of evidence we have (other than how it naively feels) indicates mechanistic origin. Eric would say that by his lights, there remains something irreducible about subjectivity that ultimately resists such analysis. That may work now, since we don't have a complete account of the mind-brain mechanism. That gap remains. But how long will it remain? If it disappears, will this ancient philosophy go that way of so many other theistic ideas- up in smoke?

    The naturalist of course looks at this evidence and comes down on the mechanism side, getting more, and more interesting, evidence all the time that this position is correct. For them the basic principles are already clearly evident. All the fealty to past philosophers and ancient intuitions isn't going to change the outcome of this trend, if naturalists are correct in this basic question. So there we are- two traditions locking horns, one extending hope of preserving its ancient intuitions within a narrowing gap of ignorance, (supported by reams of philosophy, old and new), the other taking the evidence as it comes and willing to re-adjust its intuitions to suit.

    ReplyDelete
  18. Eric,

    Thanks for this clear and thought-provoking post

    You write: “The second kind of supernaturalism, by contrast, denies the naturalist premise altogether. That is, it denies that science and its methods have taken us past the domain of phenomena. This would be Kant’s contention.

    I think that’s the only reasonable view to take until such time as naturalism actually provides sufficient evidence that shows that natural science and its methods have taken us past the domain of phenomena. If anything, modern science has removed warrant from the belief that science can take us past the domain of phenomena. (Not even the objective existence of matter can now be taken as obvious. As Bertrand Russell said: “It has begun to seem that matter, like the Cheshire Cat, is becoming gradually diaphanous and nothing is left but the grin, caused, presumably, by amusement at those who still think it is there”.)

    Metaphysical Naturalism: The theory that noumenal reality is to be exhaustively identified with the ‘idealized scientific picture of the world’

    I’d say this ontology should more properly be called “metaphysical scientific naturalism”. Arguably there are less restrictive naturalistic ontologies. On the other hand it appears that virtually all naturalists today are scientific naturalists.

    Metaphysical Supernaturalism B (Kantian Supernaturalism): The theory that the idealized scientific picture does not give us noumenal reality at all, but offers at best only an ever-more-useful phenomenal picture.

    I think Kant would agree that even though natural science cannot give us positive knowledge about the noumenal reality, it can give us negative knowledge about it, namely it can tell us how noumenal reality is *not*. Specifically, noumenal reality cannot be such as to not produce the objective/public part of the phenomenal reality which natural science both studies and extends. It is for this reason, for example, that even scientific naturalists today concede that objective reality cannot be local, an insight that moved some scientists to wonder that the universe begins to look more like a great thought than a great machine.

    This perspective is what might be called “pragmatic naturalism.” For the purposes of living our lives successfully, noumenal reality is irrelevant. The only thing that matters is what the phenomenal world is like, in terms of such things as the rules it follows and the state it is in—in short, in terms of the features that we must contend with whether we like it or not. This is “reality” in the pragmatic sense, and the best tool for putting us in touch with that reality is science. Theological speculation, by contrast, is utterly useless in telling us what we need to contend with—.

    I think one can see that this idea does not work, for the following reasons:

    First, natural science does not study and create knowledge about all phenomenal reality in which we live, but only about the objective/public part of it. The subjective/private part of it also contains order, an order which natural science does not speak about simply because by definition the scientific method does not concern itself with the subjective/private part of the phenomenal world. And, clearly, knowledge about this subjective/private order offers significant pragmatic benefits.

    Secondly, knowledge about the noumenal reality may well have implications about phenomenal reality to which we are not yet acquainted. For example, knowledge about noumenal reality can tell us whether we shall continue to experience phenomenal reality after death or not, or whether good actions may continue to be phenomenally in vain, or whether evil actions may sometimes be a smart choice.

    In conclusion, notwithstanding the obvious usefulness of natural science and its methods, it is an obvious fact that there is knowledge of momentous pragmatic usefulness that lies beyond them.

    ReplyDelete
  19. Eric,

    You wrote: “Giving reasons and arguments, making the case for valuing this or that—all of this presupposes a genuine capacity to be responsive to abstract ideas and judgments and principles, as opposed to being motivated by nothing but causal forces.

    Right. Now, in my judgment, scientific naturalism has so many absurd implications that it should not be taken seriously. On the other hand perhaps it is possible to create a more reasonable non-theistic ontology based on property dualism (which David Chalmers suggests is the most reasonable interpretation of the mind). Here is the best such ontology I can come up with:

    Suppose that what ultimately exists in the noumenal reality is matter having both physical and non-physical properties. Matter’s non-physical properties are usually latent, so that matter behaves in a purely physical manner producing all the phenomena which science studies and mathematically models. In some particular complex configurations of matter, which are characterized by being information processing systems of a high level of intelligence, matter’s latent non-physical properties become active by instantiating a conscious subject. That subject has subjective/private experiences, but has also free will by which it can causally affect matter (in a way that parallels theism’s view of how agents affect their environment). Further the non-physical properties of matter include an intrinsic pointer towards future configurations of matter and hence future subjective/private states of consciousness, which by definition are called “more valuable”. Thus values are grounded on objective reality, on which in turn a robust theory of ethics can be grounded.

    The above non-theistic ontology is something of a stretch, but I don’t see any internal incoherence in it. It does suffer from the so-called interaction problem, but all theistic ontologies suffer from the same problem, so this is not a relative disadvantage. (And in any case this problem is solvable.) Whether the above ontology should be called “naturalistic” is not particularly relevant, the point being that it is a non-theistic ontology which escapes most of the absurdities of scientific naturalism.

    This would be someone who thinks that supernatural beliefs are false but useful and so should be encouraged despite being out of touch with reality. [snip] I don’t identify reality with what is pragmatically useful."

    Why not?

    I mean it’s possible to imagine specific cases where holding a false belief might be more useful. On the other hand, in the context of one’s interpretation of the whole of one’s experience of life, “false but useful” makes no sense. It would like saying: Gardener A’s gardens turn out worse than gardener B’s, because gardener A holds true beliefs about gardening whereas B hold false beliefs.

    ReplyDelete
  20. Let me say that this has evolved into a very rich discussion thread with a number of important thoughts that deserve more attention than I have time to give them now (I'm packing to leave town for vacation and trying to put the finishing touches on a book proposal).

    One comment for the moment: Jarod, you rightly introduce a further position that I left out of my taxonomy (mentioning only in passing in an early post in this series). What you are describing here is direct realism--the view that what we perceive isn't a realm of appearances but reality itself (albeit with occasional mistakes and errors). In other words, it is the view that the Kantian phenomenal/noumenal distinction is a false one--the way things appear just IS the way things are (with some interference to be corrected for).

    Interestingly, Descartes (who was, as I indicated, a supernaturalist of a dualist sort) might very well be classified as a direct realist. But he thought that the only reason we could trust that our perception puts us in touch with reality is because we have compelling reason to believe in the existence of a good God who would not create us such that our faculties are systematically deceptive. In the absence of that conviction, Descartes would never have crawled out of his pit of methodological doubt that led him to question the veridicality of all his experiences. In other words, direct realism can be and has been paired with supernaturalism of a certain kind.

    I am less convinced that it can be paired with naturalism (or at least with the kind of naturalism I'm talking about here, which Dianelos has dubbed "scientific naturalism"). The reason I doubt that direct realism can be paired with scientific naturalism is simply because the world picture that science gives us is so unlike the world picture that immediate experience gives us. If the scientific picture is to be treated as offering an account of how things really are, then there is going to have to be a phenomenal/noumenal distinction--with noumenal reality being identified with the scientific picture.

    ReplyDelete
  21. Burk

    Thanks for your last comment, which seems to me to take us straight back to the elephant in the room. The entire theistic case appears to rely upon the assumption that there is more to the mind than brain states. There is supposedly something beyond the physical and we know about them because we have subjective experiences. Make this leap and, largely because these states are mysterious and so fortuitously malleable, any number of constructions can follow.

    And so the question I keep coming back to is: well is there more to my mind than the physical playing out of brain chemistry? And how would we know? Eric, you state that in the realm of the physical and the predictable, science is the only game in town. And the brain, if not the mind, is physical, so step one must be to use scientific method and knowledge to study the brain. Maybe, at the end of this process a wall will be encountered and the gap you crave will open up. But maybe not. Why prejudge that process?

    If we accept, and I assume everybody here does, that evolutionary pressures explain the emergence of the complex brain, then we can legitimately ask how non-physical properties (a la Chalmers say) were shaped by selective forces. We can also ask exactly what is meant by this subjective pole (Do dolphins have it, did the Neanderthals, how about my five month old sons?) We can study its link to language development, we can scan and look for the characterisitcs of brain states mirrored by it, we can bring the talents of psychologists, anthropologists, biologists, linguists and neuroscientists to bear and chip away at this mysterious force.

    This is just as you say Burk, a clash of two world views, and the evidence, at least in the physical realm, is beginning to mount. It was after all relatively recently that the same hopes were held out for the ineffability of life itself, surely it couldn't just be reduced to chemistry right? Well, wrong.

    And my question to you Eric then, is isn't it a tad disingenuous to claim that moral agency requires a non-naturalist explanation? You understand very well that just such explanations, consistent as you say with Hume's lead, have been developed. More honest perhaps to say that you, like Kant, just don't much like where they lead.

    That of course opens the other door you've peeked into. Perhaps liking an outcome is in itself a good enough reason for holding a belief. You talk of a sense of wholeness and meaning being as valid a knowledge criteria as predictive facility. As a novelist I have considerable sympathy for this approach, but with an important caveat. When the stories which bring us meaning clash with the models that provide predictions, that is, our stories make bad predictions, we find it tremendously hard to cling to them (need I mention Galileo?)

    The only evidence I see for there being a separate 'subjective pole' is that people tell me this is how it feels to them. Personally I find that under close examination this feeling dissolves somewhat, and that makes me suspicious.

    Bernard

    ReplyDelete
  22. Bernard Beckett wrote: “I am also a naturalist in the sense put in one of the links Rich put up, which is to say I suspect our brain states are equivalent to our mind, even if they are too complex for us to currently, or perhaps ever, understand. I think this remains an open question, but am excited about neuroscience's growing capacity in this regard.

    What growing capacity? I would like to suggest that the belief that neuroscience enjoys any traction at all with the hard problem of consciousness, let alone makes progress in solving it, is pure and simple another atheistic myth.

    As chance would have it I am now reading John Hick’s “The Fifth Dimension”. (John Hick is one of the greatest living theologians, and that book so far strikes me as excellent.) Bellow I copy what he has to say about neuroscience and consciousness:

    Most neuroscientists are investigating some area, often a very minute area, of the brain, and it makes no practical difference to them either way what the status of consciousness is. They see this as a philosophical issue which does not concern them. But some very eminent neuroscientists have concerned themselves with this question, and it is noteworthy that they have concluded that the nature and status of consciousness is a sheer mystery. Thus Benjamin Libet of the University of California says that, 'There is an unexplained gap between the category of physical phenomena and the category of subjective phenomena'. VS Ramachandran, also of the University of California, says that 'despite two hundred years of research, the most basic questions about the human mind ... remain unanswered, as does the really big question, What is consciousness?

    Roger Penrose of Oxford University says conscious actions and conscious perceptions - and, in particular, the conscious phenomenon of understanding - will find no proper explanation within the present-day picture of the material universe, but require our going outside this conventional framework to a new physical picture ... whose mathematical structure is very largely unknown.

    Susan Greenfield, also of Oxford, well known for her TV advocacy of mind/brain identity, speaking of the ‘ripples in the brain that I argue constitute consciousness’, nevertheless admits that 'I cannot at this stage describe exactly how a large number of neurons has the emergent property of consciousness'. Antonio Damasio, of the Department of Neurology at the University of Iowa College of Medicine, says that 'electroencephalographs and functional MRI scans capture correlates of the mind but those correlates are not the mind itself. Steven Rose, Director of the Brain and Behaviour Research Group at the Open University, UK, concludes that 'the issue of consciousness lies beyond mere neuroscience, or even psychology and philosophy'. Wolf Singer, Director of the Max Planck Institute for Brain Research in Frankfurt, believes that self-awareness and the subjective connotations of qualia 'transcend the reach of conventional neurobiological approaches'.

    These conclusions of such eminent neuroscientists reflect the fact that consciousness/brain identity is not a genuine scientific hypothesis.

    ReplyDelete
  23. Hello Dianelos Georgoudis,

    quote;
    What growing capacity? I would like to suggest that the belief that neuroscience enjoys any traction at all with the hard problem of consciousness, let alone makes progress in solving it, is pure and simple another atheistic myth.


    That's a pretty incredible statement. I do see that Hick has a theology degree, not a history degree. His wikipedia entry talks a great deal about his speculative views on supernaturalism.

    I tend to be interested in the history of Christianity, but don't have any interest in the speculative supernaturalistic ideas.

    If you know of any people that are interested in the study of Christian history, please feel free to direct them to my site in that I am always interested in meeting people that are interested in the study of history.

    Cheers! RichGriese.NET

    ReplyDelete
  24. Hi, Dianelos-

    Just a brief note- I agree that V.S. Ramachandran is an outstanding cognitive scientist, with the highest credibility among the people you mention. Yet with a book called A Brief Tour of Human Consciousness, he is also a materialist to the core and is putting his work where his mouth is.

    "As the tour guide, Ramachandran, a neuroscientist at the University of California at San Diego, leads readers through a collection of his experiments and theories, championing the idea that charting the brain on a neurological level will provide us with a robust understanding of everything from politics to love."

    As you say and as we all agree, the consciousness problem isn't solved yet. But we also do not fully understand how the brain works yet. So however you slice it, science has work left to do, perhaps a few decades at the most. Perhaps once science has a full understanding of how the brain works from its mechanistic perspective, we can reconvene and ask whether that understanding covers consciousness and the rest of mind activities to your satisfaction.

    ReplyDelete
  25. Burk,

    You posted this quote: “As the tour guide, Ramachandran, a neuroscientist at the University of California at San Diego, leads readers through a collection of his experiments and theories, championing the idea that charting the brain on a neurological level will provide us with a robust understanding of everything from politics to love."

    In the quote I gave previously, V.S. Ramachandran unambiguously states that after two hundred years of research there isn’t even a scientific definition of what consciousness is. This is a very strong statement, because if scientists cannot describe consciousness in scientific terms, then much less can they study it, let alone explain it.

    The quote you post above comes from a review of V.S. Ramachandran’s book and not from the author himself. And in any case it’s ambiguous. I agree that charting the brain on a neurological level may provide us with a robust understanding of politics or of love, because in the scientific context the latter concepts are understood as phenomena which science can study. So, neuroscience may advance to a point where a neuroscientist will be able to explain how neurological processes in a particular person's brain cause that person to express some particular political idea, or to display a specific loving behavior. But such knowledge is really about human behavior, not about human consciousness. In this context see also the difference between the so-called easy and the hard problems of consciousness.

    Perhaps once science has a full understanding of how the brain works from its mechanistic perspective, we can reconvene and ask whether that understanding covers consciousness and the rest of mind activities to your satisfaction.

    You hope then that when we know in detail how the brain works then we shall be able to understand consciousness too. There are several good arguments that show that this hope will never be realized. Let me mention two:

    David Chalmers, a very well-known philosopher of the mind, and a naturalist, has argued thus: Suppose that some time in the future and based on plenty of neuroscientific research we are given a theory according to which when such and such physical processes take place in the brain then such and such conscious experience "takes place" (or “is produced”, or “emerges”, or “becomes contingent on”, or “is identical to”, or whatever). No matter the details of the theory, one might then ask: “What would be different in the data that all scientific knowledge depends on, if that theory is wrong?” Or, in short: “How is that theory falsifiable?” It seems there can’t be an answer to this question. After all one can imagine a universe identical to ours as far as scientific data goes, with the exception that when such and such physical processes take place in the brain then such and such conscious experience does *not* take place. But if both such universes are possible, and all scientific data in both are identical, and the theory suggested is true in only one of them, then it can’t be a scientific theory (because scientific theories must not go beyond the scientific data; only metaphysical theories may do this).

    The second argument rests on the recognition that, surprisingly enough, there isn’t really any evidence that our conscious experience is produced by our brain in the first place, so the hope the neuroscience might help us understand consciousness is sheer speculation. Well-known new atheist author, Sam Harris, who is trained both as a philosopher and as a neuroscientist (so he certainly knows what he is talking about) writes this: "The idea that brains produce consciousness is little more than an article of faith among scientists at present, and there are many reasons to believe that the methods of science will be insufficient to either prove or disprove it" (“The End of Faith” page 208).

    ReplyDelete
  26. Hi Dianelos

    You ask 'what growing capacity?' and that's a fair question. Let me attempt to answer it.

    You are quite right, many neuroscientists like Susan Greenfield do not overtly concern themselves with the question of consciousness. Others, like Steven Rose, point out that we don't yet have a good working definition of consciousness and finally some philosophers like David Chalmers make the bold claim that we can never understand what consciousness is.

    As you will know many practitioners also sit on the other side of this issue and it would be foolish to simply throw authorities back at you, particularly as Chalmers is the only one I disagree with (and maybe aspects of Roger Penrose's claims). Susan Blackmore's 'conversations on consciousness interviews 21 eminent thinkers on both sides of the divide, including those you mention, if you've not already read it.

    My claim is simply that for me personally, the science is helping me define the question. This is not an unusual way for science to progress. As we have studied light over history, the questions 'what is light?' and 'how does light behave?' have been intimately linked.

    So, concrete examples. Studies of split brain patients have helped me refine my notions of consciousness as a motivating force. Studies of multiple personalities have helped me reconsider my belief that consciousness has some characteristic through time. Experiments in change blindness have urged me to rethink my intuitions about what we call qualia. Studies of child development and linguistics have helped me hone in on issues of innate versus learned concepts. Reading about chimps, crows and dolphins has reminded me that the term consciousness represents not a state but a continuum of states and so forth.

    Each of these advances strips away a little of the mystique of consciousness. Slowly this shadowy thing we are trying to define and explain comes into focus. Maybe, at the end of this clarifying exercise, there will be nothing left to explain. I don't know, and so the fascination and the study continues.

    Chalmers' zombie hunch may be wrong because he fails to distinguish between being able to imagine, and believing you are able to imagine. An example. I may naively believe I can imagine a substance that has all the chemical properties of water and yet is not wet. Hence I may claim that wetness is not a quality of water's chemistry, but is something ineffable. A patient teacher might then help me define more clearly what I mean by wet (feel, heat, resistance perhaps) and then show me how the chemical characteristics of the water molecules produce this effect. At this point I would happily revise my claim. I see, I can't really imagine something with these qualities being wet, I just didn't know enough about the terms I was using.

    And finally, shouldn't we be interested that those who cling to the hope that there is some elusive inexplicable quality to thought, are then so often in the habit of defining and discussing it?

    Hope this at least clarifies my claims. Thanks for provoking this response.

    Bernard

    ReplyDelete
  27. Hi, Dianelos-

    You ask a very interesting question. If we understand all the brain circuitry, and that understanding accounts for all the action/response sequences and other inputs/outputs of mental processes, then one could still say that consciousness per se is not necessarily covered, since it might be a disembodied "sensation" layered on top of the known circuitry (i.e. the hard problem). But still, we can say something quite interesting in this case, which is that free will is non-existent. If consciousness is a monitor of other processes of the brain, as I think it indeed is, and follows unconscious decisions/processes, then whether it is embodied or disembodied, it does not constitute mental input, and thus is not the source of free will as naively assumed.

    Secondly, once we come up with a full brain model, it will either have consciousness components (e.g. the gamma wave system, with the requisite chaotic partial localization, etc.), or it won't. If it does, then it would presumably be possible to manipulate it to alter subjective awareness specifically (say, changing a baseball being hit from subjectively white to blue). Formally, it would not be possible to distinguish this from altering computational inputs that are "sensed" by yet some other disembodied system (as above). But at some point, the increasingly precise dissection of these correlations between objective reality and subjective reality would leave this presumed disembodied function with less and less to do.

    This sort of resembles evolutionary arguments. Darwin wrote a whole book full of admittedly circumstantial evidence. Many still said.. well, you still have not shown the actual origin of species. And we still do not observe this absolutely directly today. But at some point, there is strong enough circumstantial evidence that it has driven all other hypotheses from the field. How deeply one clings to the formal remaining possibilities that science has not fully "disproved" is perhaps a matter of taste.

    ReplyDelete
  28. Bernard and Burk,

    I’d like to say how interesting I found your recent posts. I believe that the mind-body problem is central in any ontology, with all versions of theism positing that the mind is primary and the body contingent, and all versions of naturalism positing the opposite. Indeed the best argument I know for naturalism is what Burk refers to, namely that if science can explain all physical phenomena (which includes human behavior) on mechanistic principles then free will does not exist, while practically all theistic ontologies entail that free will does exist. This is the gist of the so-called interaction problem, which has bedeviled theism for centuries.

    I am on a trip right now, and I don’t know when I will have time to comment on this very interesting issue, but I certainly will. The argument I will try to make is that modern science has showed the way for a solution to the interaction problem, and how human behavior can be both free (in the strong, libertarian sense) and also the product of a purely mechanical brain. So please give me some time to respond.

    ReplyDelete
  29. Interesting that Dianelos Georgoudis and Eric went out of town at the same time. I wonder if there is any correlation there.

    Cheers! RichGriese.NET

    ReplyDelete
  30. Great minds vacation at the same time

    ReplyDelete
  31. Rich,

    >> Dianelos Georgoudis and Eric went out of town at the same time. I wonder if there is any correlation there. <<

    Seriously??

    ReplyDelete
  32. Unfortunately I am not on vacation, but on a business trip. Today I shall have many hours time riding on a plane, and I am looking forward to writing down how it is possible for our behavior to be both the result of a mechanical process and be caused by a transcendental and free mind.

    ReplyDelete
  33. The entire theistic case appears to rely upon the assumption that there is more to the mind than brain states.

    The case Eric is presenting here might (though I am not even sure of that.) But "the entire theistic case" certainly does not depend on that belief. In fact, plenty of theistic philosophers are also ardent physicalists regarding philosophy of mind (van Inwagen comes to mind.)

    (By the way, I really enjoyed Genesis.)

    ReplyDelete
  34. Hi Dustin

    Thanks for that challenge. I've just reread Eric's initial posting, lest I was indeed jumping to unwarranted conclusions. And for now I would still contend that Eric's argument does explicitly require that there is more to the mind than the empirically examinable brain state. He speaks of the 'me' that experiences, and of the necessity of including this extra element in one's world view, if only for the pragmatic reason of generating the type of moral agency he is after. For Eric, a purely scientific view leaves out something crucial about what it means to be human.

    I'm not qualified to comment more broadly on theistic arguments in general, until you mentioned Van Inwagen I'd not heard of him for instance, so perhaps there is a new and ingenious take on this that for now is trapped within the confines of philisophical academia.

    I was delighted though, upon hunting down an essay by the above on metaphysics, to then follow a link to an essay by Ted Honderich on free will and compatibilism that takes me straight to the heart of some stuff I need for my next novel, which is based around the free will problem. So thank you for your inadvertent assistance there.

    And finally, in response to Eric's need for a subjective self, I wonder why not instead do what I do, accept that I am the type of animal that needs stories? Without theatre, literature, music or the stories I tell myself about the will of others or my own continuous self, I should be most miserable. So I embrace all these tales for the pragmatic sake of my own fulfillment, while all the time being quite happy to accept they are just stories, a point of view that I hope shall one day make the inescapable fact of my own death far more acceptable to me (and that's the next novel).

    If you do have a link to van Inwagen explaining his brand of theism I am genuinely interested.

    Bernard

    ReplyDelete
  35. Here is van Inwagen's explanation of his own religious beliefs:

    http://www.informationphilosopher.com/solutions/philosophers/vaninwagen/quam_dilecta.html

    On his department website there is an essay titled "I Look to the Resurrection of the Dead and the Life of the World to Come" reconciling his belief in immortality with his physicalism regarding persons, and he devotes a chapter to philosophy of mind in his "Metaphysics" (I'm sure he also discusses it in "Material Beings," though I haven't read that.)

    In general, though, I think he just wouldn't quite understand what your question is. Eric's case here might rely on a certain position being true in philosophy of mind, but so what? Eric's posts here don't constitute the entire case for theism. Some people believe on the basis of various a priori or a posteriori arguments for the existence of God (a lot of people seem to think the Blackwell Companion to Natural Theology is one of the best collections of these out right now, though of course it's not exhaustive.) Some people believe for existentialist, Kierkegaardian reasons. And some people believe on the basis of experiences or intuitions that don't rely on a certain position being true in philosophy of mind--Plantinga's reformed epistemology, or Alston's argument from religious experience. And of course a lot of people combine these approaches in various ways.

    Philosophy of religion is a really interesting field with a lot of really good philosophers in it (many of whom have also distinguished themselves in other fields as well)--Plantinga, van Inwagen, Alston, Robert and Marilyn Adams, Leftow, Rowe, Mackie, Martin, Draper, and so on. And of course, it also interacts with other fields in interesting ways. For those of us who have the leisure to do so, I think it's well worth looking into while trying to reach conclusions about these sorts of questions.

    As for your last paragraph, I don't know--that's an existential question, different people will find different things work for them. But I think I personally would find that approach unsatisfying (I guess I find it intellectually unsatisfying as well, but for different reasons.)

    ReplyDelete
  36. It's not quite correct to say that my case for God's existence depends on postulating something about the mind that isn't ultimately a product of the brain. Rather, it is better to say that I find naturalistic worldviews problematic in part because they have implications for moral agency and subjectivity that are at odds with my immediate experience of myself.

    Taken alone, however, such troubling implications are not sufficient to make the case for some species of supernaturalism (since other considerations might force one to accept the troubling implications despite their poor fit with certain features of immediate experience). And even if one does make this case, one needs to say something about the connection between supernaturalism in one form or another and THEISM. In some comments on earlier posts, Dianelos has sketched out one way of viewing the connection between supernaturalism and theism--but I think there's more to be said.

    In any event, my reasons for being a theist are considerably more complex than what what comes out in this series of posts.

    ReplyDelete
  37. What you say, Bernard, about the human need for stories resonates with me (although, again, I wonder how stories and storytelling are to be construed on a reductionist account of the mind).

    There is no question that stories help to organize our lives. For me, it is an important and open question what kind of epistemic status the best stories have. Clearly, they are not "factual" (or, in the case of nonfiction stories, not MERELY factual). But when people have a sense that a particular story has a lot of "truth" in it, what do they mean, and are they right?

    My own inclination is to see the "truth" of stories in the intended sense as, at least in part, a truth that, through metaphors, aligns us more closely to the "noumenal Real" that defies direct understanding. I suspect that the pragmatic value of narratives emerges from this alignment, such that the pragmatic value of narratives is a clue about reality (but hardly direct evidence in anything like the way that empirical observations are evidence for scientific hypotheses).

    John Hick has a great deal to say about this sort of thing in relation to religious narratives in his central work, An Interpretation of Religion.

    ReplyDelete
  38. Hi Dustin

    You ask so what, with regard to this business of the importance of mind/brain stances for theistic belief. Because you mention such things as intuition and religious experience in your response, I wonder if there isn't still an implicit theory of mind at work here.

    The closest I can get to explaining this, and I don't claim to have any great understanding of the topic but I am curious, is that it depends not on whether one is a theist, but in how much information or knowledge you wish to crank out of a theistic stance.

    I am happy to believe, as per Eric's description of Kant, that there is much to existence that sits beyond our capacity to understand. Why, for a brain shaped by evolution, would this not be the case? If one wishes to name this acceptance of a world beyond as Theistic, then sure, why not, and things like the cosmological argument strengthen the possibility that there's something important we're just not getting.

    But, if we move beyond this and say - although we can not understand the world beyond by empirical means, we can nevertheless gain inklings of it through other methods (meditation, contemplation, story, revelation whatever) - then we are clearly proposing a hypothesis about the workings of the human brain. Specifically, we are proposing that there are some forms of knowledge, which although expressing themselves as brain states, do not come to us by physical means (so we are moving beyond the mechanisms of genetics, physical environment and culture). Another way of putting this is that there exist brain states which do not have physical antecedents.

    This seems the clear choice to me, either we say intuition is nothing more than a culturally and genetically carved behavioural tendency, or we say it is a way of getting some notion of the non-physcial world into our brain. And the second option entails a radical hypothesis about the working of the brain for which we currently hold no evidence.

    Now, for my part I therefore choose to reject this second hypothesis and that keeps me from this form of theism. I don't claim that other people should also reject it, because I accept the role my own cultural and personality based tendencies play in valuing this particular form of consistency. Others, with different starting points, will reach different conclusions and full respect to them, quite frankly.

    So Eric, I still think that when you align yourself with the Platonic approach, as you did in this original post, you are making a strong statement about the mind/brain problem.

    I would also love to hear more at some stage about this notion of moral agency. I always thought, perhaps naively, that moral agency requires determinism to hold. I want my moral choices to be based upon such things as my assessment of probable outcomes, weighted against the set of values I hold dear at that moment in time. So, essentially the application of an heuristic grounded in the physical, deterministic world. To move outside this is to introduce an element not answerable to any such causes, in other words a random element is brought into play (if it is non-random it can be incorporated into the heuristic) and I have trouble aligning the moral with the random.

    Bernard

    ReplyDelete
  39. Hi Dustin

    You ask so what, with regard to this business of the importance of mind/brain stances for theistic belief. Because you mention such things as intuition and religious experience in your response, I wonder if there isn't still an implicit theory of mind at work here.

    The closest I can get to explaining this, and I don't claim to have any great understanding of the topic but I am curious, is that it depends not on whether one is a theist, but in how much information or knowledge you wish to crank out of a theistic stance.

    I am happy to believe, as per Eric's description of Kant, that there is much to existence that sits beyond our capacity to understand. Why, for a brain shaped by evolution, would this not be the case? If one wishes to name this acceptance of a world beyond as Theistic, then sure, why not, and things like the cosmological argument strengthen the possibility that there's something important we're just not getting.

    But, if we move beyond this and say - although we can not understand the world beyond by empirical means, we can nevertheless gain inklings of it through other methods (meditation, contemplation, story, revelation whatever) - then we are clearly proposing a hypothesis about the workings of the human brain. Specifically, we are proposing that there are some forms of knowledge, which although expressing themselves as brain states, do not come to us by physical means (so we are moving beyond the mechanisms of genetics, physical environment and culture). Another way of putting this is that there exist brain states which do not have physical antecedents.

    This seems the clear choice to me, either we say intuition is nothing more than a culturally and genetically carved behavioural tendency, or we say it is a way of getting some notion of the non-physcial world into our brain. And the second option entails a radical hypothesis about the working of the brain for which we currently hold no evidence.

    Now, for my part I therefore choose to reject this second hypothesis and that keeps me from this form of theism. I don't claim that other people should also reject it, because I accept the role my own cultural and personality based tendencies play in valuing this particular form of consistency. Others, with different starting points, will reach different conclusions and full respect to them, quite frankly.

    So Eric, I still think that when you align yourself with the Platonic approach, as you did in this original post, you are making a strong statement about the mind/brain problem.

    Bernard

    ReplyDelete
  40. Hi, Bernard-

    Why so shy? If you are making a reasoned argument based on evidence, then shouldn't this view be rational for other people to hold as well? (Given their typical claim that they are "reasonable"?) If the fact of the brain & mind's material basis were a matter of one's perspective or cultural upbringing, then your conclusions would fairly be a matter of personal preference. But I don't think that is the case.

    The brain works one way, and one way only, in response to Eric's question on the matter. The vast preponderance of evidence points to its fully material basis. I may be wrong, and in that case woe to me in the hereafter or wherever, but the science of the situation is closing in fast and I think I am on extremely solid ground. Conversely, meditations on how consciousness feels, and worse still, runimations on how one's image of moral agency might be damaged by the materiality of the mind, do not amount to rational arguments, and thus do not lead to positions that should be held in view of the evidence.

    This is not to say that any of the departed philosophers were anything less than extremely intelligent. They may have literally not known what they were talking about. It is also not to sow any difficulties about conflict resolution, social intolerance, and the like. People can hold any beliefs they like. As you say, consistency is a matter of taste. But philosophers with a public platform deserve to be held to a higher standard.

    ReplyDelete
  41. Bernard--

    Plantinga and Alston both argue, in different ways, that the sort of experiential/intuitional grounds that might justify theistic belief are rationally on a par with experiential/intuitional grounds that provide solid justification for other, clearly rational beliefs. So I think both would want to say that arguments from philosophy of mind against their positions would also undercut other, clearly rational beliefs (for instance, that our sense experiences are veridical.) And Plantinga's account doesn't even rely on the idea that we are directly getting in touch with anything supernatural (I'm not certain whether Alston's does.) (And I am only a sophmore, I'm afraid I'm probably not the best person to answer your questions--they've written books, if you're really interested.)

    And Burk, yes, we all know you think people can't rationally disagree with you, you don't have to remind us every time you write something.

    ReplyDelete
  42. Bernard--Your most recent comment touches on a set of difficulties I have been wrestling with in a variety of ways. I think it is important to make a distinction between rational agency and libertarian freedom. Rational agency requires that one act "teleologically," that is, that one do act A on the basis of some reason R. A reason is some conceptually articulated consideration speaking in favor of the doing of A--for example, that doing A will reduce a child's suffering. If I do A BECAUSE it will reduce a child's suffering, then there is something that explains my action--the fact that the act will reduce a child's suffering--which is not a brain state. While a brain state might play a role in representing this reason for action to me, it is not to be identified with it.

    If all decisions are causally determined by prior physical conditions in the brain, it becomes hard to see how the fact that a child's suffering will be reduced can explain why I made the decision that I made (since that fact is not a prior physical condition in the brain). In general, it becomes hard to see how reasons can be action guiding.

    But notice that for reasons to be action guiding, it is not necessary that I have libertarian freedom. Reasons might determine my choices. Just as there are causal determinists, there are "teleological determinists," that is, those who think that my choices are determined by the weight of reasons I have for acting.

    The reasons you offer for favoring determinism (roughly, it seems as if the alternative is to act arbitrarily or at random, and it is hard to understand how randomly determined choices are ones I can be morally responsible for) strike me as speaking more strongly for teleological determinism than for causal determinism. In fact, I have often been drawn to teleological determinism myself for precisely these reasons. But at the same time, there seems to be something to "PAP" (the Principle of Alternative Possibilities), which holds basically that in order to be morally responsible for my choice C at time T, it must have been possible for me to have chosen otherwise at T.

    ReplyDelete
  43. Let me add how delighted I am, Bernard, that you have added your voice to these blog conversations. I appreciate the clarity, fairness, and thoughtfulness of your contributions, and I'm looking forward to reading some of your fiction in the near future.

    ReplyDelete
  44. Burk said: "I may be wrong, and in that case woe to me..."

    Fundamentalists attach eternal woe to human errors, but I'm pretty sure the case for that position is incoherent. Which just goes to show that I do think there is such a thing as genuine irrationality...

    ReplyDelete
  45. Dear Bernard Beckett,

    I think there is a major question that anyone needs to honestly answer before you explore philosophical ideas.

    The question is what is your goal? Often the answer fall into two general answer types. 1) I am trying to understand the world or 2) I am trying to justify some kind of supernaturalistic belief I have.

    If you are "simply" trying to understand the world, it is not very hard. I say simply for the following reason. If you get hooked on reading many of the "great philosophers" writings, you will often fall into a problem. A great deal of "the greats" have fallen into what I see as a trap. The trap being that they have often wasted a great deal of their efforts in trying to create some reasonable explanations for supernaturalistic ideas. To me this is a waste of time. I guess historically all the big name philosophers have felt the need to address questions like gods and that stuff, but at the same time, many of them lived in a time when that was an important political issue. I use political in the broadest sense. Many older societies were very theistic or even theocratic. People could be put to death or at the very least loose their jobs if they held the wrong kinds of ideas on gods.

    So you have gotten many foundational efforts that were directed to incorporate reality WITH gods, rather than simply directing their efforts towards reality. Imagine if you had to create theories of the the world, but had to include explanations for mermaids. You will find that almost all your foundational structures are going to screwed up because of your insistence that they handle mermaids. And, since the mermaid issue is going to be the cause of almost all your difficulties in a good overall explanation of the world, you will find that you end up spending a great deal of time trying to incorporate the mermaid problem into your views. you will find that you come up with a number of pretty good views, but they don't quite work because of the mermaids.

    Unfortunately what too many thinkers have done is said, "right... let's really get to the heart of this mermaid issues." "Let's really focus on the mermaids." "If we could just concentrate on these mermaids, perhaps we could get a better underlying set of ideas." See the problem with that is that the mermaids were a thing you simply handed yourself as having to be included in the structural idea. Once you say "Hey, but we have no evidence for mermaids, why am I even trying to include mermaids in my views?" you will be on your way to overcoming the fundamental problem. Once you chuck out the need for your fundamental ideas to include mermaids you will find that your fundamental ideas have a lot more chance of matching reality.

    Cheers! RichGriese.NET

    ReplyDelete
  46. If you are "simply" trying to understand the world, it is not very hard.

    Oh dear.

    ReplyDelete
  47. Bernard,

    You write: “[…] some philosophers like David Chalmers make the bold claim that we can never understand what consciousness is.

    The claim is that we can never understand *scientifically* what consciousness is. In my judgment this claim is justified by several very strong arguments.

    Susan Blackmore's 'conversations on consciousness interviews 21 eminent thinkers on both sides of the divide, including those you mention, if you've not already read it.

    I have read that book, and I understand all of the people she interviewed were naturalists. I found especially striking how about half of them believed that free will exists, notwithstanding the obvious trouble that such a belief makes for naturalism.

    My claim is simply that for me personally, the science is helping me define the question.

    But we already know what consciousness is. The trouble is fitting what we know consciousness is within a naturalistic worldview. Nobody even knows how to define consciousness is scientific terms, which, according to scientific naturalism, should suffice for anything that exists.

    This is not an unusual way for science to progress. As we have studied light over history, the questions 'what is light?' and 'how does light behave?' have been intimately linked.

    Ah, but as Bertrand Russell has made clear, what scientists mean when they speak of “light” is *not* what people mean when they speak of “light”. When scientists speak of “light” they mean that which under the right circumstances causes us to experience “light”. The difference is as huge as conceptual differences can get, but is often overlooked.

    So, concrete examples.[snip]

    I can’t imagine how the cases you mention can help one define consciousness in scientific terms, which is a prerequisite for the scientific study of consciousness. For example you write: Studies of child development and linguistics have helped me hone in on issues of innate versus learned concepts. Daniel Dennett has extensively used such studies, and his conclusion was that animals and pre-linguistic children are not conscious beings, which, it seems to me, only shows that such studies rather than clarify only obscure the naturalistic understanding of what “consciousness” means. Or perhaps naturalistic philosophers, whether knowingly or not, tend to conflate the hard and the easy problems of consciousness.

    Each of these advances strips away a little of the mystique of consciousness. Slowly this shadowy thing we are trying to define and explain comes into focus.

    I don’t see how. Perhaps you can explain. I mean, I have never seen even a vague or ambiguous definition of consciousness in scientific terms. I am not even aware of any idea, no matter how speculative, about how one may be able to find out scientifically whether consciousness exists in the first place.

    [continued in the next post]

    ReplyDelete
  48. [continued from the previous post]

    Bernard,

    You write: “Chalmers' zombie hunch may be wrong because he fails to distinguish between being able to imagine, and believing you are able to imagine. An example. I may naively believe I can imagine a substance that has all the chemical properties of water and yet is not wet.

    I am not sure it is right to say that “water is wet”. Or rather when one says “water is wet” one is not speaking of a property of water itself, but of one’s experience of water. To put it plainly, in our universe before there were any conscious beings around, there was plenty of water but there was nothing wet. Indeed in a universe in which no consciousness exists the concept of “wet” is incoherent. Another way to see this, is to consider that one can experience the presence of wetness while one dreams, and thus without any liquid being present at all.

    Hence I may claim that wetness is not a quality of water's chemistry, but is something ineffable.

    And, I think, quite rightly so. There is nothing “wet” in water, in the same way that there is nothing “red” in electromagnetic radiation of a particular frequency.

    A patient teacher might then help me define more clearly what I mean by wet (feel, heat, resistance perhaps) and then show me how the chemical characteristics of the water molecules produce this effect.

    I am sorry, you completely lost me here. Science describes how light of a particular frequency excites specific cones in my retina, but this advances not one inch towards a definition of what “red” means in scientific terms. Not to mention the fact that we all already know what “red” or what “wet” means as well as the meaning of these concepts can be known. It’s precisely because we already know the meaning of these concepts that the naturalist needs to find a naturalistic explanation for the respective experiences.

    In any case I think Chalmers’s zombie argument goes far beyond what one “believes one is able to imagine”. Rather, I’d say, the argument goes like this: Given how overwhelmingly obvious it is that a universe may exist which is identical to ours in all physical facts but without any consciousness in it, one is warranted to believe that such a universe is logically possible for as long as nobody comes up with a defeater, i.e. for as long as nobody shows that such a universe entails a logical contradiction and is therefore not possible. And I have never heard somebody suggest anything that would even approach being such a defeater. If you know of any such argument, no matter how weak or unpolished, I would be very interested to know about it.

    And finally, shouldn't we be interested that those who cling to the hope that there is some elusive inexplicable quality to thought, are then so often the habit of defining and discussing it?

    Well, theists certainly do not see any “elusive inexplicable quality” to consciousness, including thought. On theism, consciousness is the primary property of God, and thus the most basic nature of reality. Which comports very well with the fact that we all precisely know what consciousness is, and indeed cannot fail to know it. After all, anything else we may know, we know consciously.

    ReplyDelete
  49. Hi, Dianelos-

    "In any case I think Chalmers’s zombie argument goes far beyond what one “believes one is able to imagine”. Rather, I’d say, the argument goes like this: Given how overwhelmingly obvious it is that a universe may exist which is identical to ours in all physical facts but without any consciousness in it, one is warranted to believe that such a universe is logically possible for as long as nobody comes up with a defeater, i.e. for as long as nobody shows that such a universe entails a logical contradiction and is therefore not possible. And I have never heard somebody suggest anything that would even approach being such a defeater. If you know of any such argument, no matter how weak or unpolished, I would be very interested to know about it."


    I think an appropriate reply is that, were humans and animals absent, then yes, one could imagine a world like ours without consciousness. But Chalmers is trying to have his cake and eat it as well- to assert that he can imagine humans without consciousness, i.e. zombies. And I don't think we have any reason to believe that is possible, despite our frequent immersion on horror-movie zombies. It certainly doesn't happen empirically. If I am sleepwalking, I have experiences at that time- a sort of quasi-consciousness- but I don't remember them later. If I have lost consciousness, I am inert and incapable of action- a vegetable.

    So he is making a counter-factual, and I believe a counter-possible assertion, from which any conclusion is not necessarily, or even probably, valid. There does not have to be logical contradiction involved, only a recognition of the unreality of his premise. One could go further and claim that complex information processing as we are used to as humans and expect in our interactions with other humans necessitates consciousness as commonly understood. But that is a more complicated AI-related argument than I can make on my own right now.

    ReplyDelete
  50. This comment has been removed by the author.

    ReplyDelete
  51. Hi, Eric, Bernard-

    I am puzzled by Eric's argument. I would parse the first part as:

    The reason R for act A can not be brain-physical because it is identical with a future fact that has not happened yet.

    But the fact of having helped a person is not part of the picture at all. Our action may indeed hurt the person. The road to hell is paved with good intentions. Reason R is entirely a mental construct, based on our model of what might help the person at issue, nothing more. And unfortunately, it is often a highly flawed model, as we so frequently experience. Such models are easily construed to be the result of past physical brain states, information inputs, reasoning, etc.

    For the next part, I am lost on the distinction between teleological and causal determinism. Both seem to posit causal chains as responsible for everything. Does the teleological flavor imply some re-existing vague principle that envisions the end-state to which we are heading?

    At any rate, when you ask whether alternative actions were actually possible, you are in a deep bind. The fact that things happen one way eliminates other paths, and makes it unknowable whether other paths were really possible, given exactly the same physical states all around. I would say not, and there is no evidence otherwise, especially for anything other than physical states being present or relevant.

    The PAP issue is seemingly not about this sort of very basic free will issue based on physical states, but about our ability to respond to new information and training. If we can be influenced to act in desired ways, then we are moral beings, able to adapt and fit into a social group. If we can't, perhaps because we are psychopathic, then we are not moral beings, incorrigible and have to be locked up. The ability to learn and adapt does not contradict physical determinism, since training influences are all part of the computational input mix, as are reflections on past actions, reasoning, etc.

    The alternative action that PAP refers to is not something counter-physical from all the antecedent influnces, but something more high-level, like the ability to be influenced by a speaker to take another view, or to read an article about a political question and alter one's actions in response to its new information.

    ReplyDelete
  52. Hi Dianelos

    There are more challenges there than I can address in a single response, please don't see this as an attempt to avoid the argument, but I'm conscious (sorry) of not wishing to let this strand hijack the post.

    Yes, I could have been clearer in my wetness example, I thought it was implicit. To be explicit then, I meant I can say 'I can imagine two substances, both with the exact molecular characteristics of water, with one feeling wet to me and the other not. Therefore, because I can imagine such a thing, the thing that causes water to feel wet to me must be independent of its molecular structure.' And so, believing I can imagine such a thing clearly must involve some sort of misstep. I could also say, I suppose, that I can imagine a rock being conscious. and therefore consciousness must be independent of the brain, as rocks don't have them.' Feeling I am able to imagine something is never a clincher.

    Now my problem is I can't imagine a universe with zombies in it, the thought just isn't comprehensible to me. This is because, when I consider the difference between conscious and subconscious processes, in my writing for instance, the conscious comes into play in very specific circumstances, in order to allow particular cognitive tasks to occur. Sometimes I wake up with a narrative problem having solved itself offstage so to speak, but I never wake up with a new chapter having assembled itself. So part of what consciousness is for me, is this ability to deliberately manipulate language. In this sense, a zombie that can manipulate language in this respect at least conscious.

    I realise that against this you will say, no, this is not what I mean by consciousness. I mean something like the experience of seeing red, which everybody knows. Well, again, I hate to be stubborn but I don't know it in the sense I think you mean. The experience of remembering red is different to the experience of experiencing red in my peripheral vision, which is different to my experience of glancing at a red book cover which is different from attending closely to a red object, which is different from the experience of imagining a red object. I think it is unfair to claim we all precisely know what consciousness is. There was a time I thought I did, but then I looked more closely and lost all certainty. I feel the same way about time.

    The comparison Burk made with evolution is pertinent. We used to think it impossible to imagine design without a designer, but Wallace and Darwin showed we were wrong. Now we say we can not imagine perception without a perceiver, or some such thing, and we are at the stage of the 19th century biologist, patiently collecting the data, putting it all together and waiting for the aha! metaphor to take us forward. Unlike Burk, I think, I'm not sure that metaphor is in place yet, but unlike you, I see no reason to believe it can not in principle be done. (Have you read Dennett's chapter on Zombies in Sweet Dreams?)

    To return the ball to your court then, if consciousness is indeed ineffable, but corresponds exactly with brain states, then it can be neither a window into a world beyond the physical or a driver of non-determined will, unless we posit brain states that have no physical antecedents. This comes back to your promise to explain free will in a deterministic context, and I look forward to that as it'll give us plenty to talk about I'm sure.

    I hope I don't appear unsympathetic to your point of view. A part of me is deeply attracted to it. It's just that, when I examine my own conscious experiences, they fall short of what your model appears to demand of them.

    Bernard

    ReplyDelete
  53. Burk,

    You write: “But Chalmers is trying to have his cake and eat it as well- to assert that he can imagine humans without consciousness, i.e. zombies

    Chalmers does not say that we can imagine a different universe from ours where humans without consciousness exist, for the concept of “human” entails the concept of “consciousness”. Rather he is saying that it is possible that a different universe exists (indeed one with a different kind of matter than ours), where biological machines have evolved which react in exactly the same way to any scientific test as our bodies in this universe do, while these machines in that universe are devoid of consciousness.

    Chalmers’s claim strikes me as obviously true. To see this consider that even in our universe with our kind of matter it would be impossible to argue with a highly intelligent but non-conscious machine that we ourselves are conscious beings. Any argument we may use to convince that machine that we are conscious beings, that intelligent machine would wave away pointing out that a quick scan of our mechanical and non-conscious brain exactly explains why we are talking like that.

    The point here is that given what we know of science one does not require the consciousness-hypothesis. One can explain all physical phenomena, including all phenomena related to human behavior, without assuming the existence of consciousness.

    ReplyDelete
  54. Dear bernard beckett,

    I recently listened to Luke Muehlhauser interview Tom Clark from naturalism.org, and made some notes on the interview ( webulite.dyndns.org/node/… ) with a number of good links including the audio file, and a great introduction by Tom as to what naturalism is. Thought you might enjoy it as it goes directly to this discussion of naturalism which you seemed to be interested in.

    Cheers! RichGriese.NET

    ReplyDelete
  55. Bernard,

    You write: “To be explicit then, I meant I can say 'I can imagine two substances, both with the exact molecular characteristics of water, with one feeling wet to me and the other not.

    If the two substances have exactly the same physical characteristics, and your conscious experience ultimately comes from your body’s physical characteristics interacting with your environment’s physical characteristics, then it follows that you would experience both substances are wet. So far then, to say ‘I can imagine the second substance not feeling wet to me’ is like saying ‘I can imagine inferential logic being false’.

    Have you read Dennett's chapter on Zombies in Sweet Dreams?

    No, on the other hand I experience reading Dennett as an exercise of finding out where in his exposition of a plethora of interesting physical facts he is confusing the easy and hard problems of consciousness.

    I'm conscious (sorry) of not wishing to let this strand hijack the post.

    I agree. As I will argue next, there is a way for naturalists to sidestep the hard problem of consciousness by pointing out that science has by now convincingly shown that there is an exact correlation between conscious states and brain states, and the latter do behave naturalistically. It follows that conscious states too must conform to physical reality’s mechanical nature. Which leaves no place for human freedom of will, or for a god who is active in one’s experience of life.

    ReplyDelete
  56. Bernard wrote: “[Eric] speaks of the 'me' that experiences, and of the necessity of including this extra element in one's world view, if only for the pragmatic reason of generating the type of moral agency he is after. For Eric, a purely scientific view leaves out something crucial about what it means to be human.

    Yes, it looks like the “scientific” worldview makes the existence of human free will impossible (and I mean “free will” in the common “libertarian” sense). This worldview not only makes of God an absent landlord at best, but also denies what we mean when we speak of being human.

    First, here is how I’d say the naturalistic reasoning goes:

    Only a few centuries ago people observed the highly complicated trajectories of the planets in the night sky and thought there must be some intelligence guiding them, perhaps trying to tell us something. Then scientists discovered that a heliocentric model of the universe plus some simple laws (ultimately Newton’s) could explain these mysterious phenomena without assuming the God hypothesis. A little later one of the most perplexing features of the phenomenal world, namely the variety, functional complexity, and apparent design of the species, was explained by Darwin on a purely mathematical (indeed algorithmic) basis, again without the need of some supernatural agency guiding the whole thing. Life appeared to be mysterious, but now we know that it’s all a complicated electrochemical reaction. By now there is little that one might argue is not explainable by science.

    Within the ever smaller gap in which theists had to find place for God, it is now argued that science does not and cannot explain consciousness. Well, perhaps so. One way or the other, the problem of consciousness is irrelevant, because science has shown beyond reasonable doubt that all conscious experience perfectly correlates with physical processes in our brain. Therefore, even if consciousness itself should remain an unexplained mystery, what we are conscious of, including its ineffable qualitative dimension, cannot violate the closed causal system of physical phenomena that natural science describes. The implications are far-reaching for how we think about ourselves. Significantly, science models physical phenomena mathematically, therefore they evolve by blindly following physical laws. So there can’t be any genuine free will in human actions, no matter how it may feel subjectively. It is not possible for one to have acted differently from how one in fact did, so moral responsibility does not exist either. The same goes for the existence of good and evil in any objective sense. All ideas about value or goodness or justice or freedom are simply mirages induced by our conscious, albeit mechanical, brain. Similarly, God, if God exists, cannot in any way affect our experience of life, including the subjective/qualitative dimension of it. For if God did affect our experience of life, then science would be able to detect that meddling, because that supernaturally caused effect in our conscious experience would have to correlate with some physical process in our brain, which in turn would show up as some anomalous break in the causal closure of nature. It follows that mystical experiences and spiritual insights are not caused by God or by any transcendental reality, and hence cannot be used as a reason for believing in them.

    The scientific worldview is not pretty, and does not comport with how we are used to thinking about ourselves, but this is irrelevant to its truth. As Richard Dawkins writes in his “River our of Eden”: "In a universe of blind physical forces and genetic replication, some people are going to get hurt, other people are going to get lucky, and you won't find any rhyme or reason in it, or any justice. The universe that we observe has precisely the properties we should expect if there is, at bottom, no design, no purpose, no evil and no good, nothing but blind, pitiless indifference."

    [continued in the next post]

    ReplyDelete
  57. The naturalistic account in the previous post has several weak spots, but these are not significant. What’s significant is that for many a naturalist that account is not just an interpretation of scientific knowledge, but an implication of it: If it is the case that (1) all natural phenomena are causally closed, and (2) that our conscious experience exactly correlates with the natural phenomena in our brain - then it follows (3) that no free will exists, and (4) that there are no effects in our experience of life caused by some supernatural agent. I will now try to show why this logical inference is fallacious.

    Up to the beginning of the 20th century it appeared there was overwhelming scientific evidence that the universe is deterministic, i.e. that given the state of a closed system its future was unique: there could only be one future state of that system at any point of time. The discovery of quantum mechanical phenomena proved that the future state of a system was instead a probability distribution over its current state. (Incidentally the deterministic interpretations of quantum mechanics do not change this fact, because from the point of view of an observer it is always the case that a system can evolve in many different directions.)

    Let us now take the state of the universe at some early age and consider *all* its future states allowed by science.

    We shall first note that not all future states of the universe would appear to a scientist to be causally closed, For example the universe could evolve in a direction in which every time somebody cried “Jesus heal me” any ailment one suffered from, from a runny nose to terminal cancer, would immediately disappear. In short, not all possible universes allowed by science would strike a scientist as naturalistic. Let’s say that the universes in which the causal closure of natural phenomena is maintained under any scientific test, possess the “N” (naturalistic) property. Our universe, as well as the vast majority of the possible universes that might have evolved from that initial state, do possess the “N” property.

    Further let’s consider the universes with the “G” (God) property. These universes are the ones where God’s special providence happens to obtain. So, according to classical theism, God designed all living organisms, and especially humankind with our particular cognitive faculties, and so on. Clearly our universe does possess the “G” property, as do many other possible ones. Indeed their number is so vast that there will be some where, say, the brains of many people happen to form true beliefs about God, which is presumably what God wishes.

    Finally we shall consider the universes with the “H” (human) property. These are the universes where human beings possess free will, in the sense that when I freely will to raise my arm my brain’s state happens to move towards the state that will cause my arm to be raised.

    So here is a model of reality which is entirely compatible with scientific knowledge (indeed it’s inspired by it) and particularly with the causal closure of natural phenomena, and is also compatible with the premises of traditional theism: God created the universe with such physical laws that out of all the possible future states of the universe there would be plenty that possess both the “N”, the “G” and the “H” properties. Out of those God randomly picks one and actualizes it into reality (so that plenty of the events we observe are random, just like they seem to us). In other words the idea is that apart from God, we humans too are transcendental persons, capable of freely affecting the state of the physical universe, within the limitations of the natural laws that govern it. Which comports exactly with our experience of life including our experience of free will.

    I think the above model works pretty well. Indeed it’s falsifiable, for example if one were to discover that the human brain is such that it approximates a deterministic machine (as digital computers do).

    ReplyDelete
  58. Dear Dianelos Georgoudis,

    A major problem with your view is that gods do not exist, so your views will have obvious problems aligning with reality.

    Cheers! RichGriese.NET

    ReplyDelete
  59. Dear Rich Griese,

    A major problem with your view is that God does exist, so your views will have obvious problems aligning with reality.

    Cheers!

    For real, though, the present discussion is largely about determining whether God exists or not. How could you possibly think randomly asserting your position in the middle of it could possibly help anything, and could possibly make you come across as anything other than rude and arrogant?

    ReplyDelete
  60. Dear Dustin,

    Dustin quotes Rich Griese;
    A major problem with your view is that God does exist, so your views will have obvious problems aligning with reality.

    Cheers!


    Rich Griese actual quote;
    A major problem with your view is that gods do not exist, so your views will have obvious problems aligning with reality.

    Cheers! RichGriese.NET


    Please note that you misquoted me. I reposted the original quote.

    Cheers! RichGriese.NET

    ReplyDelete
  61. Dear Dustin,

    Dustin quotes Rich Griese;
    A major problem with your view is that God does exist, so your views will have obvious problems aligning with reality.

    Cheers!


    Rich Griese actual quote;
    A major problem with your view is that gods do not exist, so your views will have obvious problems aligning with reality.

    Cheers! RichGriese.NET


    Please note that you misquoted me. I reposted the original quote.

    Cheers! RichGriese.NET

    ReplyDelete
  62. Um. No. I wasn't trying to quote you. I was just saying, you know, people on either side can just baldly assert their position in an obnoxious way, it doesn't prove anything.

    ReplyDelete
  63. Dear Dustin,

    I was not badly asserting anything, or being obnoxious. I was simply declaring the fact that gods do not exist.

    Cheers! RichGriese.NET

    ReplyDelete
  64. Yes, Rich, you are being obnoxious. When people are having a grown-up discussion about something, and you jump in with, "But the trouble is, you're wrong," that is incredibly obnoxious. It serves no purpose whatsoever. The only thing that can possibly happen is that you annoy people and give the perception of arrogance.

    ReplyDelete
  65. Dear Dustin,

    I am not sure what your objection is, my original comment went to the subject of this post, naturalism. You should probably read Those Damn Mermaids for a more full explanation, as well as perhaps Tom Clark - Naturalism as a Positive Worldview.

    Cheers! RichGriese.NET

    ReplyDelete
  66. My objection should be BLINDINGLY OBVIOUS. This 60+ comment discussion has been largely about whether God exists or not. Everyone already knows who holds what position. What could POSSIBLY make you think that, in the middle of a lengthy and intellectually sophisticated discussion about whether God exists, saying, "Oh, but God doesn't exist," and nothing else, and then, I assume, sitting there with a self-satisfied grin, could POSSIBLY change anyone's position on anything?

    And yes, I read your silly post on mermaids, which you should know because you *also posted it here* and I *commented on it.* You should probably read, I don't know, a book on ettiquette or something.

    ReplyDelete
  67. Dear Dustin,

    quote;
    Um. No. I wasn't trying to quote you. I was just saying, you know, people on either side can just baldly


    So you are saying that when you copied text from my comment, and then pasted it again at the top of your comment, that you don't consider that quoting? I am talking about in your above post, where I said that you had quoted me incorrectly.

    Cheers! RichGriese.NET

    ReplyDelete
  68. A couple of things.

    First, since this post and subsequent discussion has raised questions in the philosophy of mind, and since there is clear interest in pursuing these questions, I will endeavor when I get back from my vacation to post something sufficiently provocative on the topic to provide a forum for further discussion--probably not next week, since I'll be leaving town almost as soon as I get back home in order to attend a funeral, but likely the week after that.

    Second, it can certainly be helpful on occasion to point out unhelpful comments, lapses in etiquette, etc., as a way to keep people honest and on track. But if the recipient of such corrective comments doesn't get it after one or two attempts, it's probably best to leave well enough alone so as not to distract too much from the more substantive and stimulating discussion. In general, however, I want to thank all of you for your role in producing a discussion thread rich in ideas worth further exploration.

    ReplyDelete
  69. Point taken, Eric. Sorry to hear about the funeral.

    ReplyDelete
  70. Dear Eric,

    I think I had a misunderstanding. I was responding to what I thought was a reference Dustin made to something I had said. But Dustin clearly stated that he was not quoting me, so my posts subsequent to that were a complete misunderstanding on my part.

    Cheers! RichGriese.NET

    ReplyDelete
  71. Rich Griese,

    You write: “ I recently listened to Luke Muehlhauser interview Tom Clark from naturalism.org [snip]

    Late last year I had a discussion with Tom Clark and Keith Parsons about naturalism and consciousness. If you’re interested you can find it here: http://secularoutpost.infidels.org/2009/10/further-comments-on-naturalism-and.html

    A major problem with your view is that gods do not exist, so your views will have obvious problems aligning with reality.

    Well, I'd agree that an ontology which has obvious problems aligning with reality is probably false. So perhaps it’s telling that naturalism has problems aligning with the reality of consciousness, and with the reality of quantum phenomena, and with the reality of the apparent fine-tuning of the fundamental constants, etc.

    ReplyDelete
  72. HI Dianelos

    That's a fascinating post because it gets me inside a point of view I'm not familiar with. I'll digest it properly before responding.

    I know what you mean about reading Dennett, I remember upon first reading Consciousness Explained that I was excited by his early claims that the subjective nature of consciousness would be explained, and then became frustrated as it became clear it was actually a discussion about how that might in time be possible, with examples of how approaching the easy problem could in itself chip away at the hard problem.

    The tricky thing with this debate then, is I'm not sure how one rationally progresses. I think to be fair to Dennett, he does honestly feel, as he examines his own conscious experience closely, the hard problem evaporates. Because the hard problem is defined by our subjective experience, what do we do, just take his word for it?

    I've tried in good faith to replicate some of those experiments of introspection, and enjoy playing round with my own, and I do honestly have to say I get some inkling of what Dennett is talking about. It's only an inkling and I do tend to understate the hard problem on the back of it I know, but are you able to take my word that it does indeed feel that way to me?

    In turn, you tell me that when you consider subjective experience you do just know it can not be dissolved in this way and I suppose I then should take your word for that. How does one then construct a reasoned discussion about that which is not available to third person inspection? It's awkward I think.

    Bernard

    ReplyDelete
  73. Dear Dianelos Georgoudis,

    Thanks for URL to the secular outpost post by Tom Clark with some followup comments by you. That blog entry seems to be a continuation of some prior discussion, so it sort of picks up in the middle of a thought, rather than have a well stated thesis for one to consider. But, it is a blog I was unaware of, so I will slap it in my google reader and see if it brings me any good things in the future. Thanks.

    I looked up your blogger profile linked from these posts, and see that you are from Greece, but you do not have a blog listed, or any information about your background.

    Are you a supernaturalist? I think I read something from you in the past that made me think yes, but instead of recalling incorrectly, it's better if I just ask.

    Also, are you involved in some way with philosophy? I mean, is that what your degree is in, or is that they field that you work in? Is philosophy something that you feel is useful for determining reality?

    I am a recently retired computer worker in the USA. I am not a supernaturalist. I have been interested in the study of the history of Christianity from a historical stand point for almost 20 years now. And now with my extra time, I thought I would look to meet others that are also interested in the subject.

    I am also interested in history (mostly of western society) in general, literature, and politics, which I also hope to find good internet people to chat with on those subjects.

    I hope to write a small introduction to Christian history for the generally interested based on my prior study, and have begun to set up some online tools to allow me to do with, while also allow other people to participate and make contributions and edits.

    Philosophers are not the best to learn from in my opinion. They speculate. Now back in ancient days the term was more general. So some of the greats that we call philosophers would also what we would call scientists if the term was used them. But with time, I believe that fields broke apart, and those that do more than speculate formed individual branches of science, and began to carry out experiments to confirm or dis-confirm their ideas, and began to do what today we call science. While philosophers continue to speculate.

    For understanding how we think, it would be better to follow biologists, neurologists, and any of the scientific fields that are now concentrating on the brain and it's functions, rather than the speculations of philosophers. For example, a super book that I read last year; _Mirroring People: The New Science of How We Connect with Others_, by Marco Iacoboni (some basic info here; http://en.wikipedia.org/wiki/Mirror_neuron) was very interesting, and indicated to me that we are really starting to understand the brain more than I had realized.

    BTW... is http://en.wikipedia.org/wiki/User:Dianelosis you? I looked up in google the name "Dianelos" to see if it was a man or a woman, and this was right at the top. I might have determined that it is not just a man's name, but wondered if this is actually you.

    Cheers! RichGriese.NET

    ReplyDelete
  74. I mean this wikipedia address not the one I included in my prior post;

    http://en.wikipedia.org/wiki/User:Dianelos

    Cheers! RichGriese.NET

    ReplyDelete
  75. Hi Dianelos

    Sorry, I think I must be missing something in your argument because I'm struggling to put it together in my head.

    You say if naturalism holds, then there are still many possible ways the universe could have unfolded. Hence we live in one of many possible worlds. And you use quantum uncertainty to get to that position. Up until that point I'm with you, although I personally wouldn't want to use quantum uncertainty to get there, as I'm what Eric would call a Kantian supernaturalist (sort of). Science doesn't so much describe reality as give us our best working description and hence is where we're best to look for clues - so already a myriad of other possible universes are allowed.

    But, that aside, the next stage seems to me to say, well these worlds could have different flavours. Perhaps our universe really is naturalistic according to your excellent summary, so posing the problems for will and moral values. I think this is how the universe probably is, and it does get me into bother when it comes to moral relativism.

    You then offer other possibilities, and I suppose with a little imaginative application we could describe many many variants. One of them you call the G, the God variant. And here I get confused. You seem to say that in this version we ourselves have knowledge of God, and have some transcendent ability to exert our will above and beyond physical brain constraints. And you also seem to be making the claim that this is the version we live in.

    If I have you correct, then you are still vulnerable to the naturalistic challenge. For in this world there would be evidence, would there not, of this transcendence, in the way the brain goes about its business of making meaning and indeed decisions. We would see unprompted, or spontaneous, brain states emerge.

    If you're saying such a thing is possible and we just don't have any evidence yet, then fine. I am absolutely with you. Because there is no evidence I'm inclined to rank it as less likely a world, and so assume I'm not in it. Should the evidence come in I'll be first to jump the fence. Because I'm no dyed in the wool naturalist (and neither are any of the scientists I know) I wouldn't preclude the possibility.

    Perhaps though you are also saying that the evidence already exists, in the form of our conscious, subjective experience, and this data alone makes your G world the more likely option. I can certainly see why you would make this step. And this brings us straight back to this damned issue of qualia doesn't it?

    But, I am sure I am missing something and you meant to say something more here. Sorry if this is the case.

    Bernard

    ReplyDelete
  76. Dear Bernard Beckett,

    You mentioned 'moral relativism' in a recent post. I thought it would be an opportunity to clarify something. Moralty is a topic that interests many.

    Let me start by sketching out an overview. Feel free to question any particular aspect of it.

    Reality is made up of two things; matter or objects, and thoughts.

    Objects are hopefully obvious, they are things that can be pointed to, even if REALLY small. Cars, bananas, electrons, hair, are all objects. Everything else are thoughts. Thoughts are created in brains. We don't understand completely by any means how brains work, but we are beginning to. We understand they are made up of nerves, and we know that nerves work on a type of electricity, and this electricity is carried directionally by axons and dendrites via a chemical method known as the sodium potassium pump. And we know that people think thoughts. This is not speculation, these are all things that have been demonstrated.

    Just like a car has a primary function, to travel, and a tennis racket has a primary function, to hit a ball, the brain has a primary function, to create thoughts.

    Individual brains create thoughts. You or I can think banana, or tree. We have also through experience included more than object thoughts. We can think "fast car" or "good husband". These verb or non-noun words have been created over time, and are a special type of thought, I will call them "concepts".

    Concepts are created through societal refinement, and can change over time. So, when cars were first invented and traveled at like 8 or 10 miles an hour. If someone saw a car traveling at 11 miles an hour they might think "fast car", today with car often traveling at 60 miles an hour, that 11 mile an hour car would probably get the thought "slow car".

    The point there is concepts are relative. Concepts are thoughts that have grown beyond a single brain, but have been communicated, and understood by parts or the whole of society. I think it is important to understand that thoughts and concepts are different. I can have many thoughts, but unless they are communicated and integrated into other brains they remain thoughts, not concepts. There may be a better term for this idea of a thought that is shared by more than one brain than concept, but I will use the term for now.

    In addition, thoughts and concepts can be both real or not. For example, "yellow banana" is a real thought, "1000 foot long banana" is an unreal thought. Real means that it has some relation to reality. It can get complex. In some ways "Santa Claus" can be both a real and unreal thought. In that while we share an understanding of the concept Santa Claus and probably both know that he wears a red suit, and comes down chimney's and brings good girls and boys toys once a year, and in that sense the concept has aspects of reality, we also know that no such person actually exists on the earth, and so it is an unreal thought. While we may find interest in understanding the "reality of Santa Claus" as a concept, when people talk about "Is that real?" they usually mean "Does that concept exist in reality?". So it is important to understand if you are trying to describe and understand reality, or just trying to describe and understand an unreal concept.

    Ethics and morals is the study of what is "good". Good is a concept. So of course morals are going to be relative.

    I will leave it there for now, since comments have a like 4000+ max size, and I will try to cover this topic in a series or comments that each try to cover an aspect of the concept. It will not take 20 or 50 comments to get my entire thought across, but simply 5 or 6 I believe. But we will see.

    Cheers! RichGriese.NET

    ReplyDelete
  77. Bernard,

    I take Dianelos to be saying something like this (of course, I could be wrong):

    A "possible universe" is a way a universe could be, from start to finish. Now, there are many, many possible universes--as many as you can imagine, possibly an infinite number. Possible universes in which "N" is true are possible universes in which nature is causally closed in the way we are discussing. Presumably many of these universes are possible. Possible universes in which "G" is true are universes where a certain set of God's purposes are being met--relevantly, people have experiences of the sort we are discussing. Now, of course it is possible for people to have these experiences and for "N" to be true--for matter to be behaving in the way we would expect, given its previous states--so there are many possible universes in which both N and G are true. Possible universes where "H" is true are universes in which our free choices are correlated with what actually happens, so if I decide to raise my arm, I do (as opposed to, I decide to raise my arm, but don't.) My decision here is not directly causing my arm to be raised, a prior physical state is the direct cause. But it does cause it insofar as God makes sure my choices are correlated with the appropriate physical states. So, out of all the possible universes, it would not be surprising if God made one where N, G, and H are all true. And in fact that is what we find.

    Or, anyway, is that how I understood him.

    On a side note, that our subjective experiences are correlated in the way they are with our brain states--specifically, that they are coherent and (we think) reflect the world around us--does seem to me to pose a problem for naturalism above and beyond the mere existence of consciousness. Specifically, given the causal closure of the physical, your subjective states seem causally impotent, so that natural selection is blind to them (suppose one method of reproduction is immensely pleasurable, one is immensely painful, one feels like nothing at all, and one feels like taking a bath--natural selection will select based purely on their physical characteristics, and has no reason to select one over the other based on their subjective descriptions.) So that our subjective experiences are coherent and reflect the world around us, given naturalism, seems incredibly lucky (Hasker says it "has the appearance of sheer miracle.") So that would seem to suggest either that the physical is not causally closed, or else that God has somehow ensured the correlation.

    ReplyDelete
  78. Rich Griese,

    Yes, I live in Greece, and, yes, that wikipedia entry is mine. What interests me though is to discuss ideas, not myself.

    On the other hand should you, or anybody else here, ever come down to Greece you should certainly give me a call.

    ReplyDelete
  79. Bernard,

    You write: “ You say if naturalism holds, then there are still many possible ways the universe could have unfolded.

    Depends what you mean by “naturalism” in this context. Let me take a step back here and use Kant’s nomenclature, which I think will be helpful. The model I presented above is a model of reality, so it describes both noumenal reality (i.e. reality-as-it-is) and phenomenal reality (i.e. reality-as-it-seems to us when we look around or when science studies it). “Ontological naturalism” (or “philosophical naturalism”, or “metaphysical naturalism”) is a claim about noumenal reality. Theism is a claim about noumenal reality too. Theism entails that the fundamental causal principle of noumenal reality is personal free will, mainly God’s but to a smaller degree also the free will of humans. (As Tom Clark appropriately notes, the idea that we humans have free will is tantamount to saying that we are “small gods”, because free will makes of each one of us an uncaused cause to some degree.) Ontological naturalism denies this and claims that no free will exists in noumenal reality; rather the fundamental causal principle is, in some significant sense, blindly mechanical.

    Thus theists and ontological naturalists disagree about how noumenal reality is. On the other hand all educated people, whether theists or ontological naturalists, pretty much agree about how phenomenal reality is, because the natural sciences have produced a lot of knowledge about it. So by now it is well established that phenomenal reality is “causally closed”. This is the claim that one can explain all phenomena present in phenomenal reality using only blind mechanical causes and without assuming, say, the existence of personal agents in noumenal reality freely affecting them. That claim is also called “methodological naturalism”. (In what follows I interchangeably use both “methodological naturalism” and “causal closure”).

    Now here is the problem: Many people figure that if phenomenal reality is causally closed, then the noumenal reality that produces it must be causally closed too. In other words they figure that it’s not only the case that one does never have to assume the presence of agents in noumenal reality freely affecting phenomena, but that one can see that it is not the case that such agents freely affect phenomena. Why not? Because if they did freely affect phenomena then the causal closure of phenomenal reality would be violated, and at some point natural science would notice an anomaly. This problem is called the “interaction problem”, and many people consider it intractable. The model above demonstrates that this inference is wrong, for it describes a possible reality where phenomenal reality does seem to be causally closed under all possible scientific tests, while also being massively influenced by the free will of agents (both of God and ourselves).

    Science doesn't so much describe reality [snip]

    Science does not describe noumenal reality, but it does precisely describe phenomenal reality (or at least its objective dimension). So we know that given the state of phenomenal reality now, it is not like the state of phenomenal reality 10 seconds from now is fixed. Rather the state of phenomenal reality 10 seconds from now can be one out of a large number of alternatives, each with its own probability of obtaining. I start the model by showing that contrary to what many people assume science allows phenomenal reality to evolve in a way that breaks its causal closure, and thus loses what I call the “N” property (albeit the probability of this happening is exceedingly low). In any case as per methodological naturalism the phenomenal reality we observe does maintain the “N” property.

    [continued in the next post]

    ReplyDelete
  80. [continued from the previous post]

    Let’s now take the case of human free will. In the next 10 seconds I will have typed the next sentence in this text. Theism claims that I am genuinely free to choose what I will type, but ontological naturalism denies this, pointing out the interaction problem. What does science say? Science says that what I shall type is caused by the phenomenal state of my brain evolving according to fixed laws. These fixed laws though allow for many different evolutions and therefore for many different sentences that will be typed. Suppose now that theism is true, and I do freely choose out of a great number of alternatives what I shall type. Unless what I will type turns out to be exceedingly improbable given the current state of my brain, then the fact that I have freely chosen what I do type will not violate the causal closure of phenomenal reality.

    Let me illustrate this with a numerical example. Suppose a scientist with unlimited resources studies the current phenomenal state of my brain and computes the probability that my brain will type any of the huge number of sentences which are possible. Now I don’t speak French so the probability that my next sentence will be written in flawless French is exceedingly low (but according to science not really zero). Suppose that probability is then 1/10^1000. The probability that my brain will instead type the sentence “Let me illustrate this with a numerical example” is much larger, say 1/10^5. Should my brain’s state be observed to evolve in such a way that it causes the former sentence in French to be typed then the scientist would be very perplexed and would decide that the causal closure of phenomenal reality is broken (and it has thus lost its “N” property), for it would look like some supernatural power was guiding my brain to act in miraculous ways (even if strictly speaking science allows it). Should instead the state of my brain be observed to evolve in such a way that it causes the latter innocuous sentence to be typed, then the scientist would not find anything amiss. Indeed the probability of 1/10^5 of that occurrence may well be among the largest ones. As long then that my brain’s phenomenal state is observed to evolve in a way that is not so exceedingly improbable as to be highly surprising (according to science’s knowledge about phenomena) then the claim that this evolution did not simply “happen by chance” but was supernaturally guided by a free personal will, does *not* break phenomenal reality’s causal closure, nor does it in any other way contradict scientific knowledge. Ergo, the claim that human agents in noumenal reality freely and continuously affect phenomenal reality does not contradict anything that the natural sciences have discovered about phenomenal reality, as long as it is not so improbable on the science as to look miraculous. The interaction problem of human free will is thus shown to be a false one (it was based on the mistaken view of classical physics that phenomenal reality evolves deterministically). In other words phenomenal reality as we observe it and science studies it evolves in a way that maintains both the “N” (methodological naturalism) and the “H” (human free will) properties.

    [continued in the next post]

    ReplyDelete
  81. Dear Dianelos,

    Dianelos Georgoudis says;
    Yes, I live in Greece, and, yes, that wikipedia entry is mine. What interests me though is to discuss ideas, not myself.

    I was looking for your site or blog URL, so that I could read not just your comments here, but your actual essays in full.

    Also, are you a supernaturalist?

    And, do you feel philosophy is useful for determining reality?

    Cheers! RichGriese.NET

    ReplyDelete
  82. [continued from the previous post]

    But what about theism’s claims about God’s much larger influence in phenomenal reality? For example theism claims that God has designed and created humankind. Now, according to Darwinism, our species has evolved through a very long sequence of natural events in phenomenal reality. Suppose a scientist with unlimited resources were to compute the probability that, given the initial state of the universe, that very sequence of events would take place, and thus our species would evolve. That probability is of course very small, but the question is how small. If it is too small then the evolution of humankind could only be the result of a miracle. According to methodological naturalism this is not the case, in other words the fact that humanity has evolved even though very improbable is not really surprising, and therefore does not require the assumption of a supernatural guidance. But, if so, then God could have guided the evolution of humankind without violating methodological naturalism. So the phenomenal reality we observe and science studies maintains both the “N”, the “H”, and the “G” (God’s free will), at least as far as the claim that God designed and created humankind goes.

    Interestingly enough, the model I suggest places an upper limit on what we may assume is God’s special providence in phenomenal reality. As long as God wishes to maintain phenomenal reality’s “N” property (i.e. as long as God wishes to maintain methodological naturalism) God’s special actions cannot go beyond some point. So, for example, God is capable of instantly curing all cancer on Earth, but if God did this then the respective probability of such an event would be so small as to clearly strike a scientist as being of a supernatural origin and thus destroy phenomenal reality’s causal closure. (Why God would wish to maintain phenomenal reality’s causal closure is another question which pertains to theodicy; my concern here is to show that the interaction problem is a false one, and that there is nothing in scientific knowledge that contradicts either the reality of human free will or the reality of God’s special providence.)

    One of them you call the G, the God variant

    No, out of all the universes that according to science may evolve, there is not one but a vast number in which God’s special providence would obtain.

    You seem to say that in this version we ourselves have knowledge of God, [snip]

    Yes. If part of God’s special providence is to create humankind with a natural tendency to believe in God (or at least in the existence of a good and purposeful noumenal reality) and moreover with the cognitive capacity of discovering truths about God, then God could certainly bring about such a reality without breaking the causal closure of its phenomenal/scientific dimension. Indeed that’s precisely what Daniel Dennett is arguing: that religion can be understood by science as being a completely natural phenomenon.

    For in this world there would be evidence, would there not, of this transcendence, in the way the brain goes about its business of making meaning and indeed decisions. We would see unprompted, or spontaneous, brain states emerge.

    We do see unprompted/spontaneous brain states emerge. We see that in all physical systems, even in those we specifically design in such a way as to avoid them, e.g. in the case of digital computers.

    --

    Bernard, I hope I have clarified the idea. If not please let me know, I’d appreciate it.

    ReplyDelete
  83. Hi Dustin

    I don't know who Hasker is but he/she has led you astray here I think.

    So long as subjective experience is, as you say, correlated with physical brain states, then there is no problem with natural selection applying its filter to the nature of these states, indeed we should expect it to occur. All that is required is that the behavioural algorithm includes a weighting towards how a particular state feels to us (so I am more likely to repeat pleasant than non-pleasant experiences). This statistical tendency requires no causation beyond the associated brain state and we are not moving outside the closed causation system.

    So yes, miraculous in the way that so many of natural selection's tricks are I suppose. The more interesting question behind this for me is why would this subjective correlation evolve in the first place, and examining this may lead us closer to an understanding of what the subjective is. Perhaps for example it began as a powerful means of generalising behaviour (so categories of desirable and undesirable emerge over the more ponderous of evolving response mechanisms to each specific circumstance).

    I won't go on, evolution is something of an obsessive interest of mine, one that fits very nicely with an interest in philosophy.

    Bernard

    ReplyDelete
  84. All that is required is that the behavioural algorithm includes a weighting towards how a particular state feels to us (so I am more likely to repeat pleasant than non-pleasant experiences).

    I agree, but that's exactly the problem, isn't it?

    Given the causal closure of the physical, how we evolve (you could also say: how we behave) depends on facts about the initial conditions of the universe and facts about the laws of nature. And that is it. But there is no room in that for subjective facts (whether something is pleasant or not) to make any difference whatsoever. Even going with some sort of identity theory--in which subjective processes and physical processes are logically identical--does not solve the problem, as Jaegwon Kim (who is actually himself a naturalist) points out:

    "Davidson's anomalous monism fails to do full justice to psychophysical causation in which the mental qua mental has any real causal role to play. Consider Davidson's account: whether or not a given event has a mental description (optional reading: whether or not it has a mental characteristic) seems entirely irrelevant to what causal relations it enters into. Its causal powers are wholly determined by the physical description or characteristic that holds for it; for it is under its physical description that it may be subsumed under a causal law."

    So, again, if multiple physical configurations can do the same job, natural selection has no reason to prefer one or the other based on whatever subjective states (if any) are associated with it. The only ways of solving this problem that I can see are to deny the causal closure of the physical or to invoke God, or something very like God.

    ReplyDelete
  85. Plantinga makes a similar point about the subjective content of consciousness. If this content is epiphenomenal--a mere side effect of physical processes, which is causally inefficacious--then the "pleasantness" of a given behavior is causally inefficacious. But natural selection would systematically favor the correlation of the pleasurability of a behavior with its adaptiveness only if its being pleasurable somehow increased the creature's reproductive success (by increasing the frequency with which the behavior is done). It's hard to see how that could happen if the pleasurability of it were causally inefficacious (merely a side effect of underlying physical processes). To tell a coherent Darwinian story explaining why adaptive behavior is pleasant, we need the PLEASURE to be causally efficacious in increasing the frequency with which the adaptive behavior is done.

    Can we take pleasure (and other "qualia") to be causally efficacious in promoting behavior without either implicitly or explicitly rejecting materialism? THAT is the key issue here--and, obviously, it is a hotly contested one.

    ReplyDelete
  86. Let me chime in on this qualia/selection question. While it is pretty clear that consciousness follows and is secondary to most other phenomena in the brain, (decisions are made before we are conscious of them, etc.), that doesn't leave it with no function at all. Consciousness seems to have an integrating/monitoring function, leading to new memories and learning. For instance, when you reach the end of a thriller, it is only when the import of the plot "hits you" consciously that you are able to gain further insight into your recent experiences. I recognize this is all a bit vague, but we are all groping for an approach here.

    This implies that evaluations that enter consciousness, like pleasure, add to the feedback learning effects, and are obviously ripe subjects for selection. More consciousness is better for learning some things (though worse for others, like tennis, perhaps). So by this model, consciousness is identical to a subset of physical brain processes which have a function in some sorts of learning. It is real, not an epiphenomenon, just something whose correlates we haven't yet pinned down mechanistically.

    ReplyDelete
  87. Hi Eric and Dustin

    Yes, I am exactly with Burk on this. This business of seeing consciousness as an epiphenomenon is unhelpful. And it is certainly not a move forced upon the naturalistic interpretation I don't think. I see no problem with saying that consciousness just is the feeling of a particular brain state. And by this definition the answer to your question Eric is most definitely yes. A circumstance causes a brain state associated with a particular feeling. This brain state causes a particular action, precisely because of the association, and this action, and so the feeling by which we label it, is ripe for selection.

    You may be asking, yes, but why should brain state A associate with feeling B in the first place, when it may have associated with an opposite feeling? My answer is simply because the behavioural tendency toward repeating an experience for example, is exactly what pleasure is. Some feelings we associate with wishing a repeat, we call these pleasurable, some we associate with wishing to avoid, we call these unpleasant (Obviously this is a gross simplification). So the labels we have stuck to these feelings might just be our labels for the predispositions. It's not one thing emerging from the other or associating with the other. They're just the same thing. (Otherwise we get into the 'maybe my unpleasant feels exactly like your pleasant, and my green is your blue', game).

    The apparent contradiction between consciousness and selection comes when we assume there is more to the conscious experience than the brain state, and as I've said to Dianelos, I don't yet understand why that is a necessary step. People often say, because consciousness doesn't feel that way to them to which I can only reply, in all honesty, it feels exactly that way to me.

    Pragmatically, by treating consciousness as an adaptive feature of the mind all sorts of interesting fields of study open up to us (and this is perhaps a great model of the way philosophy and science do work hand in hand). Immediately we begin to ask about levels of consciousness. At what point in the evolutionary process did something like consciousness begin to emerge? What are the characteristics of brains, organisms and environments where aspects of conscious-like behaviour, and ultimately conscious like brain functions can be found? And what variations do we find in the observable aspects of consciousness (by which I mean behaviour, reportage and measurable activity in the brain). We begin to tease apart the difference between reflective, or self aware consciousness, and that which we tend to label qualia. Experiments in genetics and AI suggest themselves, and so we find ourselves in that hugely exciting process of deepening our collective understanding. This is for me the great emotional attraction of coming at the brain from a scientific vantage point, it offers such tremendous potential for learning. And without these wonderful philosophical discussions, we don't always know what the question is we're trying to answer.

    Bernard

    ReplyDelete
  88. Dianelos

    You have given me much to think about here. What you say makes sense to me but its complexity and my limited cognitive capacity demand I chew it over a while. Let me do just this.

    Thank you though for your patience in taking me through this. I have a great interest in the various interpretations of quantum mechanics and will need to see how my own prejudices in this area line up with what you're trying to show.

    Fascinating.

    Bernard

    ReplyDelete
  89. Burk, Bernard:

    I don't think either of you are really understanding what the problem is. You seem to want to say:

    1. Physics is causally closed--that is, the complete causal history of any given physical state will only reference the objective descriptions of the previous physical states and the laws of nature.

    But then you also seem to want to say something like:

    2. Feelings are physically causally efficacious--subjective states can play a role in determining what physical states obtain, which is to say, sometimes a complete causal history of a given physical state will have to reference the subjective description of a previous physical state

    But of course these are logically incompatible.

    So, for instance, if this is true:

    A circumstance causes a brain state associated with a particular feeling. This brain state causes a particular action, precisely because of the association

    (1) is false: physics is not causally closed, because in order to explain the action, you must appeal to a subjective feeling. In order to make it compatible with (1), you have to say something like: "A brain state occurs, and then another brain state occurs because of the objective characteristics of the last brain state. They have associated feelings, but those are just sort of there." Which is of course incompatible with (2).

    As for this:

    The apparent contradiction between consciousness and selection comes when we assume there is more to the conscious experience than the brain state, and as I've said to Dianelos, I don't yet understand why that is a necessary step.

    As Kim points out, in the quote I quoted, identity theory will not solve the problem. Eliminative materialism--saying that there is actually no such thing as feeling--*will* solve the problem, but it is insane.

    As for your last paragraph, Bernard, I am not saying you can't have an evolutionary study of the development of consciousness. I am just saying that seems incompatible with naturalism.

    ReplyDelete
  90. Okay, think of it like this. Two cars have a race. They are identical, except that one is more aerodynamic and painted red and the other is less aerodynamic and painted blue. Car 1, of course, wins. Now, "the red car" and "the aerodynamic car" are the same car. Someone who asked, "So which car won, the red one or the aerodynamic one?" would be confused or else making a joke. But it is true to say, "Car 1 won *because* it was aerodynamic," while it is false to say, "Car 1 won because it was red." Because while being aerodynamic and being red are both properties of the same car, they are still distinct properties--and only one of them was causally relevant to the car winning the race.

    The parallel here should be clear. *Even if* pain (for instance) is a property of a brain state, it is still a different property than, say, mass. According to the causal closure of the physical, only a certain set of the properties of that state are causally efficacious (basically, the properties whose values get plugged into the various physical equations.) That is just what the causal closure of the physical means. Qualia, on the other hand, given the causal closure of the physical, are causally impotent, and therefore cannot affect selection.

    ReplyDelete
  91. Hi Dustin and Eric

    Your tenacity is excellent Dustin. I like that you have the drive to make your point so clearly, and in this case it has helped me understand exactly the place at which our exploration has snagged, that of my slightly loose and careless definitions.

    I did, I think, understand the gist of the problem as you stated it, and tend towards the solution you yourself propose, and then dismiss as insane. I'm happy enough to claim insanity on this one.

    You are right, those characteristics which are causal are subject to selection. And so, by one definition of feelings for example, these are absolutely subject to and shaped by selection (although here we are mistreating the term selection somewhat by thinking of it working on isolated behavioural tendencies, no matter). This is the definition I had in mind when I blithely dismissed the problem you proposed and Eric supported as no real problem.

    You, I now see, and should have seen all along, are speaking of feelings as some thing that sit outside that which can be observed in the third person, the elusive first person experience of being and feeling, or qualia. I suspect these don't exist, and to you that makes me insane. Well all right. I'll explain myself anyway.

    Consider fear. The aspects of fear open to selection are manyfold. There is the change in pulse, body temperature, dilation of the pupils, shifts in hormone balances, the priming of particular memories, the tendency to select certain words when describing the state and so forth. All of these, and many more we don't understand, are physical phenomena open to selection. And so a tendency towards survival and reproduction friendly responses is selected for. No paradox.

    Now, take away all of those things and we are left with the fear that is something more. The feeling of fear. I think, when all the other aspects of fear are fully explained, there may be nothing left, in which case the selection paradox disappears.

    And if I'm wrong? Well, it took me a moment to realise this, but the paradox disappears anyway. Because the thing that is left, remainder fear, is inscrutable. If there is a residual thing called fear there is no way of me knowing if my fear is anything like your fear. Maybe my fear feeling is like the feeling you get drinking a glass of milk. When I get it, my heart rate races and my flight or fight mechanisms are primed. When you get it you go, ah, that's better and maybe burp. So the putative problem, of evolution miraculously providing a fit between reality and the non-selected fear feeling disappears, for even if qualia exist we have no reason to suspect the fit exists. The fit that does exist is the examinable one, between reportage, response and so forth.

    This of course is one reason to suspect the residual fear isn't real, although I admit not a compelling one. Waiting for science is like waiting for Godot. It's what we do while we wait that is important.

    Bernard

    ReplyDelete
  92. Bernard,

    I'm not really sure what to say. I mean, there *is* something that it is like to be me, and I'm pretty sure I know it. In fact, I think I know it more certainly than I know the things you mentioned regarding fear: maybe some evil demon has deceived me about the existence of pupils, but he has not deceived me about the existence of fear--I know fear exists, I have felt it. Learning more about the physiological responses you talk about doesn't seem to me to take anything away from that whatsoever. It is a different sort of thing conceptually. If you say there is nothing it is like to be you, then... well, I don't really know what to say. I read your book, it sure seemed like there was something it was like to be Anaximander. *I* have qualia, at any rate.

    I am not understanding what your argument in your next to last paragraph is supposed to be. Fear doesn't seem particularly mysterious to me: I know exactly what I mean when I use it, and other people seem to know as well. Yes, it is *possible* that they mean something completely different by it--but then, it is also possible that falling trees don't really make a noise when no one is around, or that the laws of nature will suddenly change drastically at 1 pm tomorrow, or that the world sprang into existence five minutes ago--and, in fact, any arguments for the serious possibility of your skeptical scenario would also, with slight modification, be able to prove the seriousness of those possibilities, as well. And then, I don't know, I guess we would all get together and have a good cry or something.

    So, in any event, the fit still seems quite evident to me.

    ReplyDelete
  93. Dustin,

    You write: “Now, there are many, many possible universes--as many as you can imagine, possibly an infinite number.

    The model is not about all logically possible universes, but about all physically possible universes, according to what science has found out about phenomenal reality. So, given a particular physical state of the universe, there are N possible physical states at some posterior time t, each with an assigned probability of obtaining. At time t one of these N possible states will be observed to obtain, and unless the corresponding probability is so small as to seem a miraculous state, the universe’s causal closure of the physical is not violated.

    Specifically, given the causal closure of the physical, your subjective states seem causally impotent, so that natural selection is blind to them (suppose one method of reproduction is immensely pleasurable, one is immensely painful, one feels like nothing at all, and one feels like taking a bath--natural selection will select based purely on their physical characteristics, and has no reason to select one over the other based on their subjective descriptions.) So that our subjective experiences are coherent and reflect the world around us, given naturalism, seems incredibly lucky (Hasker says it "has the appearance of sheer miracle.")

    I think William Hasker’s argument becomes clearer if one uses not all experiences, but (as you do above) only the fact that some experiences are pleasant and some painful. If reality is causally closed in the physical, and if all experience supervenes on the physical, then it seems that unguided Darwinian evolution would not be able to correlate pleasure with adaptive physical states, and pain with non-adaptive physical states. In other words, a blind Darwinian process appears to be incapable of producing the reasonable state of affairs, as far as pleasure and pain goes, we actually experience.

    Perhaps another way to say this is as follows: We all agree that the experience of pleasure is such that we tend to choose those physical states that produce it, and that the experience of pain is such that we tend to avoid those physical states that produce it. A blind Darwinian process would therefore use this fact to guide our choices towards adaptive behavior. But this will only work if we are really capable of choosing among various alternatives, i.e. if freedom of will exists. But scientific naturalism claims that free will doesn’t exist, because it does not comport with the causal closure of the universe.

    We see then that scientific naturalism’s account, if we try to cover all bases, becomes incoherent. So, something has go give: Either we are not the product of blind evolutionary processes, or phenomenal reality is not causally closed in the physical, or God exists.

    ReplyDelete
  94. Burk,

    You write: “ While it is pretty clear that consciousness follows and is secondary to most other phenomena in the brain, (decisions are made before we are conscious of them, etc.), that doesn't leave it with no function at all.

    Let me suggest a thought experiment that may help clarify the issue about whether consciousness has a function or not:

    Suppose we take a human brain and simulate its behavior in a digital computer to any degree of required precision (simulating individual synapses or even individual subatomic particles). If causal closure holds then this computer would behave exactly as the human brain does. If that computer were to read that thriller to the end it would explain its experience using exactly the same words as the human brain.

    Let us now count possibilities.

    Most of us assume that the computer (which is nothing more than a pile of interconnected on/off gates) is not conscious. So if the computer is able to react to reading the thriller exactly like the human brain without being conscious, then it follows that the human brain’s consciousness has no function at all.

    But perhaps a computer programmed with the brain simulation would become conscious. Indeed that’s the claim of the so-called “strong AI” hypothesis. Still, the output of the computer would be exactly the same, whether it was conscious or not. Why? Because the computer is designed as to be causally closed in the physical: given the physical state of the computer and its physical inputs one will always get exactly the same physical output (if not then the computer is malfunctioning). So, even if the computer has become conscious, its consciousness is an epiphenomenon with no influence at all.

    So far then it seems that our thought experiment has demonstrated that consciousness has no function at all. If we dislike this conclusion we must go back and deny some of the implicit assumptions the thought-experiment depends on. The possibility of simulating a human brain in a digital computer is based on two assumptions: That the human brain is a purely physical entity, and that reality is causally closed in the physical. If both of these assumptions are true then it is clear that one can simulate the behavior of a human brain in a digital computer.

    So we are left with the following three alternatives:

    (1) The brain is not a fully physical entity.

    (2) Reality is not causally closed in the physical.

    (3) Consciousness has no function at all.

    All three alternatives create serious problems for scientific naturalism:

    Affirming (2) goes against all that natural science appears to be saying. It used to be thought that by leaving no space for agent causality the causal closure of the physical creates problems for theism, but I hope I have showed above that this is not the case. Rather the causal closure of the physical, as Dustin has noted in this thread, creates trouble for scientific naturalism.

    Affirming (3) converts the reasonableness of conscious experience into an unsolvable mystery, unless the naturalist is willing to abandon the assumption that natural evolution was unguided.

    Affirming (1) is tantamount to denying scientific naturalism. Nevertheless there are some non-theistic ontologies that sidestep scientific naturalism as too simplistic, and affirm ideas such as property dualism. According to property dualism matter has both physical and mental properties, and the latter become active in brain-like structures. Now either these mental properties have causal powers over the physical properties, or they don’t. If they don’t then we get (3), i.e. consciousness has no function at all. And if they do then we get (2), i.e. reality is not closed in the physical. It seems then that property dualism does not help the non-theist a lot; but perhaps other ideas about a non-physical brain might.

    ReplyDelete
  95. Bernard,

    You write: “The feeling of fear. I think, when all the other aspects of fear are fully explained, there may be nothing left, in which case the selection paradox disappears."

    I can’t really visualize this idea making sense. Take the simpler case of experiencing the color red. Can you really say that once all other aspects of experiencing colors are explained, there will be nothing left? I mean what can learning about the human brain, or about whatever else for that matter, possibly add or take away from the actual experience of red? Whatever we may learn, the experience of the color red will remain there as clear and undeniably real as always.

    So eliminative materialism does not make any sense at all for me. Daniel Dennett has made a lot of effort trying to convince people that the problem of consciousness is illusory, but he has clearly not being able to convince his peers. After all the idea that consciousness is an illusion is self-referentially incoherent, because the concept of illusion presupposes the reality of consciousness. In a world without consciousness there can’t be any illusions. Other philosophers, such as Paul Boghossian, have pointed out deeper reasons of why eliminative materialism appears to be self-refuting.

    Now let’s take a big step back, and do some meta-philosophy.

    On the one hand we observe theists, who all believe that there is some kind of intelligence behind their experience of life. Further we observe that they have no trouble accounting for their consciousness, and for their sense of freedom of will, and for their sense of morality, and for their sense of rationality, and for the intelligibility of the physical universe, and for the success of the natural sciences in general. They believe that that transcendental intelligence is also benevolent, and are therefore troubled by the evil they experience in their life, but there is clearly some advance in solving the apparent discrepancy. For example some theists judge that John Hick’s development of Irenaean theodicy is a huge step forward. All in all then, we observe here a rather coherent and happy mental landscape.

    On the other hand we observe naturalists, who all believe that there is not some kind of intelligence behind their experience of life, and that reality is ultimately a blindly evolving mechanism. They suffer from many conceptual problems, which rather than resolving they tend to declare as “illusions” or “human inventions”. They have trouble making sense of their sense of freedom and of morality, so they tend to claim that free will is an illusion, and that moral truths are illusions too. Some naturalists think that the objective existence of mathematical objects and truths is illusory too; these only exist in some functional sense within human brains. Some claim that time exists only in the human mind and is really a human invention. Space too is a human invention. Even the laws of physics are human inventions (as physicist Victor Stenger writes in page 262 of his book “Quantum Gods”). Finally, some claim, consciousness itself is an illusion and does not really exist. It is certainly remarkable to observe that if one assumes that God is a human invention, then a lot of other basic concepts must follow suit. It looks like that removing belief in God from the center of one’s mental landscape leaves behind a kind of black hole, which slowly eats away at the rest. At the point that consciousness itself is denied, one’s entire mental landscape is absorbed into that black hole, because any rationality and any knowledge are contingent on the presence of consciousness.

    ReplyDelete
  96. Eric,

    You write: “ Can we take pleasure (and other "qualia") to be causally efficacious in promoting behavior without either implicitly or explicitly rejecting materialism? THAT is the key issue here--and, obviously, it is a hotly contested one.

    I think William Hasker’s argument cuts deeper than just making trouble for materialism. Materialism is a claim about noumenal reality, a space where much speculation fits. Rather Hasker’s argument points out a problem of incoherence between the fact of our conscious experience and a (claimed) feature of phenomenal reality, namely its causal closure in the physical. So Hasker’s argument does not sink into the metaphysics of the noumenal, but stays on the phenomenal level in which scientific knowledge is certainly relevant and bears much weight.

    I understand that Hasker himself, who is a theist, finds here reason to deny the causal closure of the physical. I personally find that a more powerful theistic ontology can be build by affirming the causal closure of the physical. My reasoning is (1) that by now the causal closure of the physical is pretty much demonstrated by the natural sciences, and (2) that as a theist I assume that God has designed an elegant phenomenal reality, in which it’s not like the physical is causally closed everywhere except in some tiny spots hidden deeply within human brains, or perhaps at the point far back in time in which life originated on Earth.

    ReplyDelete
  97. Historians that are not supernaturalists have now done a good job at explaining how the Christian Jesus myth began.

    Cheers! RichGriese.NET

    ReplyDelete
  98. Rich,

    Besides being off-topic, almost no legitimate historians take the Jesus myth hypothesis seriously.

    ReplyDelete
  99. Hi guys

    As always thanks for engaging in such a constructive way. I'm not trying to refute you, I have no interest in winning arguments per se, but I do enjoy the learning that takes place along the way, and already I have learnt a lot.

    You are right Dustin, radical scepticism is uninspiring. There is however room to be sceptical about specific claims and cases, could we engage in philosophy without this tool? And yes, I concede too that seeing red or feeling angry does feel like something to me, absolutely. As a writer I am in the business of examining, communicating and engendering such feelings. Perhaps that colours my approach to these questions, I'd not considered the possibility before.

    The argument seems to be that if there are such things as feelings above and beyond their physical manifestations, then this above and beyond aspect is not subject to natural selection. And, if these feelings so defined match reality in some way, well that is a mighty coincidence that needs explaining. My approach has simply been to point out the two ifs, nothing more. They both need to be engaged with somehow.

    For me Dianelos, and it would take a long essay to explain this, entertaining the possibility that feelings do ultimately reduce to their physical manifestations allows me at the very least to explore and understand these feelings (like the feeling of redness) far more deeply. It remains only a possibility of course, an open question. Your challenge, how do we have an illusion without something that is subject to illusion, is of course crucial. I don't for one moment claim we have an answer, and here I think Dennett sometimes goes too far. I remain chastised though by the reminder that once we did not have an answer to how one could have design without a designer. I am not trying to convince you this is the correct approach, I am simply, and I hope politely, reminding you that the question remains open.

    The second if, if these feelings like pleasure or seeing red, somehow match reality, is not only unknown but also unknowable. Maybe we do all feel things the same types of things Dustin, I suspect we do. But my suspicion is based on the selective, causative elements of feeling which are immune to the challenge. If we retreat to the ineffable version of feelings, then of course there is no way of knowing whether they are the same from person to person, or bear any relationship to anything. This is, I think, a limited and reasonable application of scepticism, just asking if we have evidence for one of the key premises in an argument.

    And so, not being able to reach beyond the two ifs of this case, I find the argument unproblematic. And I can in all honestly see why you would see things differently. I can also imagine myself seeing it differently five years from now, because the more honest I try to be about my conception of consciousness, the more slippery it becomes.

    Dianelos, I have not forgotten your quantum inspired case, but without going back and doing some more reading on the various interpretations (particularly those that posit information moving back through time, which seem relevant here) I'd just be bluffing which is unhelpful. I do hope to have the time to do this.

    Would love to discuss this business of what it means to see red with you in more detail at some time too.

    Thank you both.

    Bernard

    ReplyDelete
  100. does feel

    That is, in fact, all the argument needs. If you can feel at all--if there are such things as feelings--then qualia exist.

    I remain chastised though by the reminder that once we did not have an answer to how one could have design without a designer.

    That is not an analogous case. We are talking here about a conceptual difference. Saying, "Give science more time before deciding" might make sense if, say, we are talking about whether there can be an equation that unites the fundamental constants. But it would be silly to say "Give science more time" if the question at hand was whether a particular hot dog might be the number 2. That would not be reasonable; that would be the death of reason.

    This is, I think, a limited and reasonable application of scepticism, just asking if we have evidence for one of the key premises in an argument.

    But the evidence for this premise is just as good as the evidence we have for all sorts of uncontroversial premises. Certainly, it is about as good as the evidence we have for the validity of inductive reasoning in general. And while we might like to have a better justification for inductive reasoning that we in fact do, you still would not excuse someone who fired a bullet at you on the grounds that it was as likely to turn into a delicious, immortality granting chocolate as it was to kill you. Essentially, if we grant that this argument should keep us from believing the relevant proposition, then there is simply no point in talking about anything, because we can't even begin to know anything about anything and it's the end of the world.

    And, even then, suppose--for some reason--we withhold judgment on whether other people have analogous mental states or not. Well, I still know the fit exists for *my* mental states. And so, in fact, the argument still goes through.

    ReplyDelete
  101. Hi Dustin

    Oh boy. Not sure where to start, and I'm conscious of the need to respect what you're saying. Um...

    If I say that I have feelings, and I do, then I am saying qualia exist? Of course this depends entirely upon what we mean by feelings (I take it we agree that qualia are these aspects of feelings which do not affect behaviour and so are not open to selection). When I say that I feel pain, I do not in any way preclude the possibility pain just is the mental network of memories, predispositions and physiological responses associated with the state in which I utter the sentence 'I feel pain' and mean it.

    And we can go further. Any aspect of this painful feeling that I can describe is an aspect open to selection. Why? Because the process of describing is also the process of categorising and clearly being able to categorise an event as painful has massive survival advantage. The availability for selection holds for any aspect of feelings I can describe. So, whatever qualia are, there is nothing of their nature that we can share. Even the 'I just know this feeling is more than that' statement predisposes me to certain beliefs and actions and is open to selection.

    The qualia crucial to this argument then is an undiscussable, but personally available quality. I can say I have feelings without embracing this aspect and know what I mean by it.

    You say the design analogy doesn't hold. I think you say it doesn't hold because qualia just are a different category of thing. But I don't see how we can, prior to investigation, make this claim, except as a matter of faith. Maybe it is possible to have an illusion that works on itself. And maybe when we fully know how the brain creates its illusions, this will become apparent. We can claim that this problem is analogous to whether a hot dog is the number two, but only after we have made a prior decision about what qualia are. And this is viciously circular. Note I am not arguing qualia don't exist, but just that they may not and further investigation before jumping to conclusions is very sensible.

    And then finally, we have the argument that you know your qualia in some sense fit reality. Now I can ask how do you know, and you are restricted to saying, well I just do, because the aspects of your feelings we can discuss are precisely those open to selection and therefore not part of the problem at hand.

    Somehow, for someone like me to get a better understanding of theism, which I wish to do partly out of respect for the great many thinking people who hold these views, there has to be a way of having a discussion that doesn't reduce to 'because I just know.' Because if that's the foundation, why bother with any sort of logical construction at all?

    Bernard

    ReplyDelete
  102. If I say that I have feelings, and I do, then I am saying qualia exist?

    When I say "qualia," I mean something like, "the subjective quality of conscious experience." If you are using "feeling" in such a way that it does not entail it being a subjective quality of conscious experience, then I guess you are not saying qualia exist. But I am not sure I can be wholly blamed for misunderstanding you, since it is, I think we must admit, an awfully strange way to use "feeling."

    But I don't see how we can, prior to investigation, make this claim, except as a matter of faith.

    I guess it depends on what sort of investigation you mean, but...

    The properties described by physics are, definitionally, publicly available. Qualia, on the other hand, are subjective. Now, I guess you can say, how do I know that I have subjective experiences? And all I really can say is, well, because I am having subjective experiences. I mean, it is just self-evident to me. I literally cannot even begin to conceive of a situation in which--my mental life, my feelings, whatever terms you find acceptable--are the way they are and I am not having subjective experiences. And that sort of thing--it is just self-evident--is the basis of our most certain beliefs (most of our beliefs rest on much less than that.) If someone asked you, how do you know a=a, you would say, well, it is self-evident. And if they said, "But that is begging the question, because of course I will not accept that it is self-evident unless I already accept that a=a," then you might figure that there was no way to convince them, but you probably would not be too shaken in your belief.

    So I guess if it not evident to you that you are having subjective experiences, we can go ahead and say that you are not going to be impressed by the argument. Of course, this doesn't make the argument pointless, because the vast, vast majority of people will agree that it is self-evident that they are having subjective experiences.

    ReplyDelete
  103. By the way, I'm sorry if I've myself come across to you as uncivil or disrespectful or whatever, but surely you know that starting your remarks with "I'm conscious of the need to respect what you're saying" is a way of disrespecting what the other person is saying...

    ReplyDelete
  104. Dustin,

    You write: “So I guess if it not evident to you that you are having subjective experiences, we can go ahead and say that you are not going to be impressed by the argument. Of course, this doesn't make the argument pointless, because the vast, vast majority of people will agree that it is self-evident that they are having subjective experiences.

    If I understand Bernard’s position correctly, what he means is not that he is not sure that he has subjective experiences, but rather that he is not sure that his subjective experiences are anything over and above his body’s physical processes and/or predispositions to behave in a certain way. So, he figures, perhaps by understanding the latter better (which is an ongoing scientific project) we *may* find out that subjective experience really is nothing more that these physical processes and/or predispositions.

    I have the feeling that many naturalists, when confronted with the famous problems related to the philosophy of mind, make the following inductive argument: There have been many cases in the past where science explained on purely physical grounds phenomena which everybody thought must have a transcendental source. A famous example is how people used to think about living things: "The behavior of living things is so dramatically different than the behavior of non-living things, that it can’t possibly be the case that living things consist of exactly the same kinds of physical bits that non-living things consist of and nothing more. Rather some non-physical 'spiritual breath' or 'life force' or 'soul' must exist which animates non-living matter and makes it a living organism." Yet science proved that no such non-physical things were necessary to explain life, for, as we now know, life is nothing more than a complex but entirely physical electrochemical reaction. Similarly, consciousness does now appear to be a phenomenon that is so different from everything else that it can’t possibly be explained on purely physical grounds. But, as has been the case so many times in the past, science may yet find a way to explain consciousness on purely physical grounds. Indeed brain science is continuously discovering new knowledge about the most complex thing there is in the universe, namely the human brain. Perhaps future discoveries will be such that all mysteries related to the mind will simply evaporate. Given the huge success of science in the past, how can we be certain that science won’t surprise us once again?

    ReplyDelete
  105. If I understand Bernard’s position correctly, what he means is not that he is not sure that he has subjective experiences, but rather that he is not sure that his subjective experiences are anything over and above his body’s physical processes and/or predispositions to behave in a certain way.

    If this is his position, I am not sure I understand it. Physics, by nature, only includes objective properties. If objective properties are all there is, then there are no subjective properties. So it seems to me that we would then be being asked to wait to see whether science might discover something that is logically impossible. Science describing pain solely in terms of the objective properties of the physical process would be science showing that pain was not subjective after all. It would be like someone saying science might show that life consisted only of electrochemical processes while also saying they were certain of the existence of the elan vital.

    ReplyDelete
  106. Hi Dustin

    Can I just apologise for starting with the respect comment, which you are quite right, reads tersely, and expresses my own frustration at not making clear the point I think Dianelos has expressed very well. So frustration with myself, not you.

    Bernard

    ReplyDelete
  107. Dustin,

    You write: “If this is his position, I am not sure I understand it. Physics, by nature, only includes objective properties. If objective properties are all there is, then there are no subjective properties. So it seems to me that we would then be being asked to wait to see whether science might discover something that is logically impossible

    Well, I don’t understand any eliminative or identity theory of the mind either. But I think I can understand a person who embraces such theories as *perhaps* true. The reasoning may go as follows: Only a few centuries ago we knew of no non-living systems with the power of self-locomotion, so it seemed obvious to us then that to be capable of self-locomotion some extra ingredient, such as the élan vital, is needed. Similarly, today, it seems obvious to us that no purely physical system can be conscious and some extra ingredient is needed, but perhaps we are wrong.

    Now above you write that the physical realm, by nature, only includes objective properties. Well, perhaps this is a false assumption right there. A property dualist, like David Chalmers, hypothesizes that material things possess both physical and mental properties. It is true that, up until now, the scientific project of physics only concerns itself with the physical/objective properties of matter, but, Chalmers suggests, future physics may concern itself also with the mental/subjective properties of matter, while preserving, broadly speaking, science’s epistemological ethos. Data, whether objective or subjective, whether third-person or first-person, are still data, and hence there is nothing to keep science, broadly understood, to use both kinds. As theists we can’t but agree with Chalmers’ suggestion, because we too use both kinds of data in natural theology, which cognitive field was for a long time considered a science (and I think should again be called that). Of course Chalmers is talking about a new kind of physics with an epistemology which is greatly evolved over its current state, but why shouldn’t the epistemology of physics evolve? As a theist I believe that all truth leads to God (for God is the foundation of all reality), so any new knowledge, whatever its source, cannot but help us see God clearer.

    Having said all that, I personally find that there are too many good arguments against the thesis that our brain is the source of our consciousness for that belief to be tenable, at least within a naturalistic ontology. (Previously I expounded an in my judgment strong argument why consciousness, if it supervenes on the brain in any way, cannot serve any function.) On the other hand, a purely materialistic sense of consciousness does attract many people. As you probably know there are even theistic philosophers who are materialists as far the human mind goes. Indeed, traditional Christianity holds that the human soul cannot properly exist if it is not co-joined to a physical body in good working order (hence all the talk about physical resurrection and so on). So the whole question is still in the air. Let us wait and see if those who hold a materialistic conception of the mind will come up with good defenses or good arguments. For now though, in my judgment, the view that consciousness is primary and basic, and the material is secondary and contingent (just as theism, and especially idealistic theism, has it) is a much more coherent view. Indeed, as far as I can see, this view is completely free of conceptual problems.

    ReplyDelete
  108. Bernard,

    No problem.

    Dianelos,

    I am not sure how that position would help. Suppose we grant that there is one sort of process--brain processes--and that this process has both subjective and objective properties. Furthermore, let's grant that some sort of identity theory is true, and that these sets of properties logically imply one another. Further, let's suppose that some future physics will expand its study to include these subjective properties.

    And the question is, so what? The claim being made regarding the causal closure of the physical is that the properties studied by our *current* physics are the only ones that need to be invoked in any explanation of why a particular state obtains. So if we say, well, maybe future physics will show that these subjective properties play a causal role, then the causal closure of the physical, as it has been used until now, is false. And if we say, no, the properties studied by our current physics explain why any given physical state obtains, then even if our future physics includes subjective properties, it will include them as causally irrelevant and therefore irrelevant to selection.

    So it still seems to me that, to solve the problem, you have to actually go for a fully fledged eliminative position and just say that the fit doesn't exist because there isn't anything to fit (and in fact, most of what Bernard has said has seemed to suggest to me that this is what he wants to say, even though he did just now say your statement of his position was an accurate one.) But of course, that solves the problem at the expense of introducing an even greater one.

    ReplyDelete
  109. Hi, Dianelos:

    "Let us wait and see if those who hold a materialistic conception of the mind will come up with good defenses or good arguments."

    1. Nothing whatsoever is known about non-material mechanisms in any aspect of reality, especially in biology. This is an amazing statement, considering the discussion we seem to be in, with beliefs in a beyond or the supernatural, hopes expressed in souls shown here and very commonly in the culture. But despite all the claims, there is zero knowledge. The reply would be that supernatural is by definition beyond our perception and knowledge. But then why the claim at all? It is self-contradictory to claim that supernature "explains" real phenomena but that we can't know anything about it because it is inscrutable. This is especially true if the supposed soul causes not only feelings of subjectivity, but free will actions that are efficacious, reality-wise.

    2. Here is a random scholarly article on brain functions- pain impairs cognition to a limited extent, and both are mappable in the brain by today's crude methods. Pain is a physical phenomenon, from the rock that hits us on the head, to the brain areas that process the information and ultimately constitute our consciousness of it. OK- that was bare assertion, but the issue is that studies like this show that the pain and other experiences we think of as "consciousness" are trackable through the brain, are dissectable, and have mutual effects explicable by those tracked brain pathways. What is left for a supernatural theory to supply? Not much, and less as time goes on.

    By supernatural theories, it is quite inexplicable that we would have brains at all. Our nerves (were we to need nerves at all) would lead to the pineal gland, where the disembodied soul would communicate back and forth, constructing our feelings and actions. Why all the evident processing of parietal, medial, thalamus, etc. etc. in the endless anatomy of the brain? Why those cells that respond to the face of Bill Clinton? What would be the point?

    The supernaturalist ball just keeps getting pushed back in the face of knowledge, to the recesses of what for the moment we don't know for sure. That does not seem like a promising trend, even if you think by your philosophy that the "hard problem" of consciousness is somehow insulated from ultimate solution in these ways. I wouldn't be so sure.

    ReplyDelete
  110. Bernard,

    In fact, I feel like there has been some confusion on this point--or at any rate, I am confused, so let me ask, are you saying:

    1) There are no such things as subjective properties (this is the view I called insane, and that you then agreed with)

    or

    2) There are subjective properties, but they are properties of brain states (this is the view Dianelos seems to attribute to you, and that you agreed with)

    2) does not solve the problem for the reasons I mentioned in my last post. 1) does, but, in the view of the vast majority of people, at the expense of creating a far greater problem (including, from what I can tell, a pretty substantial majority of naturalist philosophers, who generally accept the reality of the subjective, even though they do so begrudgingly (as evidenced by the title of Jaegwon Kim's book "Physicalism, or Something Near Enough."))

    ReplyDelete
  111. Hi Dustin

    Good question, and one I find hard to answer because I'm not yet sure what you mean by subjectivity. I'm not trying to be slippery here, I think it is a genuinely difficult concept to get a shared understanding of, and you're absolutely right to try to clear it up, because it sits at the heart of our different approaches to this question.

    So, perhaps I can get a better understanding of what you mean by subjectivity by asking you some questions, if that's okay, and from there I'l be able to tell you which of the two options I favour. (My instincts are fallibilist so favour is about the best commitment you'll get from me, I don't claim to know anything about anything but do claim to be able to make judgements about which style of guesses might best serve particular purposes).

    Subjectivity seems for you to mean something that is non-causal and hence not available to selection. Is that correct?

    You made a statement at some stage that you just know your subjective feelings match reality, is that a fair interpretation?

    If I have read you correctly I become confused, because this type of subjectivity is causal, insomuch as it causes you to have a particular belief about the world, namely that your qualia are in tune with reality in some way. And beliefs are open to selection because they predispose us to associations, motivations etc. So I am misinterpreting your use of subjectivity here I think. Am I, and in what way?

    I also don't know what you mean by qualia matching reality yet. Two people look at a bush with red flowers. One is colourblind. They both process the information differently and presumably, if qualia are at least linked to incoming information, experience different qualia. So by subjectivity you don't presumably mean the feeling associated with viewing the flowers, because in this case there is no good reason to believe there is match between qualia and reality, indeed here is a good example of no such match existing (and then along comes a bumble bee equipped to enjoy the flower's infrared pattern...) So, the linked question, if this is not what you mean by subjectivity, what do you mean?

    I am looking for a style of subjectivity that would meet the requirements of your evolution puzzle, that is, that we know matches reality and that is non-causal. And because I can't comprehend what this might be, I tend to tell you I don't believe in subjectivity, so your first option, but if can answer those questions for me I will be able to clarify.

    Bernard

    ReplyDelete
  112. Hi Dustin

    I thought of another way of explaining my position. How did subjective consciousness evolve? Well, consider three options.

    At some stage, as brains became sufficiently sophisticated, a sort of what it feels like to experience something sensation began to hum through organisms. And in time, one organism evolved the sense of self, language, introspection etc to associate this feeling with their subjective experience of reality.

    Option one, there is no such subjective experience, the reflective brain, by some not yet understood mechanism, tricked itself into believing there was. The humming is all there is, and this humming (a clumsy metaphor) just is the activity of the brain associated with any given experience. This brain behaviour is clearly shaped by selection. No problem.

    Option two, there is such a subjective state, and it evolved randomly. Maybe some qualia reflected reality well, others were totally misleading. But, being non-causal, these had no impact upon survival rates, and so we all have very different qualia, which bear no resemblence to reality at all. We all believe ours are the same and roughly right, but we're wrong. No problem for evolution here.

    Option three, we were by some unexplained mechanism able to form accurate beliefs about the fidelity of our qualia, even though in every other respect they remain non-causal. Those emerging mutants that had non-accurate representations knew it, and this made them rather glum. Due to their existential angst, they became poor partners and parents and were quickly selected out. We ended up with qualia which are accurate, we know they're accurate and we got here through selection. No problem for selection here.

    This does not show what I personally believe qualia to be. I don't think we have enough information to rule out or embrace option one yet. But these are the only options I can think of and none cause problems for evolutionary theory. So you will be able to help me understand your version of subjectivity by describing option four, under which evolution fails and god is needed to guide the development of qualia.

    Bernard

    ReplyDelete
  113. Subjectivity seems for you to mean something that is non-causal and hence not available to selection. Is that correct?

    If the causal closure of the physical holds, then, yes, that would seem to be definitionally true. (I am not making any statements here about whether physics is causally closed or not.)

    If I have read you correctly I become confused, because this type of subjectivity is causal, insomuch as it causes you to have a particular belief about the world, namely that your qualia are in tune with reality in some way.

    Given the causal closure of the physical, my beliefs about qualia are not caused by qualia. They are caused by facts about previous physical states and the laws of nature.

    I think Dianelos' suggestion that we behave as if we have free will might be a good starting point for explaining what the fit is supposed to be. It is about more than just that, but, for the sake of simplicity, let's bracket it and say that what we mean by the fit is that we behave *as if* qualia made an impact on our behavior (you might say: our qualia are as if they had been selected for, to encourage us to repeat or refrain from certain activities.)

    So, then, your option 1 violates the reality of the subjective, your option 2 violates the reality of the fit, and your option 3 violates the causal closure of the physical.

    ReplyDelete
  114. Dustin,

    You write: “No problem for evolution here.

    If I understand correctly Dustin’s argument, he is not saying that there is some problem with evolution. Rather, it’s precisely because evolution and other scientific theories can explain all physical phenomena including human behavior, that the fact that our subjective experience of life is so rational (pleasure and pain well apportioned, etc) produces a difficult conceptual problem for naturalism to solve.

    Option three, we were by some unexplained mechanism able to form accurate beliefs about the fidelity of our qualia, even though in every other respect they remain non-causal.

    That’s an interesting option, but I think one can see it’s a logically impossible one on naturalism. After all, if our beliefs are (or can be represented as) physical processes in our brain then in order to build accurate beliefs about qualia, qualia must have some causal efficaciousness on the physical.

    This does not show what I personally believe qualia to be.

    What qualia are is not a matter of personal belief; rather, one forms beliefs because of qualia.

    Perhaps you are saying that we may be confused about qualia. But a quale is an elementary experience, so it is not possible to be confused about *it*, one can only be confused or wrong about how one reasons thinking *about* it. For example, one can’t possibly be wrong, or in any way be confused, about how it is like to experience the color red. In other words, qualia is where the cognitive ball starts rolling as it were. So to deny or question qualia is to effectively deny or question rationality. Which leads to nihilism, which is kind of a discussion stopper, not to mention a thought stopper.

    A related issue, where I find people are often confused, is the issue of illusions. If you think about it you’ll see that that there are *no* illusory experiences. An experience cannot possibly be an illusion. An illusion can only be an inference we draw on the experience. So the experience that a pencil in a glass of water looks broken is not illusory, for the pencil does indeed look broken. The illusion is to think that therefore if one takes the pencil out of the water it will still look broken.

    ReplyDelete
  115. Oops, sorry, but my previous post was directed to Bernard, not Dustin. This one is for Dustin:

    You write: “I think Dianelos' suggestion that we behave as if we have free will [snip]

    No, that’s not at all what I suggested. Rather what I described is a theistic model of reality, in which we are spiritual beings having free will and using it to freely interact with a physical phenomenal world without violating its physical closure.

    ReplyDelete
  116. Hi Dustin

    Thanks, that clears things up pretty well. I, like you, don't like option three at all. Unlike you, options one and two I'm happy with as strong possibilities. Yes, they both violate premises of yours, and I'm more than happy with doing this for the purposes of my own reasoning. I have no trouble at all believing there are no feelings of the sort you describe, and no trouble believing that if such feelings do in fact exist that they do not fit reality (as per the colour blind friends).

    So, for me, there's no need to insert god. Violating the qualia reality is not like violating inductive reasoning, as per an earlier example. I am a pragmatist, and because qualia are non-causative, what we assume to be true about them has no impact upon the way we think about anything, so there are no implications for the way we interact with one another if this fit assumption is violated.

    Shall we leave it there, agreeing to disagree because of the assumptions we make from the get go? I'll let you have the last word now on this one. Dianelos, this idea of qualia you have appears to me to lead with its own conclusion so maybe I'll return to this business of redness.

    Bernard

    ReplyDelete
  117. Yes, they both violate premises of yours, and I'm more than happy with doing this for the purposes of my own reasoning.

    I would point out that they are not just premises of mine, but premises of probably something like 99.999% of the human race--including the substantial majority of philosophers, including the substantial majority of philosophers who are naturalists.

    Violating the qualia reality is not like violating inductive reasoning, as per an earlier example.

    Justifying inductive reasoning is actually *extremely* difficult--so difficult, in fact, that most philosophers have just given up on trying to do so, even as they continue to use inductive reasoning (and of course, as Wesley Salmon points out, induction must be rationally justified in order to be pragmatically justified.) So--while, of course, no one would really doubt the validity of induction--I think I actually have much better reasons to believe in qualia than in the rationality of induction, and am, in fact, more certain of the former than the latter (some people have suggested that the validity of inductive reasoning suggests an argument for the existence of God, and I am inclined to agree--but we will not get into that.)

    I am a pragmatist, and because qualia are non-causative, what we assume to be true about them has no impact upon the way we think about anything,

    That *qualia* are non-causative does not imply that *beliefs* about qualia are non-causative--I mean, if we are willing to grant that beliefs are otherwise causative. If I think torturing a child will cause her pain, that gives me a reason not to do so.

    Shall we leave it there, agreeing to disagree because of the assumptions we make from the get go?

    Fair enough.

    ReplyDelete
  118. Dianelos,

    No, that’s not at all what I suggested. Rather what I described is a theistic model of reality, in which we are spiritual beings having free will and using it to freely interact with a physical phenomenal world without violating its physical closure.

    Yes, of course, I expressed myself poorly there. What I meant to say was something like, your suggestion that we could understand the fit by thinking about the fact that we behave as if we have free will (which, of course, you and I both think we really do have free will.) Specifically, I was thinking of your statement here:

    In other words, a blind Darwinian process appears to be incapable of producing the reasonable state of affairs, as far as pleasure and pain goes, we actually experience.

    Perhaps another way to say this is as follows: We all agree that the experience of pleasure is such that we tend to choose those physical states that produce it, and that the experience of pain is such that we tend to avoid those physical states that produce it. A blind Darwinian process would therefore use this fact to guide our choices towards adaptive behavior. But this will only work if we are really capable of choosing among various alternatives, i.e. if freedom of will exists.

    ReplyDelete
  119. Bernard,

    One last thing. Regarding the fit, even if we were to just withhold judgment about what other people's qualia are like, there is still the fact of *my* qualia--the fact that my qualia are as if they had been selected for, to encourage me to repeat or refrain from certain activities. So I do not see how your option 2 addresses that.

    ReplyDelete
  120. A number of people have expressed to me that the 5 articles by Eric on what naturalism is, and the subsequent discussion here, have left them unclear on what naturalism and supernaturalism are, or more specifically what is the fundamental difference.

    I would recommend this very good essay by Richard Carrier for anyone that is still interested in understanding the difference between supernaturalism and naturalism.

    Cheers! RichGriese.NET

    ReplyDelete
  121. Dustin,

    You write: “So if we say, well, maybe future physics will show that these subjective properties play a causal role, then the causal closure of the physical, as it has been used until now, is false.

    If the causal role they play is of a mechanistic nature then the causal closure of the physical will be maintained.

    And if we say, no, the properties studied by our current physics explain why any given physical state obtains, then even if our future physics includes subjective properties, it will include them as causally irrelevant and therefore irrelevant to selection.

    Right. On the other hand I am not sure you are not covering all the bases here.

    Let’s put on our naturalistic hats on and consider how property dualism may offer non-theism an avenue to make sense of consciousness: So let’s suppose that matter is such that its mental properties become active in brain-like structures. Such mental properties, once activated, will, ex hypothesi, have causal powers on the respective physical properties of the brain, and vice versa. In this way then these mental properties will be indirectly visible to evolution and will thus be subject to selection. (Actually there may be ways to use property dualism to expand the mechanism of evolution itself, and model evolution as a not exclusively physical process, but let’s not go there.)

    Now does the above model violate the causal closure of the physical? It depends:

    If the laws that govern the causal interaction between the mental and the physical properties of the brain are purely mechanical (i.e. amenable to mathematical modeling) then the causal closure of the physical is not violated. On the other hand this alternative leaves no place for free will, but some people appear not to be overly troubled by that idea (but I for one cannot even conceive it as possibly true). Anyway that’s one alternative.

    If, on the other hand, if the laws that govern how the mental affects the physical are be not purely mechanical (in a way that would mimic theism’s understanding) then there is space for free will, but the causal closure of the physical is violated. (It seems only theism allows for the reality of free will without violating the causal closure of the physical.)

    The question of which of the two above mentioned alternatives is true is an empirical one. If some future kind of natural science (which takes into account both objective/third person data and subjective/first person data) finds that the causal laws between physical and mental properties are not amenable to a completely mechanistic understanding then the causal closure of the physical will have been proven to be false. Such a discovery would not be disastrous for non-theism. After all the causal closure of the physical is not a necessary principle of science, nor a completely certain property of physical phenomena as many non-theists harp on because they believe (falsely it turns out) that the causal closure of the physical makes trouble for theism. For all we know it is possible that the brain is a radically different kind of material object, as indeed some thinkers, e.g. Roger Penrose, believe is the case.

    ReplyDelete
  122. Dianelos,

    Okay, I think I see what you are saying now. You agree that, if the objects of our *current* physics are causally closed, qualia are causally irrelevant. But Bernard thinks we have very good reasons to think the objects of our current physics are causally closed--otherwise, his earlier argument that physics leaves no room for free will makes no sense. If we grant things like property dualism, then some future physics might be able to explain how qualia are selected, but only by violating the causal closure of our current physics. And Bernard thinks there are very good reasons to think our current physics is causally closed.

    ReplyDelete
  123. Dianelos

    You consistently write striking things that spark my thinking, thank you. I would like to follow up on one of these if I may. You write:


    What qualia are is not a matter of personal belief; rather, one forms beliefs because of qualia.

    Qualia are, by your offering, elemental things that we can not be right or wrong about. I won't follow up the causation that I perhaps incorrectly read in the because, I've run out of steam on that line for now. Much more interesting is what people mean by qualia. Are you stating in the above quote that I simply am not allowed to believe that qualia is an ill-defined term? I suspect it is, but if I reach that conclusion have I on your terms broken a law of reasoning?

    Here's how the term may be ill defined. What if qualia, the sensations of consciousness engaging with the world, just are information processing. Not caused by, emerging from, contingent upon, but just the exactly same thing. If so then when we treat brain processes and consciousness as subject and object together in a sentence we get into a tangle caused by our language rather than any state of affairs in the world. I think I have fallen for this trap during this discussion anyway.

    Why not have consciousness relate to information processing as equidistance from the centre relates to a circle. We do not get too terribly exercised by this mysterious property of equidistance that emerges from a circle. We do not try to analyse the process by which circles produce the quality of equidistance. Circles just are shapes defined by equidistance. We accept this and shuffle on.

    It may just be that this is the paradigm shift naturalists urge and it feels counter intuitive at first, as paradigm shifts are want to do. Under these conditions I can easily accept your statement regarding knowledge of qualia.

    Under this view there is no such thing as subjective knowledge. All knowledge is in principle objective. Were we to know and be able to reproduce a brain state, we would know reproduce its qualia too. It's just in practice very hard thing to measure, as was Brownian motion for a while there.

    Consciousness then becomes a little like the essence of cat or dog that plagued naive attempts to grapple with speciation. A difficult thing to pin down because it doesn't exist in the terms proposed.

    So there is no such thing as the experience of seeing red. Every time I look at a red thing the experience is different (I have tried this a lot) and smacks of the act of remembering, recognising, considering, being surprised by etc. Qualia just are these patterns of activation within the brain. And so, through careful study we can, as Burk proposes, get to know consciousness far better.

    I think this is a very attractive approach for me, not least because it places free will within the deterministic framework I need before free will can have a truly moral quality.

    As always, i do not propose this is how the world really is, I propose it's an excellent way of learning more about oneself and experience in the world. And the more I dwell upon it, and the more I read of cognitive science, the more intuitively appealling it becomes.

    I tell you all this only because I find you are very good at taking in the big picture and may be able to say, well no, there's a glaring logical misstep here, in which case I will have to retreat from the position and go quale hunting anew. And if this feels a tedious obligation to you, feel free to ignore these thoughts.

    Bernard

    ReplyDelete
  124. Burk,

    You write: “1. Nothing whatsoever is known about non-material mechanisms in any aspect of reality, especially in biology.

    True. And, on theism, that’s exactly how it should be. Because, on theism, there is a physical/material dimension in reality in which mechanisms are present, and there is a non-physical/spiritual dimension in which mechanisms are not present. All causality in that latter dimension of reality is based on agent causality, i.e. causality driven by some person’s free will, and hence not amenable to mechanistic modeling.

    The reply would be that supernatural is by definition beyond our perception and knowledge

    Who says that the supernatural is beyond our perception and knowledge? Certainly not theism. In fact, on theism, we ourselves are supernatural beings, as is all of our conscious experience of life, including then our rationality and the knowledge we build on it.

    Perhaps you are confused with the traditional theistic saying that we can’t possibly understand all of God as God in Him/Herself is. But theism’s claim is not that God only is supernatural and everything else is mechanical. Rather theism’s claim is that all of reality is ultimately God-structured, and thus, ultimately, of a spiritual and not mechanical nature.

    This is especially true if the supposed soul causes not only feelings of subjectivity, but free will actions that are efficacious, reality-wise.

    On theism free will does exist and is efficacious in reality. Theism has no trouble making sense of our everyday experience of life.

    2. Here is a random scholarly article on brain functions- pain impairs cognition to a limited extent

    There is a common confusion here. I have mentioned this issue before, but let me here actually quote what famous atheist philosopher Bertrand Russell wrote almost 100 years ago in an introductory book on philosophy: “When it is said that light is waves, what is really meant is that waves are the physical cause of our sensations of light. But light itself, the thing which seeing people experience and blind people do not, is not supposed by science to form any part of the world that is independent of us and our senses”. What Russell is saying is that what scientists mean when the speak of “light” and of its properties, is not the same as what people mean when they speak of “light” in the context of their experience of life as conscious beings. Rather scientists mean by “light” that which under normal circumstances causes us to consciously experience light. Perhaps this point can be made clearer by pointing out that we can and often do experience light when absolutely no light in the scientific sense is present.

    Similarly when scientists study “pain” they don’t really study what we mean by “pain” in the context of our experience of life, but rather the physical causes of our experience of pain. Unfortunately scientists in general do not study philosophy and are thus more often than not unaware of this huge conceptual difference. So many scientists (as well as people who read their books) are under the impression that by discovering knowledge about the physical causes of experience they advance in solving the problem of consciousness. But physically explaining why, say, LED light looks bluish, or why taking such a drug decreases pain (it decreases the physical causes of pain), says actually zero about consciousness itself.

    [continued in the next post]

    ReplyDelete
  125. [continued from the previous post]

    Burk,

    You write: “Pain is a physical phenomenon, from the rock that hits us on the head, to the brain areas that process the information and ultimately constitute our consciousness of it. OK- that was bare assertion,"

    Yeah, well, if we want to be more blunt, that was bare speculation. We do know of many physical causes of pain, but there is currently no evidence whatsoever that pain itself is a physical phenomenon. Moreover there are some philosophical arguments that show why such evidence can’t possibly exist.

    the issue is that studies like this show that the pain and other experiences we think of as "consciousness" are trackable through the brain, are dissectable, and have mutual effects explicable by those tracked brain pathways.

    To be more precise: Some of the causes of experiences (which by definition are the content of consciousness) are ultimately events that are trackable though the brain, etc. Sure. So? What exactly would you say is there in the actual science (and not in the science as interpreted by naturalists) that to the least degree contradicts theism?

    What is left for a supernatural theory to supply?

    To explain why there is consciousness in the first place, for example.

    Scientific knowledge has advanced very much in the last centuries, so much so that there are virtually no physical phenomena not yet explained by science (the phenomenon of life is an important exception, but everybody expects science to be able to explain that phenomenon too). Nevertheless, in that huge mountain of scientific knowledge there is absolutely nothing that would imply that consciousness should exist. If it weren’t for the fact that we ourselves are conscious, we would deal with somebody who claimed to be conscious as a person deluded beyond salvation.

    For me we have here yet another case where natural science creates trouble for naturalism: The more natural science advances with *nothing* in science implying that consciousness should exist, the less viable scientific naturalism becomes.

    Our nerves (were we to need nerves at all) would lead to the pineal gland, where the disembodied soul would communicate back and forth, constructing our feelings and actions.

    Can you point at even one theologian who claims such? Perhaps one reason you don’t agree with theism is that you don’t know what theism is about.

    The classical theistic understanding for over a thousand years (see “general providence”) is that God continuously upholds in existence as well as orders all of physical reality, from the falling of an apple to the structure of the human brain.

    ReplyDelete
  126. Bernard,

    Thanks for your kind words. I enjoy thinking about God very much, and when one thinks about God one finds that thinking about anything else becomes kind of easy and clear, especially if one does not feel bound to one’s religion’s traditional dogmas but feels free to adopt the best and most beautiful ideas one can find.

    You write: “Here's how the term [“qualia”] may be ill defined. What if qualia, the sensations of consciousness engaging with the world, just are information processing. Not caused by, emerging from, contingent upon, but just the exactly same thing.

    It seems to me you are looking for a definition of qualia using third-person concepts, such as “brain states”, “information processing”, “patterns of activation within the brain”, etc. Now, on naturalism, all knowledge about existents must ultimately be expressed using third-person “objective” concepts, so, on naturalism, there should be a definition of qualia that only uses such third-person concepts. I agree that we don’t now have any such a definition of qualia, but this is a problem for naturalism to solve, and does not imply that the concept of qualia itself is ill-defined.

    So what does the concept of qualia mean? Well, we are all conscious beings and we all know what it means to experience life. Now our experience of life is very complex: It displays a particular multi-dimensional structure (thinking mind, active free will, passive perception colored by beauty, etc), and it also displays a multi-level structure (so, for example, on the issue of thought we have creative thought sitting on abstract thought sitting on logical thought, etc). It’s not easy to identify the structure of conscious experience in such a way that concepts such as “justice” and “morality” naturally fit. What’s more, our experience of life is not a static thing. On the contrary, how we experience life greatly depends on how we live and on what we have done. A striking example here is how it feels like to listen to a spoken language: If we know that language it feels like an easy flow of images where the actual sounds are almost imperceptible. But if we don’t know that language, even though we listen to exactly the same sounds, the whole experience feels like meaningless gibberish. A major fact of our experience of life is then that understanding *transforms* how it is like to experience that one has understood.

    Given the huge complexity of human consciousness, philosophers of the mind realized that they should initially concentrate their attention to the elementary bits of conscious experience, in the same way that when Newton started investigating natural phenomena he did not start with the whole enchilada, say, trying to explain the shape and movement of the clouds. Rather Newton concentrated his attention to the most elementary phenomena he could find, such as how spheres roll down a tilted board. So philosophers came to the idea of using qualia, namely elementary elements of perception, such as experiencing light, or experiencing color, as the basis of their initial research.

    The definition of qualia is thus given using first-person concepts. Is that a clear definition? Well, it certainly seems very clear to me. Even Daniel Dennett, who insists that peoples’ thinking about consciousness is confused, seems to agree, for he writes that the quale is “an unfamiliar term for something that could not be more familiar to each of us: the ways things seem to us”. (So Dennett does not argue that the meaning of “qualia” as defined is vague; rather he argues that, given this clear meaning, qualia do not exist in some significant sense.) There is also objective evidence for the claim that “qualia” is well defined, namely the observational fact that philosophers of the mind, while deeply disagreeing among themselves about almost everything, do communicate effectively with each other when speaking about qualia, produce arguments that use the concept of qualia, argue intelligibly for or against such arguments, etc.

    ReplyDelete
  127. Bernard,

    You write: “So there is no such thing as the experience of seeing red. Every time I look at a red thing the experience is different (I have tried this a lot) and smacks of the act of remembering, recognising, considering, being surprised by etc.

    I agree, the experience of seeing red is not completely stable. Actually, for all we know, perhaps yesterday we experienced red very differently than we do today. Perhaps, other people’s experience of red is very different from ours today, who knows (see the inverted spectrum problem). All of that make the problem of consciousness harder, but does not make the meaning of qualia any less clear. For there is such an elementary experience of the color red I am having right now.

    I think this is a very attractive approach for me, not least because it places free will within the deterministic framework I need before free will can have a truly moral quality.

    If find this statement very perplexing.

    Free will, as normally understood, entails that I could have acted differently than how I in fact did. According to determinism it is not possible for me to have acted differently than how I in fact did, so free will and determinism are mutually incompatible. (There is the notion of “compatibilist free will” but that notion is misleading because it confuses freedom of will with freedom to act according to one’s will.)

    What most troubles me though is that you seem to be saying that unless free will is placed within a deterministic framework one cannot have a truly moral quality. Can you explain how you mean this?

    ReplyDelete
  128. Thanks Dianelos

    I'll address the qualia and free will issues in separate responses to help keep my thinking clear.

    I see how you view the breaking down of red experiences, and the explaining why red experiences are as they are, as separate problems. Qualia seem to mean then (what a slippery concept they are) the ultimate why question of experiences. Yes, but why does accessing a painful memory feel like that, why does taking that drug feel like that, why does having my cortex probed feel like that? So no matter how fine grained an explanation we reach of consciousness, qualia will become the demand for higher resolution. Fair enough. Personal experience makes for compelling data for us all.

    We seem to have canvassed three possible responses. Two take consciousness of this sort to be fundamental. So, one of these says, consciousness just is a quality of this type of activity, in the same way that curvature is a fundamental quality of a circle. It's part of the definition. Case closed. This has the problems of appearing to beg the question (yes but why is it fundamental?), of being counterintuitive and of causing a problem for free will.

    The second fundamental approach is to argue that consciousness is not a fundamental quality of some type of physical activity but rather a fundamental quality of existence itself. And this quality then attaches (I'm unsure of the best verb here) to a particular type of brain activity, giving these their perplexing feeling. This appears to save free will but has problems of interaction (how and why does it only attach to certain patterns of brain activity) and of design (why bother with building brains at all?) And it is similarly question begging (yes but why does consciousness feel like that?)

    Depending upon the precise model of interaction proposed, these apparently contrasting hypotheses are oddly similar. They both say, look, it's just how the universe is.

    The third option is the one that I would call scientific. It says, we don't know yet precisely how brain states lead to consciousness. The world is full of things we don't know. That's what makes science such a magnificent enterprise. But we're learning more and more about how the brain states that may or may not just be consciousness are manifest, so let's keep looking and wondering, and asking pesky questions about what a person means when they say there is something ineffable about redness and as has so often happened in the past, maybe we'll one day wonder how we missed something so damned obvious (cue Eratosthenes, Copernicus, Darwin...) This approach has the problem of leaving the question unanswered which has practical problems for a theory of Will, which we all need.

    Under these options the zombie hunch false apart because by at least one interpretation we are asked to imagine a creature with consciousness that isn't conscious.

    I know which of these options appeals most to me. I think, if somebody were to come up with a really compelling model for conscious/matter interaction I could be swayed. Either way, it does now all appear to come down to free will. (see next explanation)

    Bernard

    ReplyDelete
  129. Hi Dianelos

    I do remember, there is a discussion to be had about Penrose and Haneroff and microtubules etc, I've not forgotten, just become distracted.

    Free Will. I'm a high school drama teacher so will use an example from that world. Two girls, former friends, are involved in a vicious round of destructive rumour mongering and both are angry. I want to help them find their way to a peaceful reconcilation.

    It will require at the very least one girl making the moral choice to take a first step toward peace. The barriers are well constructed, a sense of injustice, emotions veering towards anger and retribution, there's pride, social standing, fear of being weak. It's endless.

    I am likely to begin by offering a statement that will make the first step easier. A 'so the last week has been sort of lousy right?' type thing. So, I'm hoping for a decision to be made and I'm hoping I can influence that decision, through removing barriers, modelling behaviour, giving confidence and helping make clear potential consequences.

    I hope that the girls are capable then of a particular type of freedom. I am hoping they feel free, so the decision feels theirs. And I'm hoping the decision is also a weighing up of, a processing algorithm for, all these many factors. They take their knowledge of the current, past and future. They apply it (I use terms of agency lightly here of course) to their personal and ever evolving heuristic, what Steven Rose might call their lifeline perhaps, and then come up with a decision, in fact a deterministic calculation that feels very different.

    This is a good world to live in. We can make contracts with each other, influence each other, learn from each other etc. And we treat ourselves and each other as if we have free will, and we do, insomuch as we face future choices, options feel open to us (the best we can ever do if time's arrow insists on being so one way, post hoc they're always determined anyway) and so we develop slowly over a lifetime into more magnificent, respectful, moral machines. Wonderful.

    Some worry away: no but it's still determined, as if this is a bad thing. Well, what's the option? At some stage during the application of the moral heuristic, an outside influence exerts itself, so making the decision jump one way. That's awful. It's an amoral decision because the outside influence must be random (if not, if it too follows a code, be it one fixed or evolving, then it could be included in the algorithm and is still therefore part of the deterministic structure). To escape determinism is to escape the very forces and considerations that make choices moral.

    Or so I see it. I just know you're going to have a stimulating response.

    Bernard

    ReplyDelete
  130. Just a quick comment on free will from a Kantian perspective. For Kant, freedom of the will is really about the freedom to decide what laws will govern the will--mechanistic natural laws as manifested in desires and appetites, and moral laws given by reason. Freedom of the will is the power to submit oneself to the latter and be DETERMINED to act by rational laws rather than being DETERMINED to act by appetite, impulse, etc. Freedom of the will is about the capacity to choose one's master, so to speak: reason or inclination. The challenge of making sense of this capacity without saying that one's master is chosen at random is one I continue to wrestle with.

    ReplyDelete
  131. Thanks, Eric-

    That clarifies a lot, since it seems a rather primitive vision of free will. Which seems entirely compatible with naturalism, once we realize that the only thing that we use to over-ride our "base" appetites are our less base appetites, like the desire for a long life and supportive community overriding the desire to rob the local bank. Naturalism in no way restricts our ability to choose our own master, but indicates that we usually choose one that benefits ourselves in some way. Even if we are brainwashed into becoming Moonies, (again based on perceived self-interest, if perhaps posthumous), that hardly bespeaks non-naturalistic free will any more than naturalistic.

    Ask an addict whether they choose their own master, and they will say no. Are they lesser, non-supernatural beings, or just somewhat more trapped in a material decision loop than the rest of us?

    ReplyDelete
  132. Bernard,

    I think the most fundamental question related to consciousness is why it exists in the first place. There is nothing in the phenomena that science studies, or in the knowledge that science accumulates, that even so much as points to the existence of consciousness, for everything can be explained without assuming that consciousness exists. From science’s point of view, consciousness is an entirely unneeded hypothesis. Yet, we can’t fail to notice that it exists, because we ourselves are conscious beings.

    A second question is this: If consciousness is something natural, then there should exist a natural way to detect its presence. More specifically, if consciousness is amenable to scientific investigation then there should be some objective scientific means to detect if something is conscious or not. But nobody knows how to objectively ascertain whether, say, cockroaches (never mind thermostats) are conscious beings or not. Actually nobody has the slightest idea of how such a scientific test might look like. Indeed, we all assume that other people are conscious the way we are, but when pressed we can’t really justify that fundamental belief (hence philosophy’s problem of other minds).

    Now about the three responses you suggest.

    The first one sounds like property dualism, which I too think is the most promising way for a naturalist to think about consciousness. On the other hand I don’t agree with the analogy that “consciousness just is a quality of this type of activity, in the same way that curvature is a fundamental quality of a circle”, because we can clearly see that a circle without curvature is logically impossible, while any physical activity without consciousness is logically possible.

    The second response is the one I believe in. I’d only qualify that consciousness is not just “a” fundamental property of existence, but rather “the” fundamental property of existence.

    The third response sounds like a cop-out to me. It’s too easy to sweep under the rug a conceptual problem one’s ontology suffers from proclaiming that science will in the future solve this problem too. In fact I have two objections with this response. First, that after quite some time and quite some noise including grandiose claims (not to mention some absurd ones, such as that animals and pre-linguistic children are not conscious beings), when one actually looks one finds that there is zero advance in solving the hard problem of consciousness based on science. Up to now it’s all smoke and mirrors, a matter of make-believe, a real world case of the fable about the naked emperor. If this situation continues unchanged for much longer even the most scientifically star struck will start noticing that there is something amiss. Secondly, we know enough about natural science (there is an entirely philosophical field devoted to it) to have a good idea about what science is capable and not capable of producing. As Dustin has pointed out science will only be able to deal with consciousness if the causal closure of the physical does not hold. So, for a naturalist to hope that science will solve the problem of consciousness is tantamount to betting against one of science’s most hallowed principles. But perhaps there will be surprises in the future a la Copernicus or Darwin (as you suggest), who knows.

    What I claim as a fact is this: Naturalism suffers today from several deep conceptual problems, such as the ones related to consciousness, to free will, to morality, and even to modern science and how to make sense of its discoveries. Theism, in comparison, suffers from none of these problems. If it weren’t for theism’s problem of evil I personally cannot imagine why any knowledgeable person would choose naturalism over theism. (Here I think I differ with Eric. I agree that ontology is an interpretation of our experience of life, and I agree that reasonable people can interpret their experience both naturalistically and theistically, but I don’t agree that naturalism and theism are equally reasonable.)

    ReplyDelete
  133. Dear Dianelos Georgoudis,

    You were saying that we know almost nothing about consciousness. You should probably stop reading works by philosophers, and start reading the works of people that do more than speculate, and actually carry on experiments. Neuroscience know a great deal about how we think, the thinking process, consciousness, and many related subjects.

    Cheers! RichGriese.NET

    ReplyDelete
  134. Thank you Dianelos

    At last I see exactly where our difference is, and this is most pleasing. It helps me understand the way you respond to Burk I think. Often you dismiss his contributions as missing the point entirely and I've not found this.

    You have a very strong intuition about what consciousness is and this leads to you saying, for example, that the circle analogy doesn't hold because a thinking functioning brain without consciousness is logically possible (the zombie hunch). Unfortunately this reasoning is circular (sorry about the pun). If consciousness is the thing your intuition tells you it is, a non-physical fact of the world, the the analogy doesn't hold, you are quite right. However, if your intuition is wrong, and consciousness just is a particular type of physical, information processing activity, that it turns out this is the definition of consciousness (which is where my intuition pulls me) then the analogy does hold.

    So, you want to cut the debate off at the pass by asserting your intuition over mine, as a simple fact of the world. Consciousness just is a fact of the world and nobody has even started to solve the hard problem. That is an entirely reasonable belief, and if your statements about consciousness were amended to 'I believe consciousness just is a fact of the world' then option one is absolutely live and makes option three anything but a cop out.

    Now, maybe the reason our initial intuitions are so different is that I am a zombie. Or maybe, as Dustin holds, I am insane to think this way. But by what rule of logic must my intuition be ruled out from the start where your intuition is deemed to be a fact? I know very many people who share my intuition, so I am not just being contrary for the sake of an argument.

    This is the aspect of our discussion that puzzles me deeply. To define consciousness as a thing naturalism can not study, and then conclude naturalism has a problem explaining consciousness, is not a compelling argument.

    This is where I can find much common ground with Eric's position. We can reason our way to both theistic and non-theistic points of view, but because, in the meantime at least, step one is intuitive, it is very hard to dismiss one as less reasonable.

    Bernard

    ReplyDelete
  135. Hi Bernard,

    I agree that understanding why the other person thinks differently is a very useful experience. In this sense I’d like to stay on the “zombie hunch”, because I think this is a powerful argument, and I’d like to understand what you find wrong with it.

    First of all I’d like to dispel an apparent misunderstanding. I do *not* believe that “a thinking functioning brain without consciousness is logically possible”. If I believed that then I would have to think that you, for example, may not be a conscious being. (On what grounds one believes that all normally functioning human brains are conscious is another question, indeed a question which I suspect theists and naturalists would answer quite differently.)

    Chalmers’ zombie argument does not address human brains, or even any brains built of the same kind of matter human brains consist of. Rather Chalmers thinks about a different world than ours, a zombie-world, or z-world for short. In that z-world, there are z-brains consisting of z-atoms with z-electrons flowing through z-synapses. The z-activity of the z-brains in that z-world is radically different from the activity of brain in our world, in that z-brains’ z-activity is not consciousness, whereas brain activity in our world (as per your hypothesis which I here accept as true) is consciousness. But given that the z-activity in the z-world corresponds one to one to activity in our world, there is in the z-world a z-person who has written a play called Hamlet, which is letter by letter identical to the play written in our world, and so on.

    So here is the key question: Do you think that that z-world is logically possible or not? If not, where do you see the logical impossibility? What premises of that z-world would you say are logically inconsistent?

    ReplyDelete
  136. Hi Dianelos

    As a first pass, (and probably I should think about this more but some things spring to mind):

    If z activity in the z world corresponds one to one to ours, then z atoms, electrons etc, while being different than ours, behave in exactly the same way as do the atoms, electrons etc in our world. They experience the same forces, bond in the same way in precisely the same circumstances, follow identical random patterns when quantum tunneling etc. Choose any electron say, and its z equivalent is doing exactly the same thing. To say they are not therefore identical is for me incoherent.

    Another way of putting this is to say imagine a z-world where z circles have z centres and are identical to our circles in every way, except that the points on the z-circumference are not equidistant from the z-centre. Curiously though, the z circle is still perfectly round. I don't see how this equivalent thought experiment establishes that circles did not have to have all their points equidistant from the centre. Instead it establishes that z world allows a contradiction.

    So, if consciousness is the concept we use to describe a particular type of brain pattern, then your z world contains the same contradiction as my circle z world I think.

    And one other way of putting it. Ask a group of twelve years old to draw a circle where not every point on the circumference is equidistant from the centre and a good proportion will try. They might draw a circle with an off-centre centre or a wiggly circumference. They have, in other words, been able to imagine a circle without these qualities. This does not tell us that curvature emerges in a mysterious and ineffable way from equidistance, or that the puzzle of unnecessary but consistently arising equidistance needs to be solved. Rather it tells us that the act of imagination is rather easy if you don't fully understand the terms. (And the term consciousness, although used with great confidence, is something of a movable feast).

    Bernard

    ReplyDelete
  137. Bernard,

    You write: “ Choose any electron say, and its z equivalent is doing exactly the same thing.

    No, each z-electron is not doing exactly the same thing, for if that z-electron is part of a brain-like structure in the z-world it does not form part of consciousness. Given that the z-world is a different reality than ours, it is clearly possible that this should be the case.

    Another way to see it is this: Take any existent in our world and consider its properties. Now imagine another reality identical to ours in all respects except that this one existent lacks one of its properties. That imagined world is clearly logically possible. Chalmers’ z-world is precisely of this kind, namely one where all physical existents lack the properties (or the properties of the properties) that are related to consciousness. - Incidentally, the opposite does not hold; if one *adds* a property to an existent then perhaps one gets an impossible world. But by *removing* a property it’s impossible that a logical contradiction ensues.

    Another way of putting this is to say imagine a z-world where z circles have z centres and are identical to our circles in every way, except that the points on the z-circumference are not equidistant from the z-centre. Curiously though, the z circle is still perfectly round.

    Actually a world in which a figure is such that the points on its circumference are not equidistant from its center, while its circumference is perfectly round – is possible. You only need a world with space bent in such a way that when you measure a distance in one direction you get a different result than when you measure a distance in another direction.

    ReplyDelete
  138. Bernard,

    You write: “ Some worry away: no but it's still determined, as if this is a bad thing.

    Well, if the girl’s will was determined by the physical state of the universe before her parents had even met, indeed by the physical state of the universe at the Big Bang, then, clearly, she is not personally responsible for her actions. Some might say that not to be personally responsible for one’s actions is a good thing, but in fact such an idea makes a mockery of society, not to mention makes a mockery of human relationships.

    The good news is that few beliefs are as certain as that the reality we find ourselves in is *not* deterministic. Modern science has demonstrated this as clearly as can reasonably be expected. For the last 100 years we know that when we shoot a photon towards two slits it will sometimes pass through one and sometimes through the other, even though the initial state of the experiment is identical. So, the initial physical state does not determine the future physical state, and thus determinism is falsified. There are moreover some philosophical arguments why determinism must be false. For example we now know with great confidence that at the big bang the physical state of the universe was of low complexity whereas it now has a very high level of complexity (in the Kolmogorov sense). But the complexity of a system which evolves deterministically does not change, which implies that our universe is not deterministic. Indeed the very idea that at the Big Bang the state of the universe included the play of Hamlet, not to mention the exact shape of every wave of the seas – is too credulity straining to contemplate seriously, especially when there is no good reason whatsoever to suspect that the universe is deterministic. Finally, determinism clearly contradicts our sense of free will. It is true that this sense is only an “intuition”, and intuitions have been known to be wrong, but it’s not like that therefore it is reasonable to doubt all intuitions we have. After all the truth of A=A is based on an intuition too. All of science is based on the principle of induction, which too is only an intuition. So, we may only doubt an intuition if we have very good reason for doing so; and to my knowledge there isn’t even the slightest reason to doubt our intuition of free will.

    ReplyDelete
  139. Hi Dianelos

    Yes, exactly so. We have arrived at out destination. If by definition consciousness is a particular arrangement of particles, then in z world, without consciousness, there is necessarily no such arrangement of particles, because they are defined in advance as the same thing.

    If however, by definition, consciousness is something other than physical arrangements, then in z world everything is the same apart from consciousness, Chalmers' case.

    So, the zombie hunch simply confirms whichever hunch you take into it, it replicates the definition of consciousness you have in mind.

    Both definitions remain logically possible and so we must use our intuition to choose between them, a devilishly difficult task which is why neither side should claim the other is unreasonable.

    And yes, if we extend the definition of circles into non-euclidian spaces then the definition doesn't hold, but the point is it doesn't hold in either world, definitions remain constant across them.

    Hence when either theists or atheists grow frustrated with each other and become sure they are just missing a simple point of logic I become frustrated.

    The same holds for free will. It all comes down to a definition of free will which is much harder than it appears. I agree, the standard definition of free will is incompatible with determinism. Now, whether the universe is deterministic is not the issue for me, rather I am suggesting that this version of free will requires a random or chaotic aspect, and for me this version is amoral which is not what I am looking for in a moral system.

    This is the last day of my school vacation and I'm back to work tomorrow, so while I'll continue to check this site with interest, forgive me if my contributions/responses become sporadic. It has been a privilege to converse with such an open and insightful mind. I have found it pleasingly challenging and for now I remain a happily uncertain, fence sitting agnostic.

    Bernard

    ReplyDelete
  140. Bernard,

    You write: “If by definition consciousness is a particular arrangement of particles,[snip]

    People knew what consciousness is long before they knew about the existence of particles, so the above does not define “consciousness”. Rather, to say that consciousness is a particular arrangement of particles is to make a claim about the world in which we exist. It’s a hypothesis that may be true or false, whereas definitions cannot be true or false.

    And, in any case, truth is not a matter of how one defines concepts. It’s not like one can define one’s way to truth. Suppose for example a theist were to define God as the “rational nature of the universe”, and then use the premise that the universe is rational (that’s how science is at all possible) to “prove” that God exists. Nobody would be very much impressed by such an argument. Likewise, suggested definitions of consciousness designed to make consciousness amenable to scientific research, or suggested definitions of free will designed to make free will compatible with determinism – should not be taken seriously. The only reasonable use of definitions is to clarify what people mean when they use a concept. This is sometimes not easy to do, and can be a philosophically illuminating exercise. But to define a concept in a way that clearly does *not* reflect what people mean when they use it, and is moreover designed to make a point, only reveals a philosopher’s weakness. After all, if a philosopher for some reason does not like what people mean when they use a concept, then she is entirely free to coin a new concept. To redefine a concept can only produce confusion.

    So, the zombie hunch simply confirms whichever hunch you take into it, it replicates the definition of consciousness you have in mind.

    It’s not like we know what “consciousness” means because we have read and agreed with a particular definition. As for hunches, it seems to me that it is those who claim that the z-world is logically impossible without explaining where the logical contradiction lies, who are only expressing a hunch.

    Both definitions remain logically possible and so we must use our intuition to choose between them, a devilishly difficult task which is why neither side should claim the other is unreasonable.

    I don’t see why one has to choose a definition. For example, if Daniel Dennett wants to define “consciousness” in a way that does not reflect what I mean when I use this concept, then I will simply coin a new term, perhaps “d-consciousness” that means what Dennett says, and be done with it. Definitions come for free. A definition cannot be false, only a proposition which uses concepts defined in a particular way can be false.

    And yes, if we extend the definition of circles into non-euclidian spaces then the definition doesn't hold,[snip]

    I see now that perhaps our difference lies with what we mean by “definition”. The way I understand this concept, as long as a definition of a concept makes sense it’s just not possible for a definition not to hold, in this world, or in any possible world.

    Hence when either theists or atheists grow frustrated with each other and become sure they are just missing a simple point of logic I become frustrated.

    The question of whether the z-world is logically possible or not, does not represent a disagreement between theists and atheists. Chalmers himself is an atheist, as are many (perhaps most) philosophers of mind who agree with him.

    I have found it pleasingly challenging and for now I remain a happily uncertain, fence sitting agnostic.

    I too have found our discussion quite stimulating and have learned from it. Still, given you agnosticism, I’d like to suggest the following idea: It is true that holding a false belief can be harmful, but to fail to hold a true belief can also be harmful. What I mean is that agnosticism is not really a secure cognitive position.

    ReplyDelete
  141. Hi Dianelos

    Yes, it is about definitions. So let us discuss this briefly. Over time, our knowledge of concepts grows. Sometimes, we choose to all but abandon a term (the ether, elan vital). other times we simply adjust what we mean by it as more information comes in(atom, species).

    You are arguing that the old, intuitive deifintion of consciousness, will do for now, that is, this thing we know we experience that is more than simply a configuration of particles (or perhaps an arrangemenet of information is a better metaphor. No mind).

    If we are to accept a purely physcial defintion, that thing we knowwe feel that is just an arrangement of physical items in the world, you suggest a new term is used for clarity and consistency. Very well, I shall use Self Referencing Capacity (SRC) for this concept.

    Now return to z-world. In this world there exist creatures that appear to us to be conscious in your sense, but in fact experience no such consciousness. This I accept. However, note that the materialist claim is now simply that we live in z world, for the materialist claim is that no such form of consciousness xists in our world either (this is the implication of eing carefully consistent with our defintion of consciousenss.

    So, the materialist claim is that in our world we have SFC, just like they do in z-world. Again, the point holds, the Chalmers case spits out only hte definition that is taken in. Your case reduces to:

    We all know what consciousness is.
    By this intuition, consciousness isnon physical.
    Taking this intuition to be true, the zombie thought experiment shows this intuition to be true.

    Well, yes.

    Bernard

    ReplyDelete